Как обозначается середина отрезка: Координаты середины отрезка — как найти? Формулы и примеры

Координаты середины отрезка — как найти? Формулы и примеры

Поможем понять и полюбить математику

Начать учиться

Отрезок — понятие, которое знакомо даже первокласснику. С начальной школы мы учились рисовать их, находить длину, складывать и вычитать друг из друга. Сегодня же мы пойдем еще дальше и рассмотрим отрезки на плоскости и даже в пространстве! Звучит впечатляюще, правда? После этой статьи вы сможете с легкостью решать сложные геометрические задачи на нахождение середины отрезка, разберетесь в непростых формулировках и еще больше убедитесь в том, что геометрия прекрасна в своей простоте.

Что такое отрезок

Чтобы изучить эту тему досконально, давайте начнем с самого простого: с определения отрезка.

Отрезок — это прямая, у которой есть начало и конец, или же прямая, которая соединяет две произвольные точки, не совпадающие друг с другом.

Отрезок называют заглавными буквами латинского алфавита по названию конечных точек. Причем можно расставлять буквы в любом порядке: АВ и ВА — равноценные варианты. Рассмотрите иллюстрацию, посчитайте и назовите все отрезки.

Узнай, какие профессии будущего тебе подойдут

Пройди тест — и мы покажем, кем ты можешь стать, а ещё пришлём подробный гайд, как реализовать себя уже сейчас

Что такое середина отрезка

Середина отрезка — это точка, которая находится на равном расстоянии от его концов. Иначе можно сказать так: это точка, которая делит отрезок пополам.

Так, на рисунке ниже D — середина отрезка СК, так как СD = DK. Обратите внимание, как на чертеже обозначаются равные по длине отрезки — мы ставим на них равное количество черточек.

Главный вопрос, который нас сегодня интересует, это координаты середины отрезка.

Координаты — это положение точки в пространстве.

Мы можем рассмотреть отрезок, который лежит на координатной прямой, тогда координата будет одна. В Декартовой системе координат оХУ будет две координаты, причем вначале записывают х, потом у. Например: С (5; 3): К (4; 8). Еще мы можем поместить отрезок в трехмерное пространство, тогда у каждой точки будет три координаты: х, у, z.

Кажется, что чем дальше, тем сложнее, но на самом деле это не совсем так. Хорошая новость: в каждом из случаев мы будем использовать один и тот же принцип, так что вы обязательно во всем разберетесь!

Как найти координаты середины отрезка на координатной прямой

Изобразим горизонтальную координатную прямую оХ и отметим на ней две точки: М и L. Координату точки М запишем как Хм, точки L — соответственно XL. Поставим лежащую на отрезке точку А — середину ML, MA = LA.

Определим координаты точек: Хм = {2}, XL = {8}. Чтобы найти середину отрезка, воспользуемся формулой XA=(XM+XL)/2 и получим:

ХА = (2 + 8)/2 = 5.

Проверим, верна ли формула. Для этого определим координаты середины отрезка графическим методом.Действительно: фактическая координата точки А совпадает со значением, которое мы получили.

Подумайте, взяли ли мы эту формулу случайно или же ее можно вывести. Да, конечно, второй вариант верный — в математике не используют ничего непроверенного. Давайте посмотрим, каким образом можно доказать истинность формулы, тем более, что мы возьмем ее за основу при решении более сложных задач.

  1. Точка А — это середина отрезка, а значит, MA = LA.

  2. Расстояние между точками можно рассчитать через разность модулей их координат: │ХА – ХМ│=│ХL – ХА│.

  3. Преобразуем правую часть, вынесем знак минуса: ХА – ХМ= — (ХА –ХL).

  4. Перенесем ХА в левую часть, а все остальное — в правую: 2ХА= ХL+ ХМ.

  5. Найдем ХА: ХА = (ХL + ХМ)/2.

Вот мы и вывели формулу координат середины отрезков! Чтобы лучше закрепить материал, сделаем пару заданий.

Задача 1

Определите координаты середины отрезка АВ, если ХА = –2, ХB = 10.

Решение

Обозначим точку середины отрезка буквой Т. Тогда Хт = (ХА + ХB)/2 = (–2 + 10)/2 = 4.

Ответ: Хк = {4}.

Задача 2

Определите координаты начала отрезка КМ с серединой в точке Н, если Хм = 5, Хн = 10.

Решение

Вначале запишем формулу для середины отрезка: Хн = (Хк + Хм)/2. Выразим Хк через нее:

Хн = (Хк + Хм)/2,
н = (Хк + Хм),
н – Хм = Хк,
Хк = 2Хн – Хм = 2 * 5 – 10 = 0.

Ответ: Хк = {0}

Как найти середину отрезка на плоскости

В Декартовой системе координат у каждой точки есть две координаты: по оси оХ и оУ. Изобразим отрезок АВ с координатами А (1; 3), В (3; 6) и точкой С — серединой отрезка.

Чтобы найти координаты точки С, мы воспользуемся уже известной нам формулой, но применим ее к каждой координате в отдельности. Вначале рассчитаем Хс:

Хс = (ХА + ХB)/2 = (1 + 3)/2 = 2.

Тогда УC = (УA + УB)/2 = (3 + 6)/2 = 4,5. Значит С (2; 4,5).

Не пугайтесь, если отрезок на чертеже параллелен оси оХ или оУ: мы четко идем по нашему алгоритму и ничего не меняем.

Важно заметить: если отрезок параллелен оси оУ, координаты концов и середины отрезка по оХ будут совпадать, ХА = ХС = ХВ. Если же отрезок параллелен оси оХ, совпадут координаты по оУ: УА = УВ = УС.

И вновь пришло время задачек. Давайте разберем несколько примеров решения.

Задача 3

В системе координат находятся две точки: С (–6; 4) и К (2; 8). Определите координаты середины отрезка.

Решение

Обозначим середину отрезка точкой О. Тогда:

ХО = (ХС + ХК)/2 = (–6 + 2)/2 = –2,
УО = (УС + УК)/2 = (4 + 8)/2 = 6.

Ответ: О (-2; 6).

Задача 4

Дан треугольник с вершинами АВС: А (-2; 4), В (4; 6), С (3; -5). Определите координаты точки М — медианы ВМ.

Решение

Медиана — отрезок, который проведен из вершины треугольника и делит противоположную сторону пополам. А значит, медиана ВМ делит на равные части сторону АС, АМ = МС. Тогда:

ХМ = (ХА + ХС)/2 = (–2 + 3)/2 = 0,5,
УМ = (УА + УС)/2 = (4 – 5)/2 = –0,5.

Ответ: М (0,5; –0,5).

Координаты середины отрезка в пространстве

Вспомните, чем пространство отличается от плоскости. Правильно, третьим измерением! В том смысле, что добавляется еще одна координатная ось: оZ. Как это выглядит, можно посмотреть на рисунке ниже.

При этом формула нахождения середины отрезка остается неизменной. Если мы изобразим в трехмерном пространстве отрезок АВ с серединой в точке С, тогда:

ХС = (ХА + ХВ)/2,
УС = (УА + УВ)/2,
ZС = (ZА + ZВ)/2.

Координаты середины отрезка через радиус-векторы его концов

По сути, этот способ нельзя назвать каким-то новым и уникальным. Он лишь еще раз доказывает истинность формулы координат середины отрезков, только через алгебру. Чтобы разобраться в нем, давайте сначала вспомним определение вектора.

Вектор — это направленный отрезок прямой, то есть отрезок, для которого указано, какая из его граничных точек является началом, а какая — концом.

Векторы — достаточно обширная тема. Чтобы разобраться в ней, не хватит и двух статей. Но сейчас мы с вами будем использовать всего несколько тезисов, которые помогут разобраться в теме.

  1. Векторы можно изображать в системах координат оХУ и оХYZ, т. е. в двумерной и трехмерной.

  2. Координаты начала и конца векторов записывают так же, как и для отрезков: (x; y) и (x; y; z).

  3. Сумму векторов можно найти по методу треугольника или параллелограмма. Картинка ниже поможет вам вспомнить, как ими пользоваться.

Радиус-вектор — вектор, который задает положение точки в пространстве относительно некоторой заранее фиксированной точки — начала координат.

Давайте разберемся, как доказать формулу для нахождения координаты середины отрезка через радиус-векторы его концов. В Декартовой системе координат нарисуем вектор с серединой в точке К. Координаты точки А (ХА; УА; ZА), К (ХК; УК; ZК), С (ХС; УС; ZС).Проведем радиус-векторы , , .

Согласно определению середины отрезка: ОК = ½(ОС + ОА). Координаты векторов ОА, ОК, ОС соответственно равны координатам точек А, К, С, так как координаты точки О (0; 0; 0).

ОА (ХА – 0, УА – 0, ZА – 0) = (ХА; УА; ZА),
ОК (ХК – 0, УК – 0, ZК – 0) = (ХК; УК; ZК,),
ОС (ХС– 0, УС – 0, ZС – 0) = (ХС; УС; ZС).

Тогда запишем равенство ОК = ½(ОС + ОА) через координаты:

ХК = 1/2(ХА + ХО),
УК = 1/2(УА + УО),
ZК = 1/2(ZА + ZО).

Напоследок мы сделаем небольшой перерыв, забудем про формулы и числа. Давайте подумаем, как можно найти середину отрезка, если мы не знаем координат его концов.

Например, нарисуем отрезок на песчаном пляже во время каникул. Определить точные координаты в таком случае будет достаточно сложно, правда? Вряд ли вы взяли с собой в отпуск набор линеек, чтобы вычислить длину отрезка. С подобным заданием вы могли столкнуться и на уроках геометрии, где учитель раздавал вам чистые нелинованные листы бумаги и просил найти середину отрезка без использования линейки.

Сейчас мы обучим вас волшебному методу, приготовьтесь! Все что вам понадобится — это циркуль. Нарисуем на бумаге отрезок АВ любой длины. Поставим иголку циркуля в точку А и начертим окружность с радиусом, равным АВ. Далее повторим действие — прочертим такую же окружность с центром в точке В.

Мы видим, что окружности пересеклись дважды: снизу и сверху. Если соединить эти две точки, эта прямая пересечет наш исходный отрезок ровно в его середине.

Скептики вспомнят наш пример с пляжем и скажут: «Линейку мы с собой в отпуск не берем, но и циркуль ведь тоже! Что вы скажете на это?» А ответим мы вот что: приходите на курсы по профильной математики в Skysmart! Там вы научитесь не только заменять настоящий циркуль на самодельный, но еще подготовитесь к экзаменам, разовьете логику и узнаете много всего интересного. Ждем вас на занятиях!

Шпаргалки для родителей по математике

Все формулы по математике под рукой

Дарья Вишнякова

К предыдущей статье

Логарифмы

К следующей статье

Перпендикулярные прямые

Получите план обучения, который поможет понять и полюбить математику

На вводном уроке с методистом

  1. Выявим пробелы в знаниях и дадим советы по обучению

  2. Расскажем, как проходят занятия

  3. Подберём курс

формула, как по координатам его концов, блок схема

Содержание:

  • Что такое середина отрезка
  • Правила нахождения координат середины отрезка, формулы
    • Середина отрезка на координатной прямой
    • Середина отрезка на плоскости
    • Середина отрезка в пространстве
  • Метод с использованием координат радиус-векторов концов отрезка
  • Примеры решения задач

Содержание

  • Что такое середина отрезка
  • Правила нахождения координат середины отрезка, формулы
    • Середина отрезка на координатной прямой
    • Середина отрезка на плоскости
    • Середина отрезка в пространстве
  • Метод с использованием координат радиус-векторов концов отрезка
  • Примеры решения задач

Что такое середина отрезка

Отрезок — это геометрическая фигура, представляющая собой ограниченный с двух сторон участок прямой.  

Пусть точки A и B не совпадают. Если провести через них прямую, то образуется отрезок AB или BA, который ограничен точками A и B. Данные точки являются концами отрезка.

Длина отрезка — это расстояние между двумя точками, ограничивающими данный отрезок. Длина отрезка AB обозначается как модуль данной геометрической фигуры, то есть |AB|.

Осторожно! Если преподаватель обнаружит плагиат в работе, не избежать крупных проблем (вплоть до отчисления). Если нет возможности написать самому, закажите тут.

Серединой отрезка является такая точка C, принадлежащая отрезку AB, которая расположена в центре данного отрезка, то есть |AC|=|CB|.

Правила нахождения координат середины отрезка, формулы

Середина отрезка на координатной прямой

Предположим, что несовпадающие точки A и B лежат на координатной прямая Ох. Известно, что A и B соответствуют действительные числа xA и xB, а точка С делит AB пополам. Определите координату xC, соответствующую С. 

Так как C — это середина AB, то справедливо следующее равенство:

\(\left|AC\right|=\left|CB\right|\)

Вычислим расстояние между A и C, а также между C и B. Для этого определим модуль разницы их координат. На математическом языке это будет иметь вид:

\(\left|AC\right|=\left|CB\right|\Leftrightarrow\left|x_C-x_A\right|=\left|x_B-x_C\right|\)

Опустим знак модуля и получим справедливость двух выражений:

\(x_C-x_A=x_B-x_C\)

\(x_C-x_A=-\left(x_B-x_C\right)\)

Исходя из первого равенства, получим формулу нахождения xC, согласно которой координата точки С равна половине суммы координат A и B:

\(x_C=\frac{x_A+x_B}2\)

Следствием второго равенства будет следующее утверждение: 

\(x_A=x_B\)

Это противоречит заданным условиям, следовательно, формула определения координат середины отрезка выглядит так:

\(x_C=\frac{x_A+x_B}2\)

Середина отрезка на плоскости

В декартовой системе координат Oxy расположены две точки A(xA,yA) и B(xB,yB), которые не совпадают между собой. Точка C является центром AB. Необходимо произвести вычисление координат xC и yC, соответствующих С.

Пусть произвольные точки А и В лежат на одной координатной прямой, а также не принадлежат прямым, располагающимся перпендикулярно к оси абсцисс или ординат. Опустим от заданных точек A, B, C перпендикуляры на ось x на ось y. Полученные точки пересечения с осями координат Ax, Ay; Bx, By; Cx, C— это проекции исходных точек.

По построению прямые AAx, BBx, CCотносительно друг друга находятся параллельно. Прямые AAy, BBy, CCy не пересекаются, то есть являются параллельными. Согласно равенству AB=BC, далее применим теорему Фалеса и получим:

\(A_xC_x=C_xB_x\)

\(A_yC_y=C_yB_y\)

Это значит, что Cи Cявляются серединами отрезков AxBx и AyBy соответственно. Теперь воспользуемся формулой определения координат середины отрезка на координатной прямой и получим:

\(x_C=\frac{x_A+x_B}2\)

\(y_C=\frac{y_A+y_B}2\)

Данные формулы подходят для вычисления координат середины отрезка в случае его расположения на осях абсцисс и ординат, а также при перпендикулярности одной из них. Следовательно, координаты центра отрезка AB, находящегося в плоскости и ограниченного точками A(xA,yA) и B(xB,yB), вычисляются следующим образом:

\(\left(\frac{x_A+x_B}2,\frac{y_A+y_B}2\right)\)

Середина отрезка в пространстве

Допустим, что в трехмерной системе координат Oxyz любые две точки с соответствующими им координатами A(xA, yA, zA) и B(xB, yB, zB). C(xC, yC, zC) — это центр АВ. Задание заключается в том, чтобы определить xC, yC, zC.

Проведем от исходных точек перпендикуляры к прямым Ox, Oy и Oz. Образовавшиеся точки пересечения с координатными осями — Ax, Ay, Az; Bx, By, Bz; Cx, Cy, C— проекции точек A, B, C на них.

Воспользуемся теоремой Фалеса:

\(\left|A_xC_x\right|=\left|C_xB_x\right|\)

\(\left|A_yC_y\right|=\left|C_yB_y\right|\)

\(\left|A_zC_z\right|=\left|C_zB_z\right|\)

Исходя из полученных равенств следует, что Cx, Cy, C— делят AxBx, AyBy, AzBz пополам, то есть являются серединами перечисленных отрезков. Значит, для определения координат центра AB с концами A(xA,yA,zA) и B(xB,yB,zB) используем формулу:

\(\left(\frac{x_A+x_B}2,\frac{y_A+y_B}2,\;\frac{z_A+z_B}2\right)\)

Метод с использованием координат радиус-векторов концов отрезка

Трактовка векторов в алгебре позволяет составить формулу для расчета координат середины отрезка.

Дано: прямоугольная система координат Oxy, в которой лежат произвольные точки A(xA,yA) и B(xB,yB), а также C, делящая пополам отрезок, ограниченный A и B.

По определению действий над вектором в геометрии:

\((1)\;\overrightarrow{OC}=\frac12\times\left(\overrightarrow{OA}+\overrightarrow{OB}\right)\)

В рассматриваемой ситуации в точке C пересекаются диагонали параллелограмма с основаниями: \(\overrightarrow{OA},\;\overrightarrow{OB} \).

Это значит, что С — это центр диагоналей.

Поскольку координаты радиус вектора совпадают с координатами точки, имеем: \(\overrightarrow{OA}=\left(x_A,\;y_A\right),\;\overrightarrow{OB}=\left(x_B,\;y_B\right) \).

Произведем подстановку в формулу (1):

\(\overrightarrow{OC}=\frac12\times\left(\overrightarrow{OA}+\overrightarrow{OB}\right)=\left(\frac{x_A+x_B}2,\;\frac{y_A+y_B}2\right) \).

Получили формулу определения координат середины отрезка, находящегося в декартовой системе координат:

\(\left(\frac{x_A+x_B}2,\;\frac{y_A+y_B}2\right)\)

По аналогично схеме можно вывести формулу для расчета координат центра отрезка, лежащего в пространстве:

\(\left(\frac{x_A+x_B}2,\frac{y_A+y_B}2,\;\frac{z_A+z_B}2\right)\)

Примеры решения задач

Задача № 1

Дано: в декартовой системе координат имеются точки M(5,4) и N(1,−2). Найти координаты середины отрезка MN.

Решение:

Пусть точка O — центр MN. Тогда вычислим ее координаты, подставив в формулы:

\(x_O=\frac{x_A+x_B}2=\frac{5+1}2=\frac62=3\)

\(y_O=\frac{y_A+y_B}2=\frac{4+\left(-2\right)}2=\frac{4-2}2=\frac22=1\)

Точка O имеет координаты (3,1).

Ответ: (3,1).

Задача № 2

Дано: треугольник ABC лежит в прямоугольной системе координат. Известны координаты его вершин: A(7,3), B(−3,1), C(2,4). Вычислите длину медианы АМ.

Решение:

Поскольку АМ является медианой треугольника ABC, то точка М делит сторону ВС на два равных отрезка, то есть является серединой отрезка ВС. Отсюда можно вычислить координат точки М:

\(x_М=\frac{x_В+x_С}2=\frac{-3+2}2=\frac{-1}2=-0,5\)

\(y_М=\frac{y_В+y_С}2=\frac{1+4}2=\frac52=2,5\)

Теперь, зная координаты начала и конца отрезка АМ, применим формулу нахождения расстояния между точками:

\(AM=\sqrt{\left(x_M-x_A\right)^2+\left(y_M-y_A\right)^2}=\sqrt{\left(-0,5-7\right)^2+\left(-2,5-3\right)^2}=\sqrt{-7,5^2+\left(-5,5\right)^2}=\sqrt{56,25+30,25}=\sqrt{86,5} \).

Ответ: √86,5.

Насколько полезной была для вас статья?

У этой статьи пока нет оценок.

Как найти середину — математика GCSE

Введение

Как найти середину отрезка

Как найти рабочий лист средней точки

Поиск отсутствующей конечной точки

Распространенные заблуждения

Потренируйтесь находить середину вопросы

Как найти середину вопросов GCSE

Контрольный список обучения

Следующие уроки

Все еще застряли?

Индивидуальные занятия по математике, созданные для успеха KS4

Теперь доступны еженедельные онлайн-уроки повторения математики GCSE

Узнать больше

Введение

Как найти середину отрезка

Как найти рабочий лист средней точки

Поиск отсутствующей конечной точки

Распространенные заблуждения

Потренируйтесь находить середину вопросы

Как найти середину вопросов GCSE

Контрольный список обучения

Следующие уроки

Все еще застряли?

Здесь мы узнаем, как найти середину, в том числе найти середину отрезка с использованием декартовых координат и найти отсутствующую конечную точку, когда заданы середина и другая конечная точка.

Существуют также рабочие листы средней точки строки, основанные на экзаменационных вопросах Edexcel, AQA и OCR, а также дополнительные указания о том, что делать дальше, если вы все еще застряли.

Что такое середина линии?

Середина отрезка линии — это точка, которая находится точно посередине между двумя точками. Это одинаковое расстояние от каждой конечной точки отрезка прямой.

Иногда это можно выяснить путем проверки — это проще сделать с положительными целыми числами.

Например, для двух точек (2,2) и (8,6) средняя точка находится ровно посередине между ними и лежит в (5, 4).

Мы видим, что 5 находится посередине между 2 и 8, а 4 — посередине между 2 и 6. В этом может помочь представление числового ряда.

Что такое середина линии?

Середина формулы линии

Если определить середину нелегко или координаты содержат дроби или отрицательные числа, мы можем использовать формулу средней точки.

Если точки \mathrm{A}\left(x_{1}, y_{1}\right) и \mathrm{B}\left(x_{2}, y_{2}\right) являются конечными точками отрезок, то середина отрезка, соединяющего точки A и B, равна \left(\frac{x_{1}+x_{2}}{2}, \frac{y_{1}+y_{2} {2}\справа).

Это выглядит сложно, если записать алгебраически, но в основном мы вычисляем (среднее) среднее значение как значений x, так и значений y.

Мы суммируем две координаты x и делим на 2, чтобы найти координату x средней точки, и суммируем две координаты y и делим на 2, чтобы найти координату y средней точки.

Например, если даны две точки A \ (-1,2) и B \ (2,4), средняя точка (M) находится точно посередине между ними и лежит в (0,5, 3). 9{2}}=\sqrt{13} .

Пошаговое руководство: Теорема Пифагора

Если вы продолжите изучение координатной геометрии на уровне A, вы можете встретить общую формулу расстояния,

d=\sqrt{\left(x_{2}- x_{1}\right)+\left(y_{2}-y_{1}\right)}, где (x_{1},y_{1}) и (x_{2},y_{2}) – координаты двух точек, а d — расстояние между ними.

Как найти середину отрезка

Чтобы найти середину отрезка, соединяющего конечные точки A и B:

  1. Найдите среднее значение \textbf{x} координат двух конечных точек.
  2. Найдите среднее значение \textbf{y} координат двух конечных точек.
  3. Запишите координаты точки.

Объясните, как найти середину отрезка

Рабочий лист с прямым графиком (включает в себя, как найти среднюю точку)

Получите бесплатный рабочий лист о том, как найти среднюю точку из более чем 20 вопросов и ответов по прямолинейному графику. Включает рассуждения и прикладные вопросы.

СКАЧАТЬ БЕСПЛАТНО

Икс

Рабочий лист с прямым графиком (включает в себя, как найти среднюю точку)

Получите бесплатный рабочий лист, как найти среднюю точку из 20+ вопросов и ответов по прямолинейному графику. Включает рассуждения и прикладные вопросы.

СКАЧАТЬ БЕСПЛАТНО

Как найти середину примеров

Пример 1: две положительные целые конечные точки

Найдите середину отрезка, соединяющего точки (0,6) и (4, 10).

  1. Найдите среднее значение \textbf{x} координат двух конечных точек.

\frac{0+4}{2}=\frac{4}{2}=2

2 Найдите среднее значение \textbf{y} координат двух конечных точек.

\frac{6+10}{2}=\frac{16}{2}=8

3 Запишите координаты точки.

(2, \ 8)

В этом случае довольно легко увидеть среднюю точку путем осмотра, особенно при работе на графике.

Пример 2: две положительные целые конечные точки с дробным ответом

Найдите середину отрезка, соединяющего точки (1,5) и (6, 0).

Найдите среднее значение \textbf{x} координат двух конечных точек.

\frac{1+6}{2}=\frac{7}{2}=3,5

Найдите среднее значение \textbf{y} координат двух конечных точек.

\frac{5+0}{2}=\frac{5}{2}=2,5

Запишите координаты точки.

(3.5,\ 2.5)


Обычно допустимо задавать пары координат в виде коротких десятичных дробей в конце. Любые более длинные или повторяющиеся десятичные дроби следует по возможности указывать в виде дробей, не забывая об упрощении ответа.

Пример 3: пары координат, содержащие отрицательные числа

Найдите середину отрезка, соединяющего точки (-2,7) и (4, 10).

Найдите среднее значение \textbf{x} координат двух конечных точек.

\frac{-2+4}{2}=\frac{2}{2}=1

Найдите среднее значение \textbf{y} координат двух конечных точек.

\frac{7+10}{2}=\frac{17}{2}=8,5

Запишите координаты точки.

(1, \ 8,5)


Графически,

Пример 4: координаты, содержащие десятичные дроби

Найдите середину отрезка, соединяющего точки (0,5, 3) и (4, 2,5).

Найдите среднее значение \textbf{x} координат двух конечных точек.

\frac{0.5+4}{2}=\frac{4.5}{2}=2.25

Найдите среднее значение \textbf{y} координат двух конечных точек.

\frac{3+2.5}{2}=\frac{5.5}{2}=2.75

Запишите координаты точки.

(2,25, \ 2,75)


Графически,

Поиск отсутствующей конечной точки

Иногда вам может быть задана одна конечная точка и средняя точка, и вам придется работать с другой конечной точкой.

Чтобы добраться из первой конечной точки (1,3) в среднюю точку (3,7), мы перемещаем 2 в направлении x и 4 в направлении y. Поэтому мы просто повторяем это снова из средней точки, чтобы найти координату другой конечной точки, которая в данном случае будет (5,11).

Как найти отсутствующую конечную точку

Чтобы найти отсутствующую конечную точку при наличии одной конечной точки и средней точки:

  1. Решите, как добраться от заданной конечной точки до средней точки.
  2. Повторите это, чтобы перейти от средней точки к отсутствующей конечной точке.
  3. Запишите координаты отсутствующей конечной точки.

Объясните, как найти отсутствующую конечную точку

Пример 5: поиск отсутствующей конечной точки при наличии одной конечной точки и средней точки

Отрезок соединяет точки A и B и имеет середину M.

A имеет координаты (4, 8), а M имеет координаты (6 , 9).

Найдите координаты точки B.

Решите, как добраться из заданной конечной точки в среднюю.

Чтобы добраться из A в M, мы добавляем 2 к координате x точки A и добавляем 1 к координате y точки A.

Повторите это, чтобы добраться от средней точки до недостающей конечной точки.

Чтобы добраться из M в B, мы добавляем 2 к координате x точки M и добавляем 1 к координате y точки M.

Запишите координату отсутствующей конечной точки.

Следовательно, координаты точки B равны (8,10).

Пример 6: поиск отсутствующей конечной точки с отрицательными координатами

Отрезок соединяет точки A и B и имеет середину M.

A имеет координаты (-9, 4), а M имеет координаты (-6, -1 ).

Найдите координаты точки B.

Решите, как добраться из заданной конечной точки в среднюю.

Чтобы перейти от A к M, мы добавляем 3 к координате x точки A и вычитаем 5 из координаты y точки A.

Повторите это, чтобы перейти от средней точки к отсутствующей конечной точке.

Чтобы добраться из M в B, мы добавляем 3 к координате x точки M и вычитаем 5 из координаты y точки M.

Запишите координату отсутствующей конечной точки.

Следовательно, координаты точки B равны (-3,-6).

Распространенные заблуждения

  • Нахождение среднего значения каждой точки, а не среднего значения \textbf{x} координат и среднее значение \textbf{y} координат

Например, для точек (2, 3) и (5, 7) убедитесь, что вы не делаете \frac{2+3}{2} и \frac{5+7}{2}.

  • Использование формулы средней точки при задании одной конечной точки и средней точки

Если одна конечная точка (3, 4), а средняя точка (6, 2), убедитесь, что вы определили, как добраться от конечной точки до средней точки, и повторите это, а не используйте формулу средней точки.

  • Ошибки вычисления отрицательных чисел

Если вы не уверены в своем ответе, нарисуйте схему и посчитайте шаги.

Практика нахождения средней точки Вопросы

(5, \ 9)

(4, \ 10)

(7, \ 7)

(4, \ 9)

\frac{2+6}{2}=\frac{8}{2}=4, а среднее значение координат y равно \frac{8+12}{2}=\frac{20}{2}=10 .

(1, \ 3)

(6, \ 7.5)

(5.5, \ 7.5)

(6.5, \ 7.5)

Среднее значение координат x равно \frac{4+7}{2}=\frac{11}{ 2}=5,5, а среднее значение координаты Y равно \frac{10+5}{2}=\frac{15}{2}=7,5.

(2, \ 3)

(4, \ 5)

(4, \ 3)

(2, \ 5)

Среднее значение координат x равно \frac{-2+6}{ 2}=\frac{4}{2}=2, а среднее значение координат y равно \frac{8+(-2)}{2}=\frac{6}{2}=3.

(7, \ 7,5)

(7.5, \ 7.25)

(6, \ 8.5)

(7.25, \ 7.25)

Среднее значение координат x равно \frac{3.5+11}{2}=\frac{14. 5}{ 2}=7,25, а среднее значение координаты Y равно \frac{6+8,5}{2}=\frac{14,5}{2}=7,5.

(0, \ 7)

(14, \ 6)

(6.5, \ 3)

(5, \ 2)

Чтобы добраться из А в М, прибавьте 5 к координате x и прибавьте 2 к координате у. Повторите это, чтобы перейти от M к B, поэтому координата B равна (14, \ 6).

(1, \ 8.5)

(4, \ 3)

(-3, \ 13)

(-5, \ 13)

Чтобы получить из A в M, вычтите 4 из координаты x и добавьте 3 к координате y. Повторите это, чтобы перейти от M к B, поэтому координата B равна (-5, \ 13).

Как найти середину вопросов GCSE

1. Отрезок соединяет точки A и B и имеет середину M.

 

A имеет координаты (3, \ -12).

B имеет координаты (-5,\10).

 

Определите координаты точки M.

 

(2 балла)

Показать ответ

Исправьте координаты x или y или найдите среднюю точку с помощью \frac{3+(-5)}{ 2} или \frac{-12+10}{2} .

(1)

(-1, \-1)

(1)

2.

 

а) Запишите координаты точки А.

 

(c) На сетке отметьте крестиком точку (-2, \ 3). Обозначьте это C.

 

(3 балла)

Показать ответ

(a) (-2, \ 1)

(1)

  90

(1)

 

(c)

Точка отмечена в правильном положении.

(1)

3. Отрезок соединяет точки P и Q и имеет середину M.

P имеет координаты (9, \ 5), а M имеет координаты (15, \ 8).

Точка R расположена так, что треугольник PQR является прямоугольным.

Координата y R равна 5. Какова координата x R?

 

(3 балла)

Показать ответ

Метод определения координат конечной точки, например, +6 к координате x или +3 к координате y.

(1)

Q = (21, \ 11)

(1)

x координата R = 21 .

(1)

Учебный контрольный список

Теперь вы научились:

  • Находить середину отрезка, соединяющего две точки
  • Найти отсутствующую конечную точку по одной конечной точке и средней точке

Все еще зависает?

Подготовьте своих учеников KS4 к успешной сдаче выпускных экзаменов по математике с помощью программы Third Space Learning. Еженедельные онлайн-уроки повторения GCSE по математике, которые проводят опытные преподаватели математики.

Узнайте больше о нашей программе повторения GCSE по математике.

Мы используем необходимые и необязательные файлы cookie для улучшения работы нашего веб-сайта. Пожалуйста, ознакомьтесь с нашей Политикой в ​​отношении файлов cookie, чтобы узнать, как мы используем файлы cookie и как управлять вашими настройками файлов cookie или изменять их. Примеры

Середина относится к точке, которая находится посередине линии, соединяющей две точки. Две опорные точки являются конечными точками линии, а средняя точка лежит между двумя точками. Середина делит линию, соединяющую эти две точки, на две равные половины. Кроме того, если провести линию, делящую пополам линию, соединяющую эти две точки, линия проходит через середину.

Формула средней точки используется для нахождения средней точки между двумя точками, координаты которых нам известны. Формула средней точки также используется для нахождения координат конечной точки, если известны координаты другой конечной точки и средней точки. В координатной плоскости, если провести линию, соединяющую две точки (4, 2) и (8, 6), то координаты середины линии, соединяющей эти две точки, равны ({4 + 8}/2, {2 + 6}/2) = (12/2, 8/2) = (6, 4). Давайте узнаем больше о формуле средней точки и различных методах нахождения середины линии.

1. Что такое середина?
2. Формула средней точки
3. Как найти середину?
4. Формулы, относящиеся к средней точке
5. Часто задаваемые вопросы о Midpoint Formula

Что такое средняя точка?

Средняя точка — это точка, лежащая между двумя точками и находящаяся посередине линии, соединяющей эти две точки. Если провести линию, соединяющую две точки, то средней точкой будет точка в середине линии, равноудаленная от двух точек. Для любых двух точек, скажем A и C, середина — это точка B, расположенная посередине между точками A и C. Следовательно, чтобы вычислить середину, мы можем просто измерить длину отрезка и разделить ее на 2.

Обратите внимание, что точка B равноудалена от A и C. Середина существует только для отрезка прямой. Линия или луч не могут иметь середины, потому что линия неопределенна в обоих направлениях, а луч имеет только один конец и поэтому может быть продолжен.

Формула средней точки

Формула средней точки определяется для точек на осях координат. Пусть (x) 1 , (y) 1 и (x) 2 , (y) 2 — концы отрезка. Средняя точка равна половине суммы x-координат двух точек и половине суммы y-координат двух точек. Формула средней точки для вычисления середины отрезка, соединяющего эти точки, может быть представлена ​​​​как

Формула средней точки в математике

Даны две точки A (x) 1 , (y) 1 и B (x) 2 , (y) 2 , средняя точка между A и B определяется как ,

М(х) 3 , (у) 3 = [(х) 1 + (х) 2 ]/2, [(у) 1 + (у) 2 2 ]/2

где M — середина между A и B, а (x) 3 , (y) 3 — ее координаты.

Давайте посмотрим на этот пример и найдем середину двух точек на одномерной оси. Предположим, у нас есть две точки, 5 и 9, на числовой прямой. Середина будет вычисляться как: (5 + 9)/2 = 14/2 = 7. Таким образом, 7 является серединой 5 и 9. , (х) 1 , (у) 1 и (х) 2 , (у) 2 . Для любого линейного отрезка средняя точка находится посередине между двумя его конечными точками. Выражение для координаты x средней точки равно [(x) 1 + (x) 2 ]/2, что является средним значением координат x. Точно так же выражение для координаты y имеет вид [(y) 1 + (y) 2 ]/2, что является средним значением координат y.

Таким образом, формула для средней точки имеет вид пример, чтобы увидеть применение формулы средней точки.

Пример: Используя формулу средней точки, найдите среднюю точку между точками X(5, 3) и Y(7, 8).

Решение: Пусть M будет серединой между X и Y.

M = ((5 + 7)/2, (3 + 8)/2) = (6, 11/2)

Следовательно, координаты середины между X и Y равны (6, 11/2).

Как найти середину?

Далее, на основе точек и значений их координат используются следующие два метода для нахождения середины линии, соединяющей две точки.

Метод 1: Если отрезок прямой вертикальный или горизонтальный, то, разделив длину на 2 и считая это значение от любой из конечных точек, мы получим середину отрезка прямой. Посмотрите на рисунок, показанный ниже. Координаты точек A и B равны (-3, 2) и (1, 2) соответственно. Длина горизонтальной линии \(\overline{AB}\) равна 4 единицам. Половина этой длины составляет 2 единицы. Перемещение на 2 единицы из точки (-3, 2) даст (-1, 2). Итак, (-1, 2) — это середина \(\overline{AB}\).

Метод 2: Другой способ найти среднюю точку — использовать формулу средней точки. Координаты точек A и B равны (-3, -3) и (1, 4) соответственно. Используя формулу средней точки, мы имеем: ({-3 + 1}/2, {-3 + 4}/2) = (-2/2, 1/2) = (-1,1/2).

Метод 3: Один из способов найти середину прямой, заданной на плоскости, — это построение. Мы можем использовать конструкцию циркуля и линейки, чтобы сначала построить линзу, используя дуги окружности одинакового (и достаточно большого) радиуса с центрами в двух конечных точках, а затем соединив вершины линзы (две точки, где дуги пересекаются). Точка пересечения линии, соединяющей бугры и отрезок, является серединой отрезка.

Вот пример поиска координат конечной точки по средней точке и координатам другой конечной точки.

Пример: Середина R между точками P и Q имеет координаты (4, 6). Если координаты Q равны (8, 10), то каковы координаты точки P? Решите его, используя формулу средней точки.

Решение:
Пусть координата x точки P равна m, а координата y точки P равна n.

Р = (м, н)
Q = (8, 10)
R = (4, 6)
Используя формулу средней точки,

R = ((m + 8)/2, (n + 10)/2) = (4, 6)
Решение для m,
(м + 8)/2 = 4
м + 8 = 8
m = 0

Решение для n,
(n + 10)/2 = 6
п + 10 = 12
n = 2

Следовательно, координаты P равны (0, 2).

Важные примечания относительно средней точки:

Следующие точки являются важными свойствами средних точек.

  • Середина делит отрезок в равном соотношении, то есть 1:1.
  • Середина делит отрезок на две равные части.
  • Биссектриса отрезка пересекает его середину.

Формула средней точки включает вычисления отдельно для x-координаты точек и y-координаты точек. Кроме того, вычисления точек между двумя заданными точками также включают в себя аналогичные вычисления координаты x и координаты y заданных точек. Следующие две формулы тесно связаны с формулой средней точки.

  • Центроид формулы треугольника
  • Формула сечения

Центроид треугольника Формула

Точка пересечения медиан треугольника называется центром треугольника. Медиана — это линия, соединяющая вершину с серединой противоположной стороны треугольника. Центроид делит медиану треугольника в отношении 2:1. Для треугольника с вершинами (x) 1 , (y) 1 , (x) 2 , (y) 2 , (x) 3 , (y) 3 формула для нахождения координат центра тяжести треугольника выглядит следующим образом.

Формула сечения

Формула сечения помогает найти координаты любой точки, которая находится на линии, соединяющей две точки. Далее, отношение, в котором точка разделила линию, соединяющую две заданные точки, необходимо, чтобы узнать координаты точки. Точка может располагаться между точками или в любом месте за точками, но на одной линии. Формула сечения для нахождения координат точки, которая делит линию, соединяющую точки (x) 1 , (у) 1 и (х) 2 , (у) 2 в соотношении m:n выглядит следующим образом. Знак плюс используется в формуле для нахождения координат точки, которая делит точки внутри, а знак минус используется, если точка делится снаружи.

☛ Связанные темы:

Ознакомьтесь с еще несколькими интересными статьями, которые содержат дополнительные концептуальные идеи, вращающиеся вокруг формулы средней точки.

  • Медиана треугольника
  • Геометрия
  • Равноудаленный
  • Калькулятор средней точки
  • Центроид Формула

Часто задаваемые вопросы о Midpoint Formula

Что такое формула средней точки в координатной геометрии?

Формула средней точки в координатной геометрии определяется как формула для нахождения центральной точки прямой с использованием координат ее концов. Формула средней точки используется для нахождения половины пути, то есть точки, которая делит прямую на две равные части.

Что означает середина?

Средняя точка определяется как точка, которая находится в середине линии, соединяющей две точки. Это точка, которая равноудалена от обеих конечных точек и, таким образом, делит отрезок пополам.

Как использовать формулу средней точки?

Формула средней точки очень проста, когда дело доходит до ее применения.

  • Шаг 1: Определите сегмент линии или две конечные точки.
  • Шаг 2: Найдите их координаты.
  • Шаг 3: Сложите координаты x обеих конечных точек и разделите на 2.
  • Шаг 4: Сложите координаты Y обеих конечных точек и разделите на 2.
  • Шаг 5: Запишите значения, полученные на шагах 3 и 4, при упоминании координат любой точки.

☛ Также проверьте: Вы можете попробовать этот калькулятор средней точки, чтобы проверить результат, полученный для средней точки отрезка линии — Калькулятор средней точки

Что такое формула средней точки в Word?

Для середины линии, соединяющей две точки, координаты которых заданы, формула середины точки словесно может быть описана как половина суммы x-координат двух точек и половина суммы y-координат две точки.

Почему важна формула средней точки?

Формула средней точки имеет различные применения в реальной жизни, например, для целей строительства и т. д. Она имеет важное значение в геометрии, например,

  • Нахождение координат центра тяжести треугольника.
  • Нахождение медианы треугольника.
  • Нахождение середины отрезка.

Может ли середина быть дробью?

Да, среднее значение также может быть дробным. Это в основном зависит от числового значения двух точек. Средняя точка представляет собой сумму числового значения двух точек, деленную на 2. Для таких точек, как -4 и 5 на числовой прямой, средняя точка равна +1/2.

Как рассчитать среднюю точку?

Середину можно найти по формуле [(x) 1 + (x) 2 ]/2, [(y) 1 + (y) 2 ]/2. Здесь (x) 1 , (y) 1 и (x) 2 , (y) 2 — координаты двух точек, а середина — точка, лежащая на равном расстоянии между этими двумя точками.

Может ли середина быть нулем?

Средняя точка может быть нулевой. Это зависит от значения двух точек. Для двух точек на числовой прямой в точках со значениями -4 и 4 середина равна 0. А для двух точек, таких как (-2, 5) и (2, -5), середина равна (0 , 0).

Что такое середина линии?

Середина линии — это точка, равноудаленная от концов линии и в середине линии. Если конечные точки линии (x) 1 , (y) 1 и (x) 2 , (y) 2 , то формула для середины линии {[(x ) 1 + (x) 2 ]/2, [(y) 1 + (y) 2 ]/2}

Что такое середина кривой?

Середина кривой — это середина наибольшей хорды, которую можно провести для кривой. Середина окружности — это середина ее наибольшей хорды, которая является диаметром окружности.

Что такое середина треугольника?

Середина треугольника является центром тяжести треугольника. Центроид – это точка пересечения медиан треугольника.

Производная c: Производная константы (числа) (c)’

Задания по теме «Производная» с ответами

Геометрический смысл производной, касательная

1. За­да­ние 7 № 27503. На ри­сун­ке изоб­ражён гра­фик функ­ции y=f(x) и ка­са­тель­ная к нему в точке с абс­цис­сой x0. Най­ди­те зна­че­ние про­из­вод­ной функ­ции f(x) в точке x0.

Ре­ше­ние.

Зна­че­ние про­из­вод­ной в точке ка­са­ния равно уг­ло­во­му ко­эф­фи­ци­ен­ту ка­са­тель­ной, ко­то­рый в свою оче­редь равен тан­ген­су угла на­кло­на дан­ной ка­са­тель­ной к оси абс­цисс. По­стро­им тре­уголь­ник с вер­ши­на­ми в точ­ках A (1; 2), B (1; −4), C(−2; −4). Угол на­кло­на ка­са­тель­ной к оси абс­цисс будет равен углу ACB:

 

 

Ответ: 2.

2. За­да­ние 7 № 512495. На ри­сун­ке по­ка­зан гра­фик дви­же­ния ав­то­мо­би­ля по марш­ру­ту. На оси абс­цисс от­кла­ды­ва­ет­ся время (в часах), на оси ор­ди­нат — прой­ден­ный путь (в ки­ло­мет­рах). Най­ди­те сред­нюю ско­рость дви­же­ния ав­то­мо­би­ля на дан­ном марш­ру­те. Ответ дайте в км/ч.

Ре­ше­ние.

Чтобы найти сред­нюю ско­рость, не­об­хо­ди­мо прой­ден­ное рас­сто­я­ние раз­де­лить на время про­хож­де­ния: км/ч

 

Ответ: 50.

3. За­да­ние 7 № 27504. На ри­сун­ке изоб­ражён гра­фик функ­ции y=f(x) и ка­са­тель­ная к нему в точке с абс­цис­сой x0. Най­ди­те зна­че­ние про­из­вод­ной функ­ции f(x) в точке x0.

Ре­ше­ние.

Зна­че­ние про­из­вод­ной в точке ка­са­ния равно уг­ло­во­му ко­эф­фи­ци­ен­ту ка­са­тель­ной, ко­то­рый в свою оче­редь равен тан­ген­су угла на­кло­на дан­ной ка­са­тель­ной к оси абс­цисс. По­стро­им тре­уголь­ник с вер­ши­на­ми в точ­ках A (2; 4), B (2; 2), C (−6; 2). Угол на­кло­на ка­са­тель­ной к оси абс­цисс будет равен углу ACB. По­это­му

 

 

 

Ответ: 0,25.

4. За­да­ние 7 № 27505. На ри­сун­ке изоб­ражён гра­фик функ­ции y=f(x) и ка­са­тель­ная к нему в точке с абс­цис­сой x0. Най­ди­те зна­че­ние про­из­вод­ной функ­ции f(x) в точке x0.

Ре­ше­ние.

Зна­че­ние про­из­вод­ной в точке ка­са­ния равно уг­ло­во­му ко­эф­фи­ци­ен­ту ка­са­тель­ной, ко­то­рый в свою оче­редь равен тан­ген­су угла на­кло­на дан­ной ка­са­тель­ной к оси абс­цисс. По­стро­им тре­уголь­ник с вер­ши­на­ми в точ­ках A (−2; −9), B (−2; −3), C (−5; −3). Угол на­кло­на ка­са­тель­ной к оси абс­цисс будет равен углу, смеж­но­му с углом ACB. По­это­му

 

.

 

Ответ: −2.

5. За­да­ние 7 № 27506. На ри­сун­ке изоб­ражён гра­фик функ­ции y=f(x) и ка­са­тель­ная к нему в точке с абс­цис­сой x0. Най­ди­те зна­че­ние про­из­вод­ной функ­ции f(x) в точке x0.

Ре­ше­ние.

Зна­че­ние про­из­вод­ной в точке ка­са­ния равно уг­ло­во­му ко­эф­фи­ци­ен­ту ка­са­тель­ной, ко­то­рый в свою оче­редь равен тан­ген­су угла на­кло­на дан­ной ка­са­тель­ной к оси абс­цисс. По­стро­им тре­уголь­ник с вер­ши­на­ми в точ­ках A (2; −2), B (2; 0), C (−6; 0). Угол на­кло­на ка­са­тель­ной к оси абс­цисс будет равен углу, смеж­но­му с углом ACB:

 

.

 

Ответ: − 0,25.

6. За­да­ние 7 № 505379. На ри­сун­ке изоб­ра­же­ны гра­фик функ­ции y = f(x) и ка­са­тель­ная к нему в точке с абс­цис­сой Най­ди­те зна­че­ние про­из­вод­ной функ­ции f(x) в точке

Ре­ше­ние.

Зна­че­ние про­из­вод­ной в точке ка­са­ния равно уг­ло­во­му ко­эф­фи­ци­ен­ту ка­са­тель­ной, ко­то­рый в свою оче­редь равен тан­ген­су угла на­кло­на дан­ной ка­са­тель­ной к оси абс­цисс. По­стро­им тре­уголь­ник с вер­ши­на­ми в точ­ках A (−2; 13), B (−2; 3), C (6; 3). Угол на­кло­на ка­са­тель­ной к оси абс­цисс будет равен углу, смеж­но­му с углом ACB:

 

.

 

Ответ: −1,25.

7. За­да­ние 7 № 40129. На ри­сун­ке изоб­ра­жен гра­фик функ­ции y=f(x). Пря­мая, про­хо­дя­щая через на­ча­ло ко­ор­ди­нат, ка­са­ет­ся гра­фи­ка этой функ­ции в точке с абс­цис­сой 8. Най­ди­те f’(8).

 

Ре­ше­ние.

По­сколь­ку ка­са­тель­ная про­хо­дит через на­ча­ло ко­ор­ди­нат, ее урав­не­ние имеет вид y = kx. Эта пря­мая про­хо­дит через точку (8; 10), по­это­му 10 = 8 · k, от­ку­да k = 1,25. По­сколь­ку уг­ло­вой ко­эф­фи­ци­ент ка­са­тель­ной равен зна­че­нию про­из­вод­ной в точке ка­са­ния, по­лу­ча­ем: f’(8) = 1,25.

 

Ответ: 1,25.

8. За­да­ние 7 № 317539. На ри­сун­ке изоб­ражён гра­фик функ­ции и во­семь точек на оси абс­цисс: , , , , . В сколь­ких из этих точек про­из­вод­ная функ­ции по­ло­жи­тель­на?

 

Ре­ше­ние.

По­ло­жи­тель­ным зна­че­ни­ям про­из­вод­ной со­от­вет­ству­ет ин­тер­ва­лы, на ко­то­рых функ­ция воз­рас­та­ет. На них лежат точки Таких точек 4.

 

Ответ:4.

9. За­да­ние 7 № 317540. На ри­сун­ке изоб­ражён гра­фик функ­ции и две­на­дцать точек на оси абс­цисс: , , , , . В сколь­ких из этих точек про­из­вод­ная функ­ции от­ри­ца­тель­на?

 

Ре­ше­ние.

От­ри­ца­тель­ным зна­че­ни­ям про­из­вод­ной со­от­вет­ству­ют ин­тер­ва­лы, на ко­то­рых функ­ция убы­ва­ет. В этих ин­тер­ва­лах лежат точки Таких точек 7.

 

 

Ответ:7.

10. За­да­ние 7 № 317543. На ри­сун­ке изоб­ра­жен гра­фик функ­ции и от­ме­че­ны точки −2, −1, 1, 2. В какой из этих точек зна­че­ние про­из­вод­ной наи­боль­шее? В от­ве­те ука­жи­те эту точку.

 

Ре­ше­ние.

Зна­че­ние про­из­вод­ной в точке ка­са­ния равно уг­ло­во­му ко­эф­фи­ци­ен­ту ка­са­тель­ной, ко­то­рый в свою оче­редь равен тан­ген­су угла на­кло­на дан­ной ка­са­тель­ной к оси абс­цисс. Про­из­вод­ная по­ло­жи­тель­на в точ­ках −2 и 2. Угол на­кло­на (и его тан­генс) явно боль­ше в точке −2.

Ответ:−2.

11. За­да­ние 7 № 40130. На ри­сун­ке изоб­ра­жен гра­фик про­из­вод­ной функ­ции . Най­ди­те абс­цис­су точки, в ко­то­рой ка­са­тель­ная к гра­фи­ку па­рал­лель­на пря­мой или сов­па­да­ет с ней.

Ре­ше­ние.

Зна­че­ние про­из­вод­ной в точке ка­са­ния равно уг­ло­во­му ко­эф­фи­ци­ен­ту ка­са­тель­ной. По­сколь­ку ка­са­тель­ная па­рал­лель­на пря­мой или сов­па­да­ет с ней, она имеет уг­ло­вой ко­эф­фи­ци­ент рав­ный 2 и Оста­лось найти, при каких про­из­вод­ная при­ни­ма­ет зна­че­ние 2. Ис­ко­мая точка .

 

Ответ: 5.

12. За­да­ние 7 № 40131. На ри­сун­ке изоб­ра­жен гра­фик про­из­вод­ной функ­ции . Най­ди­те абс­цис­су точки, в ко­то­рой ка­са­тель­ная к гра­фи­ку па­рал­лель­на оси абс­цисс или сов­па­да­ет с ней.

Ре­ше­ние.

Зна­че­ние про­из­вод­ной в точке ка­са­ния равно уг­ло­во­му ко­эф­фи­ци­ен­ту ка­са­тель­ной. По­сколь­ку ка­са­тель­ная па­рал­лель­на оси абс­цисс или сов­па­да­ет с ней, она имеет вид , и её уг­ло­вой ко­эф­фи­ци­ент равен 0. Сле­до­ва­тель­но, мы ищем точку, в ко­то­рой уг­ло­вой ко­эф­фи­ци­ент, равен нулю, а зна­чит, и про­из­вод­ная равна нулю. Про­из­вод­ная равна нулю в той точке, в ко­то­рой её гра­фик пе­ре­се­ка­ет ось абс­цисс. По­это­му ис­ко­мая точка .

 

Ответ: -3.

13. За­да­ние 7 № 27485. Пря­мая па­рал­лель­на ка­са­тель­ной к гра­фи­ку функ­ции . Най­ди­те абс­цис­су точки ка­са­ния.

 

Ре­ше­ние.

Зна­че­ние про­из­вод­ной в точке ка­са­ния равно уг­ло­во­му ко­эф­фи­ци­ен­ту ка­са­тель­ной. По­сколь­ку ка­са­тель­ная па­рал­лель­на пря­мой их уг­ло­вые ко­эф­фи­ци­ен­ты равны. По­это­му абс­цис­са точки ка­са­ния на­хо­дит­ся из урав­не­ния :

 

.

Ответ: 0,5.

14. За­да­ние 7 № 27486. Пря­мая яв­ля­ет­ся ка­са­тель­ной к гра­фи­ку функ­ции . Най­ди­те абс­цис­су точки ка­са­ния.

Ре­ше­ние.

Усло­вие ка­са­ния гра­фи­ка функ­ции и пря­мой задаётся си­сте­мой тре­бо­ва­ний:

 

 

В нашем слу­чае имеем:

 

Про­вер­ка под­ста­нов­кой по­ка­зы­ва­ет, что пер­вый ко­рень не удо­вле­тво­ря­ет, а вто­рой удо­вле­тво­ря­ет урав­не­нию (*). По­это­му ис­ко­мая абс­цис­са точки ка­са­ния −1.

 

Ответ: −1.

15. За­да­ние 7 № 119972. Пря­мая y = 3x + 1 яв­ля­ет­ся ка­са­тель­ной к гра­фи­ку функ­ции ax2 + 2x + 3. Най­ди­те a.

Ре­ше­ние.

Пря­мая яв­ля­ет­ся ка­са­тель­ной к гра­фи­ку функ­ции в точке тогда и толь­ко тогда, когда од­но­вре­мен­но и . В нашем слу­чае имеем:

 

 

Ис­ко­мое зна­че­ние а равно 0,125.

 

Ответ: 0,125.

 

При­ве­дем дру­гое ре­ше­ние.

По смыс­лу за­да­чи a ≠ 0, а зна­чит, гра­фик за­дан­ной функ­ции — па­ра­бо­ла. Ка­са­тель­ная к па­ра­бо­ле (а также и к ги­пер­бо­ле) имеет с ней един­ствен­ную общую точку. По­это­му не­об­хо­ди­мо и до­ста­точ­но, чтобы урав­не­ние ax2 + 2x + 3 = 3x + 1 имело един­ствен­но ре­ше­ние. Для этого дис­кри­ми­нант 1 − 8а урав­не­ния ax2x + 2 = 0 дол­жен быть равен нулю, от­ку­да .

16. За­да­ние 7 № 119974. Пря­мая яв­ля­ет­ся ка­са­тель­ной к гра­фи­ку функ­ции . Най­ди­те .

 

Ре­ше­ние.

Усло­вие ка­са­ния гра­фи­ка функ­ции и пря­мой задаётся си­сте­мой тре­бо­ва­ний:

 

 

В нашем слу­чае имеем:

 

 

Ответ: 7.

17. За­да­ние 7 № 119973. Пря­мая яв­ля­ет­ся ка­са­тель­ной к гра­фи­ку функ­ции . Най­ди­те , учи­ты­вая, что абс­цис­са точки ка­са­ния боль­ше 0.

 

Ре­ше­ние.

Усло­вие ка­са­ния гра­фи­ка функ­ции и пря­мой задаётся си­сте­мой тре­бо­ва­ний:

 

В нашем слу­чае имеем:

 

 

По усло­вию абс­цис­са точки ка­са­ния по­ло­жи­тель­на, по­это­му x = 0,5, от­ку­да b = −33.

 

Ответ: −33.

2. За­да­ние 14 № 506286. На ри­сун­ке изоб­ражён гра­фик функ­ции, к ко­то­ро­му про­ве­де­ны ка­са­тель­ные в четырёх точ­ках.

 

 

Ниже ука­за­ны зна­че­ния про­из­вод­ной в дан­ных точ­ках. Поль­зу­ясь гра­фи­ком, по­ставь­те в со­от­вет­ствие каж­дой точке зна­че­ние про­из­вод­ной в ней.

 

ТОЧКИ

 

ЗНА­ЧЕ­НИЯ ПРО­ИЗ­ВОД­НОЙ

А) K

Б) L

В) M

Г) N

 

1) −4

2) 3

3)

4) −0,5

 

За­пи­ши­те в ответ цифры, рас­по­ло­жив их в по­ряд­ке, со­от­вет­ству­ю­щем бук­вам:

А

Б

В

Г

 

 

 

 

По­яс­не­ние.

Зна­че­ние про­из­вод­ной в точке равно уг­ло­во­му ко­эф­фи­ци­ен­ту ка­са­тель­ной, про­ведённой в этой точке. Он по­ло­жи­те­лен и мень­ше 1, если ка­са­тель­ная на­кло­не­на к по­ло­жи­тель­но­му на­прав­ле­нию оси абс­цисс под углом мень­ше 45°; боль­ше 1, если угол на­кло­на боль­ше 45°, но мень­ше 90°; … По­это­му в точке К уг­ло­вой ко­эф­фи­ци­ент по­ло­жи­те­лен и боль­ше 1, в точке L — от­ри­ца­те­лен и мень­ше −1, М — от­ри­ца­те­лен и боль­ше −1, N — по­ло­жи­те­лен и мень­ше 1. Таким об­ра­зом, по­лу­ча­ем со­от­вет­ствие А — 2, Б — 1, В — 4 и Г — 3.

 

Ответ: 2143.

5. За­да­ние 14 № 506377. На ри­сун­ке изоб­ражён гра­фик функ­ции y = f(x). Числа a, b, c, d и e за­да­ют на оси x че­ты­ре ин­тер­ва­ла. Поль­зу­ясь гра­фи­ком, по­ставь­те в cоот­вет­ствие каж­до­му ин­тер­ва­лу ха­рак­те­ри­сти­ку функ­ции или её про­из­вод­ной.

 

 

Ниже ука­за­ны зна­че­ния про­из­вод­ной в дан­ных точ­ках. Поль­зу­ясь гра­фи­ком, по­ставь­те в со­от­вет­ствие каж­дой точке зна­че­ние про­из­вод­ной в ней.

 

ТОЧКИ

 

ЗНА­ЧЕ­НИЯ ПРО­ИЗ­ВОД­НОЙ

А) (a; b)

Б) (b; c)

В) (c; d)

Г) (d; e)

 

1) про­из­вод­ная от­ри­ца­тель­на на всём ин­тер­ва­ле

2) про­из­вод­ная по­ло­жи­тель­на в на­ча­ле ин­тер­ва­ла и от­ри­ца­тель­на в конце ин­тер­ва­ла

3) функ­ция от­ри­ца­тель­на в на­ча­ле ин­тер­ва­ла и по­ло­жи­тель­на в конце ин­тер­ва­ла

4) про­из­вод­ная по­ло­жи­тель­на на всём ин­тер­ва­ле

 

 

 

 

По­яс­не­ние.

Если функ­ция воз­рас­та­ет, то про­из­вод­ная по­ло­жи­тель­на и на­о­бо­рот.

На ин­тер­ва­ле (a;b) про­из­вод­ная по­ло­жи­тель­на вна­ча­ле ин­тер­ва­ла и от­ри­ца­тель­на в конце, по­то­му что функ­ция вна­ча­ле воз­рас­та­ет, а потом убы­ва­ет.

На ин­тер­ва­ле (b;c) про­из­вод­ная от­ри­ца­тель­на, по­то­му что функ­ция убы­ва­ет.

На ин­тер­ва­ле (c;d) функ­ция от­ри­ца­тель­на в на­ча­ле ин­тер­ва­ла и по­ло­жи­тель­на в конце ин­тер­ва­ла.

На ин­тер­ва­ле (d;e) про­из­вод­ная по­ло­жи­тель­на, по­то­му что функ­ция воз­рас­та­ет.

Таким об­ра­зом, по­лу­ча­ем со­от­вет­ствие А — 2, Б — 1, В — 3 и Г — 4.

 

Ответ: 2134.

12. За­да­ние 14 № 506722. На ри­сун­ке изоб­ражён гра­фик функ­ции y = f(x) и от­ме­че­ны точки K, L, M и N на оси x. Поль­зу­ясь гра­фи­ком, по­ставь­те в со­от­вет­ствие каж­дой точке ха­рак­те­ри­сти­ку функ­ции и её про­из­вод­ной.

 

Ниже ука­за­ны зна­че­ния про­из­вод­ной в дан­ных точ­ках. Поль­зу­ясь гра­фи­ком, по­ставь­те в со­от­вет­ствие каж­дой точке зна­че­ние про­из­вод­ной в ней.

 

ТОЧКИ

 

ХА­РАК­ТЕ­РИ­СТИ­КИ ФУНК­ЦИИ ИЛИ ПРО­ИЗ­ВОД­НОЙ

А) K

Б) L

В) M

Г) N

 

1) функ­ция по­ло­жи­тель­на, про­из­вод­ная по­ло­жи­тель­на

2) функ­ция от­ри­ца­тель­на, про­из­вод­ная от­ри­ца­тель­на

3) функ­ция по­ло­жи­тель­на, про­из­вод­ная равна 0

4) функ­ция от­ри­ца­тель­на, про­из­вод­ная по­ло­жи­тель­на

По­яс­не­ние.

В точке K функ­ция от­ри­ца­тель­на, про­из­вод­ная по­ло­жи­тель­на.

В точке L функ­ция по­ло­жи­тель­на, про­из­вод­ная равна 0.

В точке M функ­ция от­ри­ца­тель­на, про­из­вод­ная от­ри­ца­тель­на.

В точке N функ­ция по­ло­жи­тель­на, про­из­вод­ная по­ло­жи­тель­на

Таким об­ра­зом, по­лу­ча­ем со­от­вет­ствие А — 4, Б — 3, В — 2 и Г — 1.

24. За­да­ние 14 № 509699. На ри­сун­ке изоб­ражён гра­фик функ­ции y = f(x) . Точки a, b, c, d и e за­да­ют на оси Ox ин­тер­ва­лы. Поль­зу­ясь гра­фи­ком, по­ставь­те в со­от­вет­ствие каж­до­му ин­тер­ва­лу ха­рак­те­ри­сти­ку функ­ции или её про­из­вод­ной.

 

 

Поль­зу­ясь гра­фи­ком, по­ставь­те в со­от­вет­ствие каж­до­му ин­тер­ва­лу вре­ме­ни ха­рак­те­ри­сти­ку дви­же­ния ав­то­мо­би­ля на этом ин­тер­ва­ле.

 

ИН­ТЕР­ВА­ЛЫ ВРЕ­МЕ­НИ

 

ХА­РАК­ТЕ­РИ­СТИ­КИ

А) ( a; b)

Б) (b; c )

В) (c; d )

Г) ( d ; e)

 

1) Зна­че­ния функ­ции по­ло­жи­тель­ны в каж­дой точке ин­тер­ва­ла.

2) Зна­че­ния про­из­вод­ной функ­ции по­ло­жи­тель­ны в каж­дой точке ин­тер­ва­ла.

3) Зна­че­ния функ­ции от­ри­ца­тель­ны в каж­дой точке ин­тер­ва­ла.

4) Зна­че­ния про­из­вод­ной функ­ции от­ри­ца­тель­ны в каж­дой точке ин­тер­ва­ла.

 

По­яс­не­ние.

Если функ­ция воз­рас­та­ет, то про­из­вод­ная по­ло­жи­тель­на и на­о­бо­рот.

На ин­тер­ва­ле (a;b) зна­че­ния функ­ции по­ло­жи­тель­ны в каж­дой точке ин­тер­ва­ла.

На ин­тер­ва­ле (b;c) зна­че­ния про­из­вод­ной функ­ции от­ри­ца­тель­ны в каж­дой точке ин­тер­ва­ла.

На ин­тер­ва­ле (c;d) зна­че­ния функ­ции от­ри­ца­тель­ны в каж­дой точке ин­тер­ва­ла.

На ин­тер­ва­ле (d;e) зна­че­ния про­из­вод­ной функ­ции по­ло­жи­тель­ны в каж­дой точке ин­тер­ва­ла.

Таким об­ра­зом, по­лу­ча­ем со­от­вет­ствие А — 1, Б — 4, В — 3 и Г — 2.

 

НК РФ Статья 220.1. Налоговые вычеты при переносе на будущие периоды убытков от операций с ценными бумагами и операций с производными финансовыми инструментами \ КонсультантПлюс

Подготовлены редакции документа с изменениями, не вступившими в силу

КонсультантПлюс: примечание.

Ст. 220.1 (в ред. ФЗ от 23.11.2020 N 372-ФЗ) применяется в отношении доходов, полученных начиная с 01.01.2021.

НК РФ Статья 220.1. Налоговые вычеты при переносе на будущие периоды убытков от операций с ценными бумагами и операций с производными финансовыми инструментами

(в ред. Федерального закона от 03.07.2016 N 242-ФЗ)

(см. текст в предыдущей редакции)

(введена Федеральным законом от 25.11.2009 N 281-ФЗ)

1. При определении размера налоговых баз в соответствии с пунктом 2.3 статьи 210 настоящего Кодекса налогоплательщик имеет право на получение налоговых вычетов при переносе на будущие периоды убытков от операций с ценными бумагами, обращающимися на организованном рынке ценных бумаг, и с производными финансовыми инструментами, обращающимися на организованном рынке.

(в ред. Федеральных законов от 03.07.2016 N 242-ФЗ, от 23.11.2020 N 372-ФЗ)

(см. текст в предыдущей редакции)

Перенос на будущие периоды убытков от операций с ценными бумагами и операций с производными финансовыми инструментами осуществляется в соответствии с пунктом 16 статьи 214. 1 настоящего Кодекса.

(в ред. Федеральных законов от 28.12.2010 N 395-ФЗ, от 03.07.2016 N 242-ФЗ)

(см. текст в предыдущей редакции)

2. Налоговые вычеты при переносе на будущие периоды убытков от операций с ценными бумагами и операций с производными финансовыми инструментами предоставляются:

(в ред. Федерального закона от 03.07.2016 N 242-ФЗ)

(см. текст в предыдущей редакции)

1) в размере сумм убытков, полученных от операций с ценными бумагами, обращающимися на организованном рынке ценных бумаг. Указанный налоговый вычет предоставляется в размере сумм убытков, фактически полученных налогоплательщиком от операций с ценными бумагами, обращающимися на организованном рынке ценных бумаг, в предыдущих налоговых периодах в пределах размера налоговой базы по таким операциям;

2) в размере сумм убытков, полученных от операций с производными финансовыми инструментами, обращающимися на организованном рынке. Указанный налоговый вычет предоставляется в размере сумм убытков, фактически полученных налогоплательщиком от операций с производными финансовыми инструментами, обращающимися на организованном рынке, в предыдущих налоговых периодах в пределах размера налоговой базы по таким операциям.

(в ред. Федерального закона от 03.07.2016 N 242-ФЗ)

(см. текст в предыдущей редакции)

3. Размер налоговых вычетов, предусмотренных настоящей статьей, определяется исходя из сумм убытков, полученных налогоплательщиком в предыдущих налоговых периодах (в течение 10 лет считая с налогового периода, за который производится определение налоговой базы). При этом при определении размера налогового вычета в налоговом периоде, за который производится определение налоговой базы, суммы убытков, полученных налогоплательщиком в течение более чем одного налогового периода, учитываются в той очередности, в которой понесены соответствующие убытки.

Размер предусмотренных настоящей статьей налоговых вычетов, исчисленный в текущем налоговом периоде, не может превышать величину налоговой базы, определенную по соответствующим операциям в этом налоговом периоде. При этом суммы убытков налогоплательщика, не учтенные при определении размера налогового вычета, могут быть учтены при определении размера налогового вычета в следующих налоговых периодах с учетом положений настоящей статьи.

4. Для подтверждения права на налоговые вычеты при переносе на будущие периоды убытков от операций с ценными бумагами и операций с производными финансовыми инструментами налогоплательщик представляет документы, подтверждающие объем понесенного убытка в течение всего срока, когда он уменьшает налоговую базу текущего налогового периода на суммы ранее полученных убытков.

(в ред. Федерального закона от 03.07.2016 N 242-ФЗ)

(см. текст в предыдущей редакции)

5. Налоговый вычет предоставляется налогоплательщику при представлении налоговой декларации в налоговые органы по окончании налогового периода.

(в ред. Федерального закона от 27.07.2010 N 229-ФЗ)

(см. текст в предыдущей редакции)

6. Положения настоящей статьи не применяются к отрицательному финансовому результату (убытку), полученному по операциям, учитываемым на индивидуальном инвестиционном счете.

(п. 6 введен Федеральным законом от 28.11.2015 N 327-ФЗ)

Реализация производной на C/C++

спросил

Изменено 4 года, 9 месяцев назад

Просмотрено 71к раз

Как производная f(x) обычно рассчитывается программно для обеспечения максимальной точности?

Я применяю метод Ньютона-Рафсона, и он требует получения производной функции.

  • c++
  • c
  • алгоритм
  • математика
  • числовые методы

3

Я согласен с @erikkallen, что (f(x + h) - f(x - h)) / 2 * h — это обычный подход для численного приближения производных. Однако получить правильный размер шага h немного сложно.

Ошибка аппроксимации в ( f(x + h) - f(x - h)) / 2 * h уменьшается как h становится меньше, что говорит о том, что вы должны взять h как можно меньше. Но по мере того, как ч становится меньше, ошибка вычитания с плавающей запятой увеличивается, поскольку числитель требует вычитания почти равных чисел. Если ч слишком мало, вы можете потерять большую точность при вычитании. Таким образом, на практике вы должны выбрать не слишком маленькое значение ч , которое минимизирует комбинацию аппроксимации ошибки и числовой ошибки . 9-8 .

Дополнительные сведения см. в этих примечаниях по выбору размера шага для дифференциальных уравнений.

6

Newton_Raphson предполагает, что у вас могут быть две функции f(x) и ее производная f'(x). Если у вас нет производной в виде функции и вам нужно оценить производную от исходной функции, вам следует использовать другой алгоритм поиска корня.

Поиск корней Википедии дает несколько предложений, как и любой текст числового анализа. 9{-5}. Исправьте эти значения, если вы знаете f»(x) или f»'(x).

5

 fprime(x) = (f(x+dx) - f(x-dx)) / (2*dx)
 

для небольшого дх.

1

Что вы знаете о f(x)? Если у вас есть только f как черный ящик, единственное, что вы можете сделать, это численно аппроксимировать производную. Но точность обычно не очень.

Ты можешь многое лучше, если вы сможете коснуться кода, вычисляющего f. Попробуйте «автоматическое дифференцирование». Для этого есть несколько хороших библиотек. Применив немного библиотечной магии, вы можете легко преобразовать свою функцию во что-то, что автоматически вычисляет производную. Простой пример C++ см. в исходном коде в этом обсуждении на немецком языке.

Вы определенно хотите принять во внимание предложение Джона Кука относительно выбора h, но обычно не хотите использовать центрированную разность для аппроксимации производной. Основная причина заключается в том, что это требует дополнительной оценки функции, если вы используете прямую разницу, т.е.

 f'(x) = (f(x+h) - f(x))/ч
 

Тогда вы получите значение f(x) бесплатно, потому что вам нужно вычислить его уже для метода Ньютона. Это не так уж важно, когда у вас есть скалярное уравнение, но если x — вектор, то f'(x) — матрица (якобиан), и вам нужно будет выполнить n дополнительных вычислений функции, чтобы аппроксимировать ее. с использованием метода центрированных разностей.

В дополнение к приведенному выше ответу Джона Д. Кука важно не только учитывать точность с плавающей запятой, но и надежность функции f(x). Например, в финансах часто случается, что f(x) на самом деле является симуляцией Монте-Карло, а значение f(x) имеет некоторый шум. Использование очень маленького размера шага может в этих случаях сильно ухудшить точность производной.

Обычно шум сигнала больше всего влияет на качество производной. Если у вас есть шум в f(x), Савтицкий-Голей — отличный алгоритм сглаживания, который часто используется для вычисления хороших производных. В двух словах, SG подбирает полином локально к вашим данным, затем этот полином можно использовать для вычисления производной.

Пол

Зарегистрируйтесь или войдите в систему

Зарегистрируйтесь с помощью Google

Зарегистрироваться через Facebook

Зарегистрируйтесь, используя адрес электронной почты и пароль

Опубликовать как гость

Электронная почта

Обязательно, но не отображается

Опубликовать как гость

Электронная почта

Требуется, но не отображается

cython — «Сохранение небезопасной производной временной ссылки Python на C» при попытке доступа к указателю структуры

Задавать вопрос

спросил

Изменено 4 года, 10 месяцев назад

Просмотрено 528 раз

Я хочу использовать библиотеку, которая дает мне динамический массив. Структура динамического массива имеет свойство void* _heap_ptr , которое дает начало массива.

После создания списка я хочу получить доступ к этому указателю в cython (чтобы сделать копию массива). Но я не могу получить элемент указателя из структуры.

Вот мой pyx:

 cimport src.clist as l
деф основной():
    cdef l.ptr_list базовый_список
    cdef целое я = 42
    basic_list = l.create_list_size(sizeof(i), 100)
    l.list_add_ptr(базовый_список, &i)
    cdef int* обр;
    arr = основной_список._heap_ptr
    для я в диапазоне (1):
        печать (обр[я])
 9-------------------------------------------------- ----------
src/test.pyx:14:20: Хранение небезопасной производной C временной ссылки на Python
 

И мой pxd:

 cdef extern из "src/list.h":
    ctypedef структура _list:
        пустота* _heap_ptr
    структура ctypedef ptr_list:
        проходить
    ptr_list create_list_size (размер size_t, длина int)
    list_destroy (ptr_list this_list)
    void* list_at_ptr (ptr_list this_list, индекс int)
    list_add_ptr(ptr_list this_list, недействительное* значение)
 

Как исправить код? Почему это происходит? Из моих исследований это сообщение об ошибке появляется, если вы забыли объявить что-то как C (например, используйте malloc, а не libc.

Метод крамера слау: Метод Крамера для решения СЛАУ: алгоритм, примеры задач

Метод Крамера – теорема, примеры решений

Габриель Крамер – математик, создатель одноименного метода решения систем линейных уравнений

Габриель Крамер – известный математик, который родился 31 июля 1704 года. Ещё в детстве Габриель поражал своими интеллектуальными способностями, особенно в области математики. Когда Крамеру было 20 лет, он устроился в Женевский университет штатным преподавателем.

Во время путешествия по Европе Габриель познакомился с математиком Иоганном Бернулли, который и стал его наставником. Только благодаря Иоганну, Крамер написал много статей по геометрии, истории математики и философии. А в свободное от работы время изучал математику всё больше и больше.

Наконец-то наступил тот день, когда Крамер нашёл способ, при помощи которого можно было бы легко решать не только лёгкие, но и сложные системы линейных уравнений.

В 1740 году у Крамера были опубликованы несколько работ, где доступно изложено решение квадратных матриц и описан алгоритм, как находить обратную матрицу. Далее математик описывал нахождения линейных уравнений разной сложности, где можно применить его формулы. Поэтому тему так и назвали: «Решение систем линейных уравнений методом Крамера».

Учёный умер в возрасте 48 лет (в 1752 году). У него было ещё много планов, но, к сожалению, он так и не успел их осуществить.

О чем статья

Вывод формулы Крамера

Пусть дана система линейных уравнений такого вида:

 

где , , – неизвестные переменные, – это числовые коэффициенты, в – свободные члены.

Решением СЛАУ (систем линейных алгебраических уравнение) называются такие неизвестные значения  при которых все уравнения данной системы преобразовываются в тождества.

Если записать систему в матричном виде, тогда получается , где

В данной главной матрице находятся элементы, коэффициенты которых при неизвестных переменных,

Это матрица-столбец свободных членов, но есть ещё матрица-столбец неизвестных переменных:

После того, когда найдутся неизвестные переменные, матрица и будет решением системы уравнений, а наше равенство преобразовывается в тождество. . Если умножить , тогда . Получается: .

Если матрица – невырожденная, то есть, её определитель не равняется нулю, тогда у СЛАУ есть только одно единственное решение, которое находится при помощи  метода Крамера.

Как правило, для решения систем линейных уравнений методом Крамера, нужно обращать внимания на два свойства, на которых и основан данный метод:

1. Определитель квадратной матрицы равняется сумме произведений элементов любой из строк (столбца) на их алгебраические дополнения:

, здесь – 1, 2, …, n; – 1, 2, 3, …, n.

2. Сумма произведений элементов данной матрицы любой строки или любого столбца на алгебраические дополнения определённых элементов второй строки (столбца) равняется нулю:

,

,

где – 1, 2, …, n; – 1, 2, 3, …, n. .

Итак, теперь можно найти первое неизвестное . Для этого необходимо умножить обе части первого уравнения системы на , части со второго уравнения на , обе части третьего уравнения на и т. д. То есть, каждое уравнение одной системы нужно умножать на определённые алгебраические дополнения первого столбца матрицы :

 

Теперь прибавим все левые части уравнения, сгруппируем слагаемые, учитывая неизвестные переменные и приравняем эту же сумму к сумме правых частей системы уравнения:

.

Можно обратиться к вышеописанным свойствам определителей и тогда получим:

И предыдущее равенство уже выглядит так:

Откуда и получается .

Аналогично находим . Для этого надо умножить обе части уравнений на алгебраические дополнения, которые находятся во втором столбце матрицы .

 

Теперь нужно сложить все уравнения системы и сгруппировать слагаемые при неизвестных переменных. Для этого вспомним свойства определителя:

Откуда получается .

Аналогично находятся все остальные неизвестные переменные.

Если обозначить:

тогда получаются формулы, благодаря которым находятся неизвестные переменные методом Крамера:

, , .

Замечание.

Тривиальное решение при может быть только в том случае, если система уравнений является однородной . И действительно, если все свободные члены нулевые, тогда и определители равняются нулю, так как в них содержится столбец с нулевыми элементами. Конечно же, тогда формулы , , дадут

Нужна помощь в написании работы?

Мы — биржа профессиональных авторов (преподавателей и доцентов вузов). Наша система гарантирует сдачу работы к сроку без плагиата. Правки вносим бесплатно.

Заказать работу

Метод Крамера – теоремы

Прежде чем решать уравнение , необходимо знать:

  1. теорему аннулирования;
  2. теорему замещения.

Теорема замещения

Теорема

Сумма произведений алгебраических дополнений любого столбца (строки) на произвольные числа равняется новому определителю, в котором этими числами заменены соответствующие элементы изначального определителя, что отвечают данным алгебраическим дополнениям.

Например,

=

где – алгебраические дополнения элементов первого столбца изначального определителя:

 

Теорема аннулирования

Теорема

Сумма произведений элементов одной строки (столбца) на алгебраические дополнения соответствующих элементов другой строки (столбца) равняется нулю.

Например:

Алгоритм решения уравнений методом Крамера

Метод Крамера – простой способ решения систем линейных алгебраических уравнений. Такой вариант применяется исключительно к СЛАУ, у которых совпадает количество уравнений с количеством неизвестных, а определитель отличен от нуля.

Итак, когда выучили все этапы, можно переходить к самому алгоритму решения уравнений методом Крамера. Запишем его последовательно:

Шаг 1. Вычисляем главный определитель матрицы

и необходимо убедиться, что определитель отличен от нуля (не равен нулю).

Шаг 2. Находим определители

Это и есть определители матриц, которые получались из матрицы при замене столбцов на свободные члены.

Шаг 3. Вычисляем неизвестные переменные

Теперь вспоминаем формулы Крамера, по которым вычисляем корни (неизвестные переменные):

, , .

Шаг 4. Выполняем проверку

Выполняем проверку решения при помощи подстановки в исходную СЛАУ. Абсолютно все уравнения в системе должны быть превращены в тождества. Также можно высчитать произведение матриц . Если в итоге получилась матрица, которая равняется , тогда система решена правильно. Если же не равняется , скорей всего в одном из уравнений есть ошибка.

Давайте для начала рассмотрим систему двух линейных уравнений, так как она более простая и поможет понять, как правильно использовать правило Крамера. Если вы поймёте простые и короткие уравнения, тогда сможете решить более сложные системы трёх уравнений с тремя неизвестными.

Кроме всего прочего, есть системы уравнений с двумя переменными, которые решаются исключительно благодаря правилу Крамеру.

Итак, дана система двух линейных уравнений:

Для начала вычисляем главный определитель (определитель системы):

Значит, если , тогда у системы или много решений, или система не имеет решений. В этом случае пользоваться правилом Крамера нет смысла, так как решения не получится и нужно вспоминать метод Гаусса, при помощи которого данный пример решается быстро и легко.

В случае, если , тогда у система есть всего одно решение, но для этого необходимо вычислить ещё два определителя и найти корни системы.

и

Часто на практике определители могут обозначаться не только , но и латинской буквой , что тоже будет правильно.

Корни уравнения найти просто, так как главное, знать формулы:

,

Так как мы смогли решить систему двух линейных уравнений, теперь без проблем решим и систему трёх линейных уравнений, а для этого рассмотрим систему:

(1)

Здесь алгебраические дополнения элементов – первый столбец . Во время решения не забывайте о дополнительных элементах. Итак, в системе линейных уравнений нужно найти три неизвестных – при известных других элементах.

Создадим определитель системы из коэффициентов при неизвестных:

Умножим почленно каждое уравнение соответственно на , , – алгебраические дополнения элементов первого столбца (коэффициентов при ) и прибавим все три уравнения. Получаем:

Согласно теореме про раскладывание, коэффициент при равняется . Коэффициенты при и будут равняться нулю по теореме аннулирования. Правая часть равенства по теореме замещения даёт новый определитель, который называется вспомогательным и обозначается

После этого можно записать равенство:

(2)

Для нахождения и перемножим каждое из уравнений изначальной системы в первом случае соответственно на , во втором – на и прибавим. Впоследствии преобразований получаем:

 где

,

Если  , тогда в результате получаем формулы Крамера:

= , = , =

Порядок решения однородной системы уравнений

Отдельный случай – это однородные системы:

(3)

Среди решений однородной системы могут быть, как нулевые решения , так и решения отличны от нуля.

Теорема

Если определитель однородной системы (3) отличен от нуля , тогда у такой системы может быть только одно решение.

Действительно, вспомогательные определители , как такие у которых есть нулевой столбец и поэтому, за формулами Крамера

Теорема

Если у однородной системы есть отличное от нуля решение, тогда её определитель  равняется нулю 

Действительно, пусть одно из неизвестных , например, , отличное от нуля. Согласно с однородностью  Равенство (2) запишется: . Откуда выплывает, что 

Примеры решения методом Крамера

Рассмотрим на примере решение методом Крамера и вы увидите, что сложного ничего нет, но будьте предельно внимательно, так как частые ошибки в знаках приводят к неверному ответу.

Пример 1

Задача

Решить систему линейных уравнений методом Крамера:

Решение

Первое, что надо сделать – вычислить определитель матрицы:

Как видим, , поэтому по теореме Крамера система имеет единственное решение (система совместна). Далее нужно вычислять вспомогательные определители. Для этого заменяем первый столбец из определителя на столбец свободных коэффициентов. Получается:

Аналогично находим остальные определители:

И проверяем:

,

.

Ответ

, .

Пример 2

Задача

Решить систему уравнений методом Крамера:

Решение

Находим определители:

Ответ

= =   = =   = =

Проверка

* = * = =

* = * = =

* = * = =

Уравнение имеет единственное решение.

Ответ

=   =   =

Пример 3

Задача

Решить систему методом Крамера

Решение

Как вы понимаете, сначала находим главный определитель:

Как мы видим, главный определитель не равняется нулю и поэтому система имеет единственное решение. Теперь можно вычислить остальные определители:

При помощи формул Крамера находим корни уравнения:

, , .

Чтобы убедиться в правильности решения, необходимо сделать проверку:

Как видим, подставив в уравнение решённые корни, у нас ответ получился тот же, что и в начале задачи, что говорит о правильном решении уравнений.

Ответ

Система уравнений имеет единственное решение: , , .

Есть примеры, когда уравнение решений не имеет. Это может быть в том случае, когда определитель системы равен нулю, а определители при неизвестных неравны нулю. В таком случае говорят, что система несовместна, то есть не имеет решений. Посмотрим на следующем примере, как такое может быть.

Пример 4

Задача

Решить систему линейных уравнений методом Крамера:

Решение

Как и в предыдущих примерах находим главный определитель системы:

В этой системе определитель равняется нулю, соответственно, система несовместна и определенна или же несовместна и не имеет решений. Чтобы уточнить, надо найти определители при неизвестных так, как мы делали ранее:

Мы нашли определители при неизвестных и увидели, что все они не равны нулю. Поэтому система несовместна и не имеет решений.

Ответ

Система не имеет решений.

Часто в задачах на системы линейных уравнений встречаются такие уравнения, где есть не одинаковые буквы, то есть, кроме букв, которые обозначают переменные, есть ещё и другие буквы и они обозначают некоторое действительное число. На практике к таким уравнениям и системам уравнений приводят задачи на поиск общих свойств каких-либо явлений и предметов. То есть, изобрели вы какой-либо новый материал или устройство, а для описания его свойств, общих независимо от величины или количества экземпляра, нужно решить систему линейных уравнений, где вместо некоторых коэффициентов при переменных – буквы. Давайте и рассмотрим такой пример.

Пример 5

Задача

Решить систему линейных уравнений методом Крамера:

Решение

В этом примере – некоторое вещественное число. Находим главный определитель:

Находим определители при неизвестных:

Используя формулы Крамера, находим:

, .

Ответ

,

.

И наконец, мы перешли к самой сложной системе уравнений с четырьмя неизвестными. Принцип решения такой же, как и в предыдущих примерах, но в связи с большой системой можно запутаться. Поэтому рассмотрим такое уравнение на примере.

Пример 6

Задача

Найти систему линейных уравнений методом Крамера:

Здесь действуют система определителей матрицы высших порядков, поэтому вычисления и формулы рассмотрены в этой теме, а мы сейчас просто посчитаем систему уравнений с четырьмя неизвестными.

Решение

В изначальном определители из элементов второй строки мы отнимали элементы четвёртой строки, а из элементов третьей строки отнимались элементы четвёртой строки, которые умножались на 2. Также отнимали из элементов четвёртой строки элементы первой строки, умноженной на два. Преобразования первоначальных определителей при трёх первых неизвестных произведены по такой же схеме. Теперь можно находить определители при неизвестных:

Для преобразований определителя при четвёртом неизвестном из элементов первой строки мы вычитали элементы четвёртой строки.

Теперь по формулам Крамера нужно найти:

,

,

,

.

Ответ

Итак, мы нашли корни системы линейного уравнения:

,

,

,

.

Подведём итоги

При помощи метода Крамера можно решать системы линейных алгебраических уравнений в том случае, если определитель не равен нулю. Такой метод позволяет находить определители матриц такого порядка, как на благодаря формулам Крамера, когда нужно найти неизвестные переменные. Если все свободные члены нулевые, тогда их определители равны нулю, так как в них содержится столбец с нулевыми элементами. И конечно же, если определители равняются нулю, лучше решать систему методом Гаусса, а не Крамера, только тогда ответ будет верный.

Рекомендуем почитать для общего развития

Анкилов А. В. Высшая математика, ч. 1: учеб. Пособие/П. А. Вельмисов, Ю. А. Решетников – Ульяновск – 2011 – 252 с.

Письменный Д. – Конспект лекций по высшей математике: учеб. для вузов/Письменный Д. – М. 2006 – 602 с.

Решение методом Крамера в Excel

Метод Крамера в Excel 2003 (XLS)

Метод Крамера в Excel от 2007 (XLSX)

Как решить систему линейных уравнений методом Крамера:

Sign in

Password recovery

Восстановите свой пароль

Ваш адрес электронной почты

MicroExcel.ru Математика Алгебра Метод Крамера для решения СЛАУ

В данной публикации мы рассмотрим формулировку и формулу метода Крамера, а также разберем пример практической задачи для закрепления теоретического материала.

  • Теорема Крамера
  • Пример задачи

Теорема Крамера

Система линейных уравнений может решаться несколькими способами, и один из них – это метод Крамера (или теорема), который так назван в честь швейцарского математика Габриэля Крамера.

Формулировка теоремы:

Если определитель матрицы, соответствующий квадратной СЛАУ не равняется нулю, значит система является совместной и имеет одно решение, которое можно найти следующим образом:

  • Δ – определитель матрицы системы;
  • Δi – определитель, в котором место столбца i расположен столбец правых частей.

Примечание: если определитель матрицы, соответствующей системе, равняется нулю, то она может быть и совместной, и несовместной.

Пример задачи

Давайте с помощью метода Крамера решим систему линейных уравнений ниже:

Решение

1. Для начала представим заданное СЛАУ в виде расширенной матрицы A.

2. Определитель матрицы (без учета столбца свободных членов) не равен нулю, значит у системы есть решение, причем единственное.

3. Считаем определитель, заменив первый столбец на третий (т.е. для корня x).

4. Теперь аналогичным образом вычислим определитель, подставив вместо второго столбца третий (для y).

5. Остается только воспользоваться формулой выше, чтобы найти x и y.

Ответ: x = 2, y = 5.

ЧАЩЕ ВСЕГО ЗАПРАШИВАЮТ

Таблица знаков зодиака

Нахождение площади трапеции: формула и примеры

Нахождение длины окружности: формула и задачи

Римские цифры: таблицы

Таблица синусов

Тригонометрическая функция: Тангенс угла (tg)

Нахождение площади ромба: формула и примеры

Нахождение объема цилиндра: формула и задачи

Тригонометрическая функция: Синус угла (sin)

Геометрическая фигура: треугольник

Нахождение объема шара: формула и задачи

Тригонометрическая функция: Косинус угла (cos)

Нахождение объема конуса: формула и задачи

Таблица сложения чисел

Нахождение площади квадрата: формула и примеры

Что такое тетраэдр: определение, виды, формулы площади и объема

Нахождение объема пирамиды: формула и задачи

Признаки подобия треугольников

Нахождение периметра прямоугольника: формула и задачи

Формула Герона для треугольника

Что такое средняя линия треугольника

Нахождение площади треугольника: формула и примеры

Нахождение площади поверхности конуса: формула и задачи

Что такое прямоугольник: определение, свойства, признаки, формулы

Разность кубов: формула и примеры

Степени натуральных чисел

Нахождение площади правильного шестиугольника: формула и примеры

Тригонометрические значения углов: sin, cos, tg, ctg

Нахождение периметра квадрата: формула и задачи

Теорема Фалеса: формулировка и пример решения задачи

Сумма кубов: формула и примеры

Нахождение объема куба: формула и задачи

Куб разности: формула и примеры

Нахождение площади шарового сегмента

Что такое окружность: определение, свойства, формулы

Обратная стратегия Джима Крамера

Скотт Грайс

На протяжении всей истории благодаря сочетанию технологических инноваций, создававших необходимую инфраструктуру, и достижений в отношении среднего уровня финансового образования существовала очень четкая и отчетливая тенденция Инвестиции на фондовом рынке постепенно отдаляются от профессионалов и становятся все более популярными, поэтому отныне это торговля, которой занимаются обычные люди.

Раньше, за годы до рождения и популяризации социальных сетей, именно крупные медиа-конгломераты извлекли выгоду из этого процесса, запустив множество телевизионных шоу, посвященных финансам и инвестициям, которые почти преимущественно были ориентированы как по стилю, так и по содержанию на розничных инвесторов. . Одним из пионеров в этой области был CNBC, и одно имя стало почти синонимом финансового телевидения: Джим Крамер.

В сегодняшней статье мы будем использовать альтернативные наборы данных, которые мы собрали по основным телешоу CNBC за последние пару лет, чтобы проанализировать его решения по выбору акций, сравнить его результаты с рынком и, в конечном итоге, посмотреть, как мы можем собрать урожай. силу больших данных, чтобы попытаться вывести альфа-версию на рынок. Мы представляем вам нашу новую торговую стратегию, основанную на данных, собранных на хосте CNBC, где мы создали торговый механизм, который намеренно инвертирует большинство самых популярных рекомендаций Крамера по акциям, которые до сих пор доказали свою эффективность в генерация альфы на рынке.

Отчет Джима Крамера

В то время как сегодня многие считают финансовые телешоу не более чем дрянным развлечением, нет никаких сомнений в том, что сами шоу по-прежнему имеют верных поклонников и являются одними из самых популярных программ на кабельном телевидении.

Давно прошли те дни, когда Mad Money и другие прямые трансляции CNBC могли собирать около полумиллиона зрителей за ночь, но данные ясно указывают на тот факт, что они по-прежнему остаются актуальной платформой, поддерживая аудиторию около 200 000 в течение года.

Mad Money Ratings (Cable Rankings)

Именно здесь мы видим истинную ценность возможности отслеживать и систематизировать такие данные. Ежемесячно на телешоу CNBC Finance в обоих направлениях публикуются сотни рекомендаций по акциям, и мы собираем их все в течение многих лет, чтобы иметь возможность протестировать данные и создать на их основе уникальные торговые стратегии.

Даже если рекомендации способны повлиять лишь на небольшой процент их зрителей в процессе выбора акций, они все равно окажут влияние на объемы торгов обсуждаемым базовым активом или, по крайней мере, дадут аналогичный результат косвенно через ожидание воздействия на базовый актив.

Рейтинги шоу CNBC (кабельные рейтинги)

В сегодняшней статье мы рассмотрим выступление ведущего, чье имя с годами стало почти синонимом финансового телевидения, не кто иной, как Джим Крамер. Его рекомендации по акциям помогали поколениям инвесторов ориентироваться, иногда к лучшему, в то время как его критики сразу же отмечали, что зачастую это было к худшему.

История медвежьего рынка S&P (Нью-Йорк Таймс)

Джим Крамер пережил семь последних медвежьих рынков и так или иначе активно участвовал в рынке в течение последних четырех медвежьих рынков, включая пузырь доткомов. , финансовый кризис, относительно короткий медвежий рынок пандемии и текущий продолжающийся медвежий рынок, что делает его опыт ценным. Итак, давайте посмотрим, как его выбор акций отразился на рыночном спаде, когда S&P 500 (SPY) потерял более 14% своей стоимости с начала года. До этого момента в этом году Джим Крамер дал в общей сложности 876 рекомендаций по покупке акций или иным образом выразил оптимистичные взгляды на отдельные акции. Вот наш список из десяти наиболее рекомендуемых акций Джима, измеряющих эффективность с первой до последней рекомендации купить:

Procter & Gamble (PG)
  • Рейтинг QuiverQuant: #1

  • Всего рекомендаций по покупке: 24

  • Первая рекомендация: 3 марта
  • 9005 1
    • Последняя рекомендация: 29 июля

    • Отслеживание эффективности: -10,03%

    Отчет QQ Procter & Gamble (Quiver Quantitative)

    Walt Disney (DIS)
    • QuiverQuant Ranking #2

    • Всего рекомендаций по покупке: 22

    • Первая рекомендация: 23 февраля
    • Последняя рекомендация: 21 июля

    • Производительность Трекер: -29,03%

    Отчет QQ Disney (количественный анализ )

    Qualcomm (QCOM)
    • QuiverQuant Ranking #3

    • Всего рекомендаций по покупке: 16

    • Первая рекомендация: 3 января
    • Последняя рекомендация: 11 мая

    • Отслеживание производительности: -28,95% 004

      Constellation Brands (STZ)
      • QuiverQuant Ranking #4

      • Всего рекомендаций по покупке: 15

      • Первая рекомендация: 5 мая
      • Последняя рекомендация: 11 июля

      • Отслеживание производительности: -4,27%

      Отчет Constellation Brands QQ (Quiver Quantitative)

      Morgan Stanley (MS)
      • QuiverQuant Ranking #5

      • Итого Купить Рекомендации: 14

      • Первая рекомендация: 24 февраля
      • Последняя рекомендация: 11 июля. 0039

        • QuiverQuant Ranking #6

        • Всего рекомендаций по покупке: 14

        • Первая рекомендация: 20 апреля
        • Последняя рекомендация : 25 июля

        • Отслеживание эффективности: -6,21%

        Отчет QQ Johnson & Johnson (Quiver Quantitative)

        Halliburton Company (HAL)
        • QuiverQuant Ranking #7

        • Всего рекомендаций по покупке: 14

        • Первая рекомендация: 8 марта
        • Последняя рекомендация: 26 июля

        • Отслеживание производительности: -22,76% Отчет Q (Quiver Quantitative)

          Meta Platforms (META)
          • QuiverQuant #8 в рейтинге

          • Всего рекомендаций по покупке: 14

          • Первая рекомендация: 4 апреля
          • Последняя рекомендация: 1 июня

          • Отслеживание производительности: -19,34%

          Отчет Meta QQ (Quiver Quantitative)

          Pioneer Natural Resources (PXD)
          • QuiverQuant Рейтинг #9

          • Всего рекомендаций по покупке: 11

          • Первый Рекомендация: 17 марта
          • Последняя рекомендация: 21 июля

          • Отслеживание эффективности: -9,25%

          Отчет QQ Pioneer Natural Resources (Q uiver Количественный)

          Marvell Technology (MRVL)
          • QuiverQuant Ranking #10

          • Всего рекомендаций по покупке: 11

          • Первая рекомендация: 28 февраля 9004 4
          • Последняя рекомендация: 7 июля

          • Отслеживание производительности : -31,80%

          Отчет Marvell QQ (Quiver Quantitative)

          Помимо нашего списка наиболее рекомендуемых акций, Джим Крамер, похоже, особенно любит: Danaher (DHR), Costco Wholesale (COST), Coterra Energy (CTRA), Humana (HUM), Alphabet (GOOG), Bausch Health Companies (BHC), Eli Lilly and Co. (LLY), Enterprise Products Partners (EPD), Airbnb (ABNB), Hertz Global Holdings (HTZ), Devon Energy (DVN), Rio Tinto (RIO), Microsoft (MSFT), SoFi Technologies (SOFI), Tellurian (TELL) и другие.

          С другой стороны, более заметные рекомендации «продать» или «урезать» от хоста Mad Money в этом году включают такие компании, как ZIM Integrated Shipping Services (ZIM), Chevron Corporation (CVX), Intel (INTC), Norwegian Cruise Line. Holdings (NCLH), Roblox Corp. (RBLX), Rivian Automotive (RIVN), Shopify (SHOP), Upstart Holdings (UPST), American Eagle Outfitters (AEO), Amazon (AMZN), Boeing (BA), Coinbase (COIN) , Mastercard (MA), NortonLifeLock (NLOK), Nvidia (NVDA), Rocket Companies (RKT), Roku (ROKU), AT&T (T) и другие.

          Наш подход и стратегии

          С идеей извлечь выгоду из собранных наборов данных, мы создали стратегию «обратного Крамера», которая использует данные довольно интересным и уникальным способом.

          Стратегия отслеживает показатели компаний, которые были наиболее рекомендованы популярным телеведущим в течение последнего месяца. Он делает это, открывая короткую позицию в самых рекомендуемых компаниях, хеджируя ее длинной позицией в рыночном индексе. В стратегии используется равновзвешенный портфель с реализованной системой еженедельной ребалансировки. Этот подход был успешным для получения среднегодового темпа роста в 26,18%, учитывая дату его начала 1 января 2021 года. Годовая доходность составляет 20,13% по сравнению с S&P500, который дал отрицательный результат 6,29.% возвращаться.

          Стратегия перевернутого Джима Крамера (Quiver Quantitative)

          Самая большая короткая позиция на данный момент в стратегии находится в Archer-Daniels-Midland Co. (ADM), взвешенная в настоящее время на уровне 9,23%. За этой позицией вскоре следуют акции Disney (DIS) на 9,05% и Johnson & Johnson (JNJ) на 9,01%. Если бы в начале 2021 года кто-то вложил 100 миллионов долларов в изменение рекомендаций по акциям известного телеведущего, то на сегодняшний день те же инвестиции составили бы 144 миллиона долларов, что немного ниже июньского максимума, когда портфель достиг своего пика. почти 152 миллиона долларов.

          Лучшие короткие позиции (Quiver Quantitative)

          Заключительные мысли и выводы

          Несмотря на то, что ведущий «Безумных денег» часто подвергается критике из-за своей слабой работы по выбору акций, он, без сомнения, является одним из самых влиятельных телеведущих в мире. История финансов ТВ. Тем не менее, остается фактом, что его способности выбирать акции, похоже, медленно ухудшались с годами. При индексации его акций, взвешенных по частоте его рекомендаций по покупке в эфире, кажется, что в среднем они дают далекие от отличных результаты с начала года, часто отставая от рынка. Это то, что привело нас на путь создания торговой стратегии, которая позволяет нам переворачивать сделки, рекомендованные легендой о Безумных деньгах.

          Есть ценность в измерении настроений харизматичных ведущих CNBC, таких как Джим Крамер, учитывая, что акции, которые чаще всего рекомендуются или осуждаются, обречены на дополнительный объем торгов благодаря влиянию таких ведущих, как Джим Крамер. , или просто иллюзией влияния, ожидаемого другими инвесторами. Из когда-то авторитетного и уважаемого менеджера хедж-фонда, который зарабатывает деньги на инвестициях, Крамер изменился и создал еще более успешный бизнес, предоставляя контент для инвесторов, не обязательно беспокоясь об успешности своих акций. В настоящее время, будучи больше озабоченным количеством выступлений и рейтингами, он, кажется, следует философии, согласно которой чем более возмутительным и спорным является выбор акций, тем лучше. Хотя диковинность и противоречивость могут привести к хорошим рейтингам, эта философия в сочетании с большим количеством рекомендаций, которые он должен дать, в конечном итоге привела к плохой отдаче.

          Как мы уже говорили ранее, только в этом квартале телеведущий дал около 900 рекомендаций по акциям. Тем не менее, это может быть использовано в интересах среднего инвестора, поскольку его неэффективность укрепляется до такой степени, что инвертирование его рекомендаций по акциям в действительности обеспечивает альфу на рынке, тем самым представляя интересную и, возможно, прибыльную инвестиционную стратегию.

          Перевернутая стратегия Джима Крамера: обновление за 3 квартал 2022 года

          Скотт Грайс

          В последние пару десятилетий наблюдается четкая тенденция инвестирования на фондовом рынке. Медленно отходит от профессионалов и с каждым днем ​​становится все более популярным. Крупные медийные конгломераты извлекли выгоду из этого процесса на раннем этапе, запустив программы, связанные с финансами и инвестициями, которые были преимущественно ориентированы как по стилю, так и по содержанию на среднего розничного инвестора. Одним из пионеров в этой области был CNBC, и одно имя стало почти синонимом финансового телевидения: Джим Крамер.

          Его инвестиционные советы помогали поколениям инвесторов направлять его, иногда к лучшему, в то время как его критики сразу же отмечали, что часто это было к худшему. Этот человек год за годом зарекомендовал себя как надежный первоклассный источник лучших инвестиционных советов, но только тогда, когда мы инвертируем его процесс принятия решений. Это представляет собой основу нашей торговой стратегии «Инверсия Джима Крамера», которая является торговым механизмом, который инвертирует большинство самых популярных рекомендаций Крамера по акциям и до сих пор доказал свою эффективность в создании альфы на рынке.

          Сегодняшняя статья представляет собой обновление, посвященное эффективности стратегии в третьем квартале года. С нашей оригинальной статьей об обратной стратегии Джима Крамера можно ознакомиться по этой ссылке.

          Обновление стратегии обратного Крамера

          С идеей извлечь выгоду из собранных наборов данных мы создали стратегию «обратного Крамера», которая использует данные довольно интересным и уникальным способом.

          Стратегия отслеживает эффективность компаний, которые были наиболее рекомендованы популярным телеведущим в течение последних 30 дней. Он делает это, открывая короткую позицию в самых рекомендуемых компаниях, хеджируя ее длинной позицией в рыночном индексе. В стратегии используется равновзвешенный портфель с реализованной системой еженедельной ребалансировки. Этот подход был успешным для получения среднегодового темпа роста в 17,16%, учитывая дату его начала 1 января 2021 года. Стратегия дала отрицательную доходность с начала года в размере 1,37%, а также годовую доходность в размере 18,66%. За тот же период индекс S&P500 (SPY) с начала года показал отрицательное значение 22,9.6% и отрицательный годовой доход в размере 18,24% за тот же период.

          Обратная стратегия Джима Крамера (Quiver Quantitative)

          Самая крупная короткая позиция на данный момент в стратегии — Macy’s (M) с текущим распределением 5,12%. Вскоре за компанией следуют акции General Electric (GE) на 5,09% и Devon Energy (DVN) на 5,07%. Если бы средний инвестор вложил 100 миллионов долларов, отменив самые высокие рекомендации Джима Крамера по акциям в начале 2021 года, те же инвестиции составили бы 132 миллиона долларов на сегодняшний день, что немного ниже июньского максимума, когда портфель достиг своего пика. почти $152 млн.

          Strategy Holdings (Quiver Quantitative)

          The Walt Disney Company (DIS)

          Основанная в Бербанке интегрированная развлекательная компания, которой скоро исполнится сто лет, большую часть года переживала тяжелые времена. Помимо и без того сложного с точки зрения макроэкономики года, весь сектор потокового вещания все еще страдает от распродажи, которая последовала после того, как Netflix (NFLX) опубликовал свои катастрофические результаты за первый квартал.

          Отчет проливает свет на несколько фундаментальных недостатков бизнес-модели потоковой передачи, которая в течение многих лет волновала многих инвесторов и аналитиков. Именно тогда Джим Крамер проявил значительный интерес к компании, с тех пор рекомендуя инвесторам покупать акции. Обоснование довольно простое: компания Микки Мауса по-прежнему, возможно, является самой разносторонней развлекательной компанией в отрасли после довольно спорного приобретения 20th Century Fox за 71 миллиард долларов в 2019 году.. У нее также есть солидный портфель интеллектуальной собственности, которому другие конкуренты могут только позавидовать, и унаследованный бизнес, который обеспечит стабильность денежных потоков даже в самые трудные времена. Disney торгуется по NTM EV/EBITDA на уровне 13,79x, NTM P/E на уровне 21,09x и NTM P/FCF на уровне 42,10x. Как указывалось ранее, помимо практически всего, что связано с потоковой передачей, DIS значительно отставал от рынка в этом году. Компания получила отрицательную доходность с начала года в размере 36,89% и отрицательную годовую доходность в размере 42,22%.

          Примерно в это же время в прошлом году акции Disney торговались по цене $171,18 за акцию. Авторы Seeking Alpha по-прежнему настроены оптимистично в отношении DIS, присвоив ей рейтинг «Покупать» со средним баллом 3,56/5,00. средний балл 4,44 / 5,00.Дисней остается в верхней части списка покупок акций Крамера, и в последний раз ведущий Mad Money рекомендовал его всего несколько недель назад на 29сентября. В настоящее время компания торгуется на уровне $100,76.

          Walt Disney Company и S&P500 Результаты YTD с начала года (Seeking Alpha)

          Macy’s Inc. (M)

          Macy’s — еще одна компания с чрезвычайно богатой историей, которая заняла первое место в списке пожеланий Крамера. Гигант розничной сети из Нью-Йорка был основан еще в 1830 году и вырос, чтобы стать одним из лидеров среди универмагов высокого класса. Компания владеет и управляет знаковыми брендами, в том числе Macy’s, Bloomingdale’s и Bluemercury. Блокировки и ограничения в начале 2020 года сильно ударили по ее бизнесу, вызвав проблемы с ликвидностью и даже заставив компанию зафиксировать убыток в размере 0,63 миллиарда долларов за первый квартал года. С тех пор Macy’s успешно восстановилась, но теперь ее бизнес-модели, ориентированной на розничную торговлю, угрожают новые препятствия. Ухудшающаяся макроэкономическая среда может привести к сокращению спроса, поскольку домохозяйства ограничивают расходы по своему усмотрению.

          Тем не менее, Крамер по-прежнему оптимистичен в отношении бизнеса и даже принимал генерального директора Macy’s Джеффа Дженнетта в прошлую среду на своем шоу. Учитывая сложную ситуацию, в которой оказалась компания, неудивительно, что она показала себя хуже на рынке, получив отрицательную доходность с начала года в размере 31,63% и отрицательную годовую доходность в размере 29,41%. Macy’s торгуется по относительно привлекательной оценке с NTM EV/EBITDA на уровне 4,74x и NTM P/E на уровне 5,21x. Он также продается по невероятной цене 5,88x NTM P/FCF, так что ценностное предложение в этом случае остается ясным.

          Seeking Alpha Авторы довольно позитивно оценивают компанию, присваивая ей рейтинг «Покупать» со средним баллом 4.00/5.00. Тем не менее, аналитики Уолл-Стрит не убеждены, присвоив ему рейтинг «Держать» со средним баллом 3,33/5,00. В настоящее время компания продается по цене $19,51.

          Macy’s vs S&P500: доходность с начала года (Seeking Alpha)

          Amazon.com, Inc. (AMZN)

          Детище Джеффа Безоса в последнее время стало одной из наиболее рекомендуемых акций Джима и поднялось почти на вершину наших активов по обратной стратегии . Amazon, как и многие высокопоставленные технологические акции, после получения необычайной прибыли для инвесторов за последние пару лет, в последнее время значительно отставала от рынка и показала отрицательную доходность с начала года на 32,47% и отрицательную доходность на 33,17%. -год возвращения. Трудно представить, что только в ноябре прошлого года компания торговалась по цене $183,83. Крупный откат привел, пожалуй, к лучшей оценке, которую AMZN могла предложить за последние годы, поскольку в настоящее время она продается по NTM EV/EBITDA на уровне 16,15x, NTM P/E на уровне 74,08x и NTM P/FCF на уровне 43,18x.

          Seeking Alpha Авторы по-прежнему оптимистично оценивают перспективы AMZN даже после падения со средним рейтингом 3,50/5,00. С другой стороны, аналитики Уолл-Стрит по-прежнему убеждены и присвоили акциям рейтинг «Активная покупка» со средним баллом 4,62/5,00. Акции Amazon в настоящее время торгуются по $118,11.

          Amazon против SYP500 YTD Результаты (Seeking Alpha)

          Morgan Stanley (MS)

          Morgan Stanley — единственный крупный банк США, который попал в список покупок Крамера и стал одной из его наиболее рекомендуемых покупок. Недавно MS разочаровала инвесторов, пропустив в своем последнем отчете о прибылях и убытках за третий квартал как верхние, так и нижние оценки. Акции банка упали почти на 5%, поскольку рынок изо всех сил пытался переварить неприятный отчет. Выручка составила $12,9.9 миллиардов, меньше оценки в 13,3 миллиарда долларов, в то время как прибыль не достигла 0,02 доллара на акцию. Крамер подтвердил свое оптимистичное мнение после отчета, заявив, что «инвесторы должны быть терпеливы с Morgan Stanley». Банк торговался в основном в соответствии с рынком, получив отрицательную доходность с начала года в размере 22,51%, а также отрицательную годовую доходность в размере 23,70%. В настоящее время MS продает по P/BV на акцию 1,46x, предлагая дивидендную доходность в размере 3,90%.

          Банк пользуется большим уважением как у авторов Seeking Alpha, так и у аналитиков Уолл-Стрит. Авторы Seeking Alpha долгое время оптимистично оценивали компанию, присваивая ей рейтинг «Покупать» со средним баллом 3,83/5,00. Аналитики с Уолл-стрит были настроены еще более оптимистично и присвоили MS рекомендацию «Решительно покупать» со средним баллом 4,10/5,00. Банк в настоящее время торгуется на уровне около $ 78,76 за акцию.

          Результаты Morgan Stanley против S&P500 с начала года (Seeking Alpha)

          General Electric Company (GE)

          В ноябре прошлого года General Electric изложила хитрый план разделения на три публично торгуемые компании, которые будут отдельно сосредоточены на энергетике, здравоохранении и авиацию, чтобы максимизировать акционерную стоимость. Первым важным шагом в этом плане является выделение своего бизнеса в области здравоохранения в новую публичную компанию в начале 2023 года. GE последует этому, объединив свои энергетические подразделения в единую компанию, которая должна быть выделена в начале 2024 года9.0004

          После двух дочерних предприятий оставшаяся компания будет сосредоточена исключительно на авиации, что очень нравится Джиму Крамеру, сохраняющему оптимистичный взгляд на GE. Конгломерат с более чем столетней историей лишь немного отстал от рынка в этом году, получив отрицательную доходность с начала года в размере 26,71%, сопровождаемую отрицательной доходностью за год в размере 32,66%. GE продает акции по цене, которая может считаться справедливой в соответствии с общей рыночной оценкой. В настоящее время они торгуются по NTM EV/EBITDA на уровне 10,64x, NTM P/E на уровне 20,71x и NTM P/FCF на уровне 14,87x.

          Как авторы Seeking Alpha, так и аналитики Уолл-стрит по-прежнему оптимистично оценивают перспективы старого гиганта, присваивая GE рейтинг «покупать», при этом первый дает средний балл 3,60/5,00, в то время как последний пришел с большим более восторженный 4. 22/5.00. Акции General Electric в настоящее время торгуются по $70,53.

          Результаты General Electric против S&P500 с начала года (в поисках альфа-версии)

          Последние квартальные данные

          Мы собрали, обобщили и проанализировали более 3637 рекомендаций по подбору акций в прямом эфире. Только в этом квартале он добавил к этому счету еще 440 рекомендаций. В 225 случаях Джим Крамер рекомендовал купить акции или иным образом выражал бычьи взгляды на отдельные акции в течение третьего квартала этого года. Обратите внимание, что в обсуждаемых здесь компаниях будут некоторые различия, поскольку этот список охватывает весь третий квартал, а наша стратегия отслеживает его рекомендации за последние 30 дней.

          Выбор акций Джима Крамера (Quiver Quantitative)

          Вот наш список наиболее рекомендуемых акций Джима:

          Jonhson & Jonhson (JNJ)

          • Рейтинг QuiverQuant Buy: #1
          • Всего рекомендаций по покупке : 8
          • Первый Рекомендация: 13 июля
          • Последняя рекомендация: 29 августа

          Jonhson and Jonhson QQ Report (Quiver Quantitative)

          Tellurian Inc (TELL)

          • QuiverQuant Buy Ranking #2
          • Всего рекомендаций по покупке: 6
          • Первая рекомендация: 19 июля
          • Последняя рекомендация: 19 сентября F)

            • QuiverQuant Buy Рейтинг #3
            • Всего рекомендаций по покупке: 6
            • Первая рекомендация: 5 июля
            • Последняя рекомендация: 12 сентября

            Отчет QQ Ford Motor Company (Quiver Quantitative)

            The Walt Disney Company

            • QuiverQuant Buy Ranking #4
            • Всего рекомендаций по покупке: 6
            • Первая рекомендация: 5 июля
            • Последняя рекомендация: 29 сентября

            Отчет QQ компании Walt Disney (количественный анализ)

            Starbucks Corporation (SBUX)

            • QuiverQuant Buy Ranking #5
            • Всего рекомендаций по покупке: 5
            • Первая рекомендация: 8 августа
            • Последняя рекомендация: 6 сентября. , Devon Energy Corporation (DVN), The TJX Companies (TJX), Morgan Stanley (MS), Constellation Brands (STZ), SoFi Technologies (SOFI), Procter & Gamble Company (PG), MP Materials (MP), Honeywell International ( HON), Halliburton Company (HAL), Alphabet Inc. (GOOGL), Cisco Systems, Inc. (CSCO), Salesforce, Inc. (CRM), Wells Fargo & Company (WFC), Twitter, Inc. (TWTR), Tesla , Inc. (TSLA), Seagen Inc. (SGEN), Qualcomm Incorporated (QCOM), Realty Income Corporation (O), Cloudflare, Inc. (NET), Microsoft Corporation (MSFT) и другие.

              Джим Крамер также дал в общей сложности 185 рекомендаций по продаже акций в прямом эфире или иным образом выразил свое негативное отношение к отдельным акциям в течение третьего квартала этого года.

              Вот наш список самых рекомендуемых продаж Джима:

              Humana Inc (HUM)
              • QuiverQuant Sell Ranking #1
              • Всего рекомендаций по продажам: 3
              • Первая рекомендация: 6 июля
              • 90 041 Последняя рекомендация: 14 числа Сентябрь

              Humana Inc QQ Report (Quiver Quantitative)

              AT&T Inc (T)
              • QuiverQuant Sell Ranking #2
              • Всего рекомендаций по продажам: 3
              • Первая рекомендация: 14 июля
              • Последняя Рекомендация: 30 августа

              AT&T QQ Report (Quiver Quantitative)

              The Boeing Company (BA)
              • QuiverQuant Sell Ranking #3
              • Всего рекомендаций по продажам: 3
              • Первая рекомендация: 6 июля
              • Последняя рекомендация: 29 сентября. 2
              • Первая рекомендация: 29 числа Август
              • Последняя рекомендация: 31 августа.0044
              • Всего рекомендации по продаже: 2
              • Первая рекомендация: 15 июля
              • Последняя рекомендация: 29 июля

              Эли Лилли и отчет Компании QQ (Quiver Quantitative)

              С другой рекомендации ведущего Mad Money в этом году включают такие компании, как AT&T Inc. (T), 3M Company (MMM), Upstart Holdings, Inc. (UPST), ChargePoint Holdings, Inc. (CHPT), Alibaba Group Holding Limited ( BABA), Uber Technologies, Inc. (UBER), Marathon Digital Holdings, Inc. (MARA), Kohl’s Corporation (KSS), Verizon Communications Inc. (VZ), Snap Inc. (SNAP), Carnival Corporation & plc (CCL) , Warner Bros. Discovery, Inc. (WBD), Block, Inc. (SQ), Vale S.A. (VALE), Zoom Video Communications (ZM), Sunnova Energy (NOVA), Nucor Corporation (NUE), Occidental Petroleum (OXY) , и другие.

              Заключительные мысли и выводы

              Несмотря на то, что он стал мишенью критики из-за слабого выбора акций, ведущий Mad Money остается одним из самых влиятельных и популярных деятелей финансового телевидения в истории. Он ветеран инвестирования, пока на улицах льется кровь, учитывая, что он пережил семь последних медвежьих рынков и так или иначе профессионально занимался рынком в течение последних четырех. Как мы упоминали ранее, телеведущий дал около 440 рекомендаций по выбору акций только в этом квартале, добавив к нашему счету более 3600 вариантов, которые мы собрали и обобщили на протяжении многих лет. Тем не менее, есть одна сильная основная тенденция, которая может быть зафиксирована при анализе его выбора акций: подавляющее большинство этих советов по акциям, как правило, дают низкие результаты, как это снова было продемонстрировано в этом квартале.

              В то время как его диковинные и спорные выборы могут привести к хорошим рейтингам, эта философия в сочетании с большим количеством рекомендаций, которые он должен дать, в конечном итоге привела к плохой прибыли. Только в третьем квартале обратная стратегия Крамера принесла около 2,2% прибыли, в то время как общий рынок за тот же период снизился на 2,37%.

Doc в rtf формат: Онлайн-конвертер DOC в RTF | Бесплатные приложения GroupDocs

Онлайн-конвертер DOC в RTF | Бесплатные приложения GroupDocs

Вы также можете конвертировать DOC во многие другие форматы файлов. Пожалуйста, смотрите полный список ниже.

DOC TO PDF Конвертер (Портативный документ)

DOC TO HTM Конвертер (Файл языка гипертекстовой разметки)

DOC TO HTML Конвертер (Язык гипертекстовой разметки)

DOC TO MHTML Конвертер (MIME-инкапсуляция совокупного HTML)

DOC TO MHT Конвертер (MIME-инкапсуляция совокупного HTML)

DOC TO XPS Конвертер (Спецификация документа Open XML)

DOC TO TEX Конвертер (Исходный документ LaTeX)

DOC TO PPT Конвертер (Презентация PowerPoint)

DOC TO PPS Конвертер (Слайд-шоу Microsoft PowerPoint)

DOC TO PPTX Конвертер (Презентация PowerPoint Open XML)

DOC TO PPSX Конвертер (Слайд-шоу PowerPoint Open XML)

DOC TO ODP Конвертер (Формат файла презентации OpenDocument)

DOC TO OTP Конвертер (Шаблон графика происхождения)

DOC TO POTX Конвертер (Открытый XML-шаблон Microsoft PowerPoint)

DOC TO POT Конвертер (Шаблон PowerPoint)

DOC TO POTM Конвертер (Шаблон Microsoft PowerPoint)

DOC TO PPTM Конвертер (Презентация Microsoft PowerPoint)

DOC TO PPSM Конвертер (Слайд-шоу Microsoft PowerPoint)

DOC TO FODP Конвертер (Плоская XML-презентация OpenDocument)

DOC TO EPUB Конвертер (Формат файла цифровой электронной книги)

DOC TO MOBI Конвертер (Электронная книга Mobipocket)

DOC TO AZW3 Конвертер (Kindle eBook format)

Преобразовать DOC TO TIFF (Формат файла изображения с тегами)

Преобразовать DOC TO TIF (Формат файла изображения с тегами)

Преобразовать DOC TO JPG (Файл изображения Объединенной группы экспертов по фотографии)

Преобразовать DOC TO JPEG (Изображение в формате JPEG)

Преобразовать DOC TO PNG (Портативная сетевая графика)

Преобразовать DOC TO GIF (Графический файл формата обмена)

Преобразовать DOC TO BMP (Формат растрового файла)

Преобразовать DOC TO ICO (Файл значка Майкрософт)

Преобразовать DOC TO PSD (Документ Adobe Photoshop)

Преобразовать DOC TO WMF (Метафайл Windows)

Преобразовать DOC TO EMF (Расширенный формат метафайла)

Преобразовать DOC TO DCM (DICOM-изображение)

Преобразовать DOC TO DICOM (Цифровая визуализация и коммуникации в медицине)

Преобразовать DOC TO WEBP (Формат файла растрового веб-изображения)

Преобразовать DOC TO JP2 (Основной файл изображения JPEG 2000)

Преобразовать DOC TO EMZ (Расширенный сжатый метафайл Windows)

Преобразовать DOC TO WMZ (Метафайл Windows сжат)

Преобразовать DOC TO SVGZ (Сжатый файл масштабируемой векторной графики)

Преобразовать DOC TO TGA (Тарга Графика)

Преобразовать DOC TO PSB (Файл изображения Adobe Photoshop)

Преобразовать DOC TO SVG (Файл масштабируемой векторной графики)

Преобразовать DOC TO DOC (Документ Microsoft Word)

DOC TO DOCM Преобразование (Документ Microsoft Word с поддержкой макросов)

DOC TO DOCX Преобразование (Документ Microsoft Word с открытым XML)

DOC TO DOT Преобразование (Шаблон документа Microsoft Word)

DOC TO DOTM Преобразование (Шаблон Microsoft Word с поддержкой макросов)

DOC TO DOTX Преобразование (Шаблон документа Word Open XML)

DOC TO ODT Преобразование (Открыть текст документа)

DOC TO OTT Преобразование (Открыть шаблон документа)

DOC TO TXT Преобразование (Формат обычного текстового файла)

DOC TO MD Преобразование (Уценка)

DOC TO XLS Преобразование (Формат двоичного файла Microsoft Excel)

DOC TO XLSX Преобразование (Электронная таблица Microsoft Excel Open XML)

DOC TO XLSM Преобразование (Электронная таблица Microsoft Excel с поддержкой макросов)

DOC TO XLSB Преобразование (Двоичный файл электронной таблицы Microsoft Excel)

DOC TO ODS Преобразование (Открыть электронную таблицу документов)

DOC TO XLTX Преобразование (Открытый XML-шаблон Microsoft Excel)

DOC TO XLT Преобразование (Шаблон Microsoft Excel)

DOC TO XLTM Преобразование (Шаблон Microsoft Excel с поддержкой макросов)

DOC TO TSV Преобразование (Файл значений, разделенных табуляцией)

DOC TO XLAM Преобразование (Надстройка Microsoft Excel с поддержкой макросов)

DOC TO CSV Преобразование (Файл значений, разделенных запятыми)

DOC TO FODS Преобразование (Плоская XML-таблица OpenDocument)

DOC TO SXC Преобразование (Электронная таблица StarOffice Calc)

Конвертировать DOC В RTF Бесплатно

DOC в RTF

Разработано на базе программных решений от aspose. com а также aspose.cloud

Выберите DOC файлы или перетащите DOC файлы мышью

Google Drive Dropbox

Использовать пароль

Этот пароль будет применяться ко всем документам

Использовать распознавание текста Использовать распознавание текста

АнглийскийАрабскийИспанскийИтальянскийКитайский упрощенныйНемецкийПерсидскийПольскийПортугальскийРусскийФранцузский

Загружая свои файлы или используя наш сервис, вы соглашаетесь с нашими Условиями обслуживания и Политикой конфиденциальности.

Сохранить как

RTFDOCXTXTDOCDOTDOCMDOTXDOTMODTOTT

КОНВЕРТИРОВАТЬ

Ваши файлы были успешно сконвертированы

СКАЧАТЬ

Загрузить в Google Загрузить в Dropbox

Конвертация других документов Отправить на электронную почту
Пройдите наш опрос

Хотите сообщить об этой ошибке на форуме Aspose, чтобы мы могли изучить и решить проблему? Когда ошибка будет исправлена, вы получите уведомление на email. Форма отчета

Google Sheets
Mail Merge Облачный API

Конвертировать DOC в RTF онлайн

Используйте конвертер DOC в RTF для экспорта файлов DOC в RTF формат онлайн. Наш конвертер файлов проанализирует содержимое исходного DOC файла до мельчайших деталей и воссоздаст содержимое в целевом RTF формате.

Вы можете использовать конвертер из DOC в RTF совершенно бесплатно, в любое время и с любого устройства.

Онлайн Конвертер DOC в RTF

Конвертация DOC файлов в RTF формат — одна из самых распространенных операций. Нам часто нужны обе функции, предоставляемые форматами DOC и RTF. DOC и RTF в определённых случаях дополняют друг друга.

Конвертировать файл DOC в RTF онлайн

Чтобы конвертировать DOC в RTF формат, просто перетащите DOC файл в область загрузки данных, укажите параметры преобразования, нажмите кнопку ‘Конвертировать’ и получите выходной RTF файл за считанные секунды.

Бесплатный онлайн конвертер DOC в RTF основан на продуктах компании Aspose, которые широко используются во всем мире для программной обработки DOC и RTF с высокой скоростью и профессиональным качеством результата.

Как преобразовать DOC в RTF

  1. Загрузите DOC файлы, чтобы преобразовать их в RTF формат онлайн.
  2. Укажите параметры преобразования DOC в RTF.
  3. Нажмите кнопку, чтобы конвертировать DOC в RTF онлайн.
  4. Загрузите результат в RTF формате для просмотра.
  5. Вы можете отправить ссылку для скачивания по электронной почте, если хотите получить результаты позже.

Вопросы-Ответы

Как конвертировать DOC в RTF бесплатно?


Просто используйте наш DOC в RTF Converter. Вы получите выходные файлы RTF одним кликом мыши.

Сколько DOC файлов я могу конвертировать в RTF формат за раз?


Вы можете конвертировать до 10 DOC файлов за раз.

Каков максимально допустимый размер DOC файла?


Размер каждого DOC файла не должен превышать 10 МБ.

Какие есть способы получить результат в RTF формате?


После завершения преобразования DOC в RTF вы получите ссылку для скачивания. Вы можете скачать результат сразу или отправить ссылку на скачивание RTF на свой e-mail позже.

Как долго мои файлы будут храниться на ваших серверах?


Все пользовательские файлы хранятся на серверах Aspose в течение 24 часов. По истечении этого времени они автоматически удаляются.

Можете ли вы гарантировать сохранность моих файлов? Все безопасно?


Aspose уделяет первостепенное внимание безопасности и защите пользовательских данных. Будьте уверены, что ваши файлы хранятся на надежных серверах и защищены от любого несанкционированного доступа.

Почему конвертация DOC в RTF занимает немного больше времени, чем я ожидал?


Конвертация больших DOC файлов в RTF формат может занять некоторое время, поскольку эта операция включает перекодирование и повторное сжатие данных.

Online DOC to RTF Converter

Вы также можете конвертировать DOC во многие другие форматы файлов. Пожалуйста, ознакомьтесь с полным списком ниже.

Преобразователь DOC в PDF (переносимый документ)

Преобразователь DOC в HTM (файл языка гипертекстовой разметки)

Конвертер DOC TO HTML (язык гипертекстовой разметки)

Преобразователь DOC TO MHTML (инкапсуляция MIME совокупного HTML)

Конвертер DOC TO MHT (инкапсуляция MIME совокупного HTML)

Преобразователь DOC в XPS (спецификация Open XML Paper)

Конвертер DOC TO TEX (исходный документ LaTeX)

Преобразователь DOC в PPT (презентация PowerPoint)

Преобразователь DOC в PPS (слайд-шоу Microsoft PowerPoint)

Конвертер DOC TO PPTX (презентация PowerPoint Open XML)

Преобразователь DOC в PPSX (слайд-шоу PowerPoint Open XML)

Преобразователь DOC в ODP (формат файла презентации OpenDocument)

Конвертер DOC TO OTP (шаблон исходного графика)

Конвертер DOC TO POTX (шаблон Microsoft PowerPoint Open XML)

Конвертер DOC TO POT (шаблон PowerPoint)

Конвертер DOC TO POTM (шаблон Microsoft PowerPoint)

Преобразователь DOC в PPTM (презентация Microsoft PowerPoint)

Конвертер DOC TO PPSM (слайд-шоу Microsoft PowerPoint)

Конвертер DOC TO FODP (представление OpenDocument Flat XML)

Конвертер DOC TO EPUB (формат файлов цифровых электронных книг)

Конвертер DOC TO MOBI (электронная книга Mobipocket)

Конвертер DOC TO AZW3 (формат электронной книги Kindle)

Преобразование DOC в TIFF (формат файла изображения с тегами)

Преобразование DOC в TIF (формат файла изображения с тегами)

Конвертировать DOC в JPG (файл изображения Объединенной группы экспертов по фотографии)

Преобразовать DOC в JPEG (изображение JPEG)

Конвертировать DOC в PNG (переносимая сетевая графика)

Преобразование DOC в GIF (файл формата графического обмена)

Преобразование DOC в BMP (формат растрового файла)

Преобразовать DOC в ICO (файл значка Microsoft)

Преобразовать DOC в PSD (документ Adobe Photoshop)

Преобразовать DOC в WMF (метафайл Windows)

Преобразовать DOC в EMF (расширенный формат метафайла)

Преобразовать DOC в DCM (изображение DICOM)

Преобразовать DOC в DICOM (цифровые изображения и коммуникации в медицине)

Конвертировать DOC в WEBP (формат файла растрового веб-изображения)

Конвертировать DOC в JP2 (основной файл изображения JPEG 2000)

Преобразование DOC в EMZ (расширенный сжатый метафайл Windows)

Конвертировать DOC в WMZ (сжатый метафайл Windows)

Конвертировать DOC в SVGZ (файл сжатой масштабируемой векторной графики)

Конвертировать DOC В TGA (Targa Graphic)

Конвертировать DOC в PSB (файл изображения Adobe Photoshop)

Конвертировать DOC в SVG (файл масштабируемой векторной графики)

Преобразовать DOC в DOC (документ Microsoft Word)

Преобразование DOC в DOCM (документ Microsoft Word с поддержкой макросов)

Преобразование DOC в DOCX (документ Microsoft Word Open XML)

Преобразование DOC в DOT (шаблон документа Microsoft Word)

Преобразование DOC в DOTM (шаблон Microsoft Word с поддержкой макросов)

Преобразование DOC в DOTX (шаблон документа Word Open XML)

Преобразование DOC в ODT (текст открытого документа)

Преобразование DOC в OTT (открытый шаблон документа)

Преобразование DOC в TXT (формат обычного текстового файла)

Преобразование DOC в MD (уценка)

Преобразование DOC в XLS (формат двоичных файлов Microsoft Excel)

Преобразование DOC в XLSX (электронная таблица Microsoft Excel Open XML)

Преобразование DOC в XLSM (электронная таблица Microsoft Excel с поддержкой макросов)

Преобразование DOC в XLSB (двоичный файл электронной таблицы Microsoft Excel)

Преобразование DOC в ODS (открытая таблица документов)

Преобразование DOC в XLTX (шаблон Microsoft Excel Open XML)

Преобразование DOC в XLT (шаблон Microsoft Excel)

Преобразование DOC в XLTM (шаблон Microsoft Excel с поддержкой макросов)

Преобразование DOC в TSV (файл значений, разделенных табуляцией)

Преобразование DOC в XLAM (надстройка Microsoft Excel с поддержкой макросов)

Преобразование DOC в CSV (файл значений, разделенных запятыми)

Преобразование DOC TO FODS (таблица OpenDocument Flat XML)

Преобразование DOC в SXC (электронная таблица StarOffice Calc)

Преобразование DOC в RTF онлайн

Пакетное преобразование файлов doc в формат rtf онлайн бесплатно

Выберите файлы или перетащите их сюда.
Доступ к вашим файлам есть только у вас.
Все файлы будут удалены через час.

Загрузить файл DOC

Перетащите файл DOC в область загрузки. Максимальный размер файла – 100 МБ.

DOC в RTF

Нажмите «Преобразовать», чтобы преобразовать документ в rtf. Преобразование обычно занимает несколько секунд.

Загрузите файл RTF

Теперь вы можете загрузить файл RTF. Ссылка для скачивания работает только на вашем устройстве.

Конвертер DOC

DOC в 7Z

DOC в CSV

DOC в DOCX

DOC в EPUB

DOC в HTML

DOC в JPEG 90 003

DOC в JPG

DOC в MOBI

DOC в MP3

DOC в ODT

DOC в PAGES

DOC в PDF

DOC в PNG

DOC в PPT

DOC в RAR

9000 2 DOC в RTF

DOC в TIFF

DOC в TXT

DOC в XLS

DOC в XLSX

DOC в XML

DOC в ZIP

Преобразовать в RTF

DOC в RTF

DOCX в RTF

90 002 HTML в RTF

PDF в RTF

XML в RTF

Часто задаваемые вопросы

  • ❓ Как конвертировать файлы из DOC в RTF?

    Сначала вам нужно добавить файл для конвертации: перетащите файл DOC или нажмите кнопку «Выбрать файл». Затем нажмите кнопку «Конвертировать». Когда преобразование DOC в RTF завершено, вы можете загрузить файл RTF.

  • ⏱️ Сколько времени нужно, чтобы конвертировать DOC в RTF?

    Преобразование документов происходит довольно быстро. Вы можете изменить DOC на RTF за несколько секунд.

  • 🛡️ Безопасно ли конвертировать DOC в RTF на AnyConv?

    Конечно! Мы удаляем загруженные файлы сразу, а сконвертированные через 1 час. Никто не имеет доступа к вашим файлам. Преобразование файлов (в том числе DOC в RTF) абсолютно безопасно.

  • 💻 Могу ли я конвертировать DOC в RTF на Mac OS или Linux?

    Да, AnyConv можно использовать в любой операционной системе с веб-браузером. Наш конвертер DOC в RTF работает онлайн и не требует установки программного обеспечения.

9 0252 формат файла документа
🔸 Формат файла DOC RTF 902 55
🔸 Полное имя DOC — формат двоичного файла Microsoft Word RTF — формат расширенного текста
🔸 Расширение файла . doc .rtf
🔸 Тип MIME application/ms слово текст/rtf, приложение/rtf
🔸 Разработано Microsoft Microsoft
🔸 Тип формата формат файла документа
🔸 Описание DOC — это расширение имени файла для текстовых документов, чаще всего в проприетарном формате двоичных файлов Microsoft Word. В Microsoft Word 2007 и более поздних версиях двоичный формат файла был заменен в качестве формата по умолчанию форматом Office Open XML, хотя Microsoft Word по-прежнему может создавать файлы DOC. Формат Rich Text (часто сокращенно RTF) — это собственный формат файлов документов с опубликованными спецификациями, разработанный корпорацией Microsoft для межплатформенного обмена документами с продуктами Microsoft. Большинство текстовых процессоров могут читать и записывать некоторые версии RTF.
🔸 Технические детали Двоичные файлы DOC часто содержат больше информации о форматировании текста (а также сценарии и информацию об отмене), чем некоторые другие форматы файлов документов, такие как Rich Text Format и язык гипертекстовой разметки, но обычно менее широко совместимы. В отличие от многих форматов обработки текста, код RTF может быть удобочитаемым: когда файл RTF просматривается как обычный текстовый файл, содержащийся в нем текст ASCII становится разборчивым. Кроме того, код форматирования не слишком отвлекает и не противоречит интуиции, при условии, что создатель документа придерживался лаконичного форматирования.
🔸 Преобразование файлов Преобразование DOC Преобразование RTF
🔸 Связанные программы 9025 5 Microsoft Word, OpenOffice.org Writer, IBM Lotus Symphony, Apple Pages, AbiWord. WordPad, LibreOffice, Microsoft Word.

Таблица определения реактивной мощности конденсаторной установки

Сертификаты

   

 

Новости

27.01.23

В 2023 году производственная компания «СлавЭнерго» продолжает наращивать объемы поставок силовых трансформаторов и конденсаторных установок на крупнейшие предприятия и важные объекты инфраструктуры страны. Цеха и офис нашего предприятия работают в обычном режиме.

подробнее…

01.01.23

Уважаемые партнеры!

 

Поздравляем вас с Новым Годом и Рождеством 2023! 

Успехов и процветания!

С уважением,  ПК «СлавЭнерго»

подробнее…

Главная » Комплектные конденсаторные установки от 0,4 до 10 кВ » Конденсаторные установки 6,3 — 10,5 кВ » Таблица определения реактивной мощности конденсаторной установки — КРМ (кВАр) необходимой для достижения заданного cos(φ)

Текущий (действующий)Требуемый (достижимый) cos (φ)
tan (φ)cos (φ)0. 800.820.850.880.900.920.940.960.981.00
Коэффициент K
3.180.302.432.482.562.642.702.752.822.892.983.18
2.960.322.212.262.342.422.482.532.602.672.762.96
2.770.342.022.072.152.232.282.342.412.482.562.77
2.590.361.841.891.972.052.102.172.232.302.392.59
2.430.381.681.731.811.891.952.012. 072.142.232.43
2.290.401.541.591.671.751.811.871.932.002.092.29
2.160.421.411.461.541.621.681.731.801.871.962.16
2.040.441.291.341.421.501.561.611.681.751.842.04
1.930.461.181.231.311.391.451.501.571.641.731.93
1.830.481.081.131.211.291.341.401.471.541.621.83
1.730.500.981.031. 111.191.251.311.371.451.631.73
1.640.520.890.941.021.101.161.221.281.351.441.64
1.560.540.810.860.941.021.071.131.201.271.361.56
1.480.560.730.780.860.941.001.051.121.191.281.48
1.400.580.650.700.780.860.920.981.041.111.201.40
1.330.600.580.630.710.790.850.910.971.041.131.33
1. 300.610.550.600.680.760.810.870.941.011.101.30
1.270.620.520.570.650.730.780.840.910.991.061.27
1.230.630.480.530.610.690.750.810.870.941.031.23
1.200.640.450.500.580.660.720.770.840.911.001.20
1.170.650.420.470.550.630.680.740.810.880.971.17
1.140.660.390.440.520.600.650.710.780. 850.941.14
1.110.670.360.410.490.570.630.680.750.820.901.11
1.080.680.330.380.460.540.590.650.720.790.881.08
1.050.690.300.350.430.510.560.620.690.760.851.05
1.020.700.270.320.400.480.540.590.660.730.821.02
0.990.710.240.290.370.450.510.570.630.700.790.99
0.960.720.210.260.340. 420.480.540.600.670.760.96
0.940.730.190.240.320.400.450.510.580.650.730.94
0.910.740.160.210.290.370.420.480.550.620.710.91
0.880.750.130.180.260.340.400.460.520.590.680.88
0.860.760.110.160.240.320.370.430.500.570.650.86
0.830.770.080.130.210.290.340.400.470.540.630.83
0.800. 780.050.100.180.260.320.380.440.510.600.80
0.780.790.030.080.160.240.290.350.420.490.570.78
0.750.800.050.130.210.270.320.390.460.550.75
0.720.810.100.180.240.300.360.430.520.72
0.700.820.080.160.210.270.340.410.490.70
0.670.830.050.130.190.250.310.380.470.67
0.650. 840.030.110.160.220.290.360.440.65
0.620.850.080.140.190.260.330.420.62
0.590.860.050.110.170.230.300.390.59
0.570.870.080.140.210.280.360.57
0.540.880.060.110.180.250.340.54
0.510.890.030.090.150.220.310.51
0.480.900.060.120.190.280. 48
0.460.910.030.100.170.250.46
0.430.920.070.140.220.43
0.400.930.040.110.190.40
0.360.940.070.160.36
0.330.950.130.33

КРМ (кВАр) = Pa х (tg(φ1)-tg(φ2))
КРМ (кВАр) = Pa х K =Активная мощность [кВт] х коэффициент K
Pa = S х cos(φ) = Полная мощность х cos (φ)
tg(φ1+φ2) согласуются со значениями cos (φ) в таблице.

ПРИМЕР:
Активная мощность двигателя: P=100 кВт
Действующий cos (φ) 0. 60
Требуемый cos (φ) 0.90
Коэффициент K из таблицы 0.85
Необходимая реактивная мощность КРМ (кВАр) = 100 х 0.85=85 кВАр

0-0 | брейд вики | Fandom

Дом Тима

Дом Тима — это место, где расположены миры игры. Хотя в игре нет явного указания на то, что дом принадлежит Тиму, фанаты игры очень предполагают это, и поэтому это известно. Однако Дэвид Хеллман также называет его Домом Тима в своем блоге. Сразу за входом (с левой стороны дома) в Дом Тима открывается вид на небо. Дальше слева есть мост, на котором Тим, по-видимому, начинает свое путешествие. На заднем плане моста охваченный пламенем город.

Содержание

  • 1 Макет
    • 1.1 Комнаты
    • 1.2 Картины
    • 1,3 звезды
  • 2 Общая информация

Layout[]

Дом Тима разбит на комнаты. Комнаты загораются, когда миры в этих комнатах разблокированы, что делается путем прохождения предыдущих миров.

Комнаты[]

Первая комната находится на первом этаже и содержит двери в Мир 2 и 3. Первая секция, в которой находится Мир 2, загорается при первом входе Тима в дом. Секция, в которой находится Мир 3, загорается после завершения Мира 2.

Вторая комната находится в дальнем правом углу дома и содержит дверь в Мир 4. Миры 4 и 5 становятся доступными после завершения Мира 3. Завершение Мира 4 открывает Миры 5 и 6.

Третья комната находится на нижнем этаже дома и содержит двери в Миры 5 и 6. Ванная также находится на этом этаже, что намекает на альтернативную схему управления в игре.

В последнюю комнату или на чердак можно попасть по лестнице. Эта комната содержит вход в Мир 1. Изначально последняя комната недоступна из-за незавершенной лестницы. Лестница восстанавливается путем завершения каждой из картин (всего 5), по одной для каждого мира, кроме последней. Часть лестницы можно восстановить, собрав все части головоломки каждого мира и правильно сложив их вместе.

Картины[]

В доме Тима есть несколько холстов, которые используются для размещения кусочков пазла; и когда головоломка решена, формируется картина. Каждая картина отражает свой уважаемый мир, вращаясь вокруг содержания учебников этого мира.

Пазл Мир 2

Пазл Мир 3

Пазл Мир 4

Пазл Мир 5

Пазл Мир 6

Пазл Мир 1

Звезда[]

Дом Тима, возможно, содержит самую легкую для получения звезду, но единственную звезду, которую невозможно получить. Верх звезды можно увидеть поверх картины «Мир 3», и игрок должен выровнять левый верхний и правый нижний углы головоломки, чтобы открыть звезду, а затем использовать платформу на головоломке «Мир 2», чтобы добраться до нее. Если головоломка была решена до получения звезды, то она навсегда блокируется до тех пор, пока игрок не перезапустит игру.

Мелочи[]

За пределами дома Тима можно увидеть созвездие Андромеды (включая настоящую Галактику Андромеды).

Классическое изображение Андромеды

Принцесса Андромеда навсегда прикована к скале в ночном небе. Это может быть изображение Тима по отношению к принцессе. Галактика отдельно, но всегда там.

Характеристики Milwaukee M18 BRAID-0

Общие
0092 RU — 37 (PDF)
Milwaukee
Оплетка M18-0 | 4933447891
Винтовой станок
4002395006663
7 70799
Характеристики
Цвет изделия Черный, серый, Красный
Тип изделия Ударный гайковерт
Производительность
Холостой ход (макс.

Римские цифры буквы: Как написать римские цифры на клавиатуре в Word или другой программе быстро

Римские цифры на клавиатуре ПК, телефона и планшета

Выучить их несложно:

  • 1 = I
  • 5 = V
  • 10 = X
  • 50 = L
  • 100 = C
  • 500 = D
  • 1000 = M

Все цифры от 1 до 100 (остальные по аналогии) можно посмотреть в таблице:

I 1 XXVI 26 LI 51 LXXVI 76
II 2 XXVII 27 LII 52 LXXVII 77
III 3 XXVIII 28 LIII 53 LXXVIII 78
IV 4 XXIX 29 LIV 54 LXXIX 79
V 5 XXX 30 LV 55 LXXX 80
VI 6 XXXI 31 LVI 56 LXXXI 81
VII 7 XXXII 32 LVII 57 LXXXII 82
VIII 8 XXXIII 33 LVIII 58 LXXXIII 83
IX 9 XXXIV 34 LIX 59 LXXXIV 84
X 10 XXXV 35 LX 60 LXXXV 85
XI 11 XXXVI 36 LXI 61 LXXXVI 86
XII 12 XXXVII 37 LXII 62 LXXXVII 87
XIII 13 XXXVIII 38 LXIII 63 LXXXVIII 88
XIV 14 XXXIX 39 LXIV 64 LXXXIX 89
XV 15 XL 40 LXV 65 XC 90
XVI 16 XLI 41 LXVI 66 XCI 91
XVII 17 XLII 42 LXVII 67 XCII 92
XVIII 18 XLIII 43 LXVIII 68 XCIII 93
XIX 19 XLIV 44 LXIX 69 XCIV 94
XX 20 XLV 45 LXX 70 XCV 95
XXI 21 XLVI 46 LXXI 71 XCVI 96
XXII 22 XLVII 47 LXXII 72 XCVII 97
XXIII 23 XLVIII 48 LXXIII 73 XCVIII 98
XXIV 24 XLIX 49 LXXIV 74 XCIX 99
XXV 25 L 50 LXXV 75 C 100

Где на клавиатуре римские цифры?

Отдельная нумерационная линейка на клавиатурах отсутствует. По своей сути римские цифры – это буквы латинского алфавита. Соответственно, для проставления нумерации мы переключаем раскладку на латиницу, чтобы вставить соответствующие буквы в верхнем регистре.

Как написать на клавиатуре персонального компьютера или ноутбука на операционной системе Windows?

Для перехода на латинский алфавит нам потребуется переключение раскладки клавиатуры на английскую версию. Это можно сделать так: в английской раскладке нажмите CapsLock и набирайте любые латинские буквы сразу в верхнем регистре.

Как набрать в Ворде (текстовый редактор Microsoft Office Word)?

Чтобы поставить римские цифры в текстовой программе Ворд, мы нажимаем заглавные буквы в английской версии, набирая нужное число.

Как ввести в тексте?

Для ввода цифр в римской системе используются следующие комбинации:

Комбинация клавиш

Описание

Shift+Ш (I) нужное количество раз

I, II, III

Shift+Ш (I)+М (V) нужное количество раз

IV, V, VI, VII, VIII

Shift+Ч (Х)+Ш (I)

X, IX, X

Shift+Д (L)

L

Shift+Ь (M)

M

Shift+C

C

Некоторые буквы аналогичны русским (М, С), их можно напечатать и в русской версии раскладки.

Нумерация списка

Для проставления автоматической нумерации воспользуйтесь панелью инструментов – нам нужен раздел «Абзац» и функция нумерованного списка (не с точками или комбинированный). Далее выбирайте соответствующий вариант нумерации с римскими обозначениями. Можно ввести подпункты с числовой или символической нумерацией.

Как напечатать в экселе (табличный редактор Microsoft Office Excel)?

Аналогично включают раскладку клавиатуры на английский вариант и набирают буквы в верхнем регистре (см. п. 2). Есть еще вариант «Вставка символа» через меню «Вставка», где можно скопировать из алфавита соответствующую букву, но это займет больше времени.

Как сделать римские цифры в PowerPoint (редактор презентаций)?

Так как римские цифры – это текст, здесь потребуется функция вставки текста из панели инструментов. Нажав на нее, вы создадите рабочее окно, где можно набирать англоязычные цифры.

Если требуется автоматическая нумерация пунктов для текста, активируют меню списков и выбирают вариант с римскими обозначениями, его легко найти из предлагаемых.

Как ввести на телефоне?

Андроид (Самсунг и другие).

Андроид – популярнейшая в мире операционная система, так как подходит для большинства смартфонов. Здесь набор римских цифр осуществляется через английскую раскладку. Нажимаем иконку с изображением глобуса, затем зажимаем клавишу Shift для повышения регистра – и можно набирать цифры.

Айфон

Операционная система iOs упрощена до предела, но принцип набора римских символов на клавиатуре айфона тот же. Нажимаем в телефонной клавиатуре  иконку с изображением глобуса, затем зажимаем клавишу Shift для повышения регистра – и можно набирать символы.

Как набрать на планшете (Айпад и др.)?

В англоязычной раскладке устанавливаем написание заглавными буквами через клавишу Shift и набираем буквы на клавиатуре.

Как напечатать в Опен офис?

Принцип аналогичен работе в процессоре Microsoft Word. Здесь можно скопировать буквы из набора в меню или писать вручную буквы английским текстом.

Ручной набор на макбуке или аймаке (MacOS)

Если нажать на клавиатуре Макбука комбинацию Command + Пробел, то можно переключить на английский язык, латиница станет доступной для набора.

Как набрать, используя ASCII-коды?

Это не самый удобный и современный способ набора нужного символа на ПК, но им тоже иногда требуется воспользоваться, когда работаете через комп. Для этой цели потребуется включение дополнительной клавиатуры NumLock. Затем используется параллельное нажатие клавиши Alt и цифр на нумпаде. Как только символ отобразится на экране, клавишу Alt надо отпустить, затем нажать для ввода следующего.

Используются такие комбинации:

Комбинация клавиш

Описание

Alt+73

I

Alt+86

V

Alt+88

X

Alt+76

L

Alt+67

C

Alt+68

D

Alt+77

M

Теперь вы знаете, как можно пронумеровать текст разными способами и на разных видах клавиатуры. Пользуйтесь тем, который наиболее удобен, чтобы четко и красиво пронумеровать любой список, проставить исторические даты и элегантно оформить документ в соответствии с нормативами, стандартами и общепринятыми требованиями!

Как написать римские цифры на клавиатуре для свидетельства о рождении

Главная » Компьютер

Автор admin На чтение 4 мин Просмотров 2.4к. Опубликовано

В номерах некоторых документов имеются не только арабские, но и римские цифры, а также латинские буквы. Указывая личные данные на различных сервисах, вы можете столкнуться с проблемой ввода этих символов. Сегодня мы разберемся, как написать римские цифры на клавиатуре для свидетельства о рождении и любых других документов.

Содержание

  1. Ввод латинскими буквами
  2. Ввод с помощью средств Word
  3. Если ничего не помогло
  4. Видеоинструкция
  5. Заключение

Ввод латинскими буквами

Итак, вы пытаетесь указать данные на Госуслугах, сайте РЖД или любом другом ресурсе, но остановились на вводе римских цифр. Первый способ, который следует применить на практике – это ввод заглавными буквами английского алфавита. Выполняется данная операция следующим образом:

  1. Сначала переведите раскладку клавиатуры на английский язык. Сделать это можно через панель задач или сочетание клавиш [knopka]Shift[/knopka]+[knopka]Alt[/knopka].
  2. Теперь перейдите в браузер ко вкладке с формой для заполнения.
  3. Поставьте курсор в поле для ввода номера свидетельства о рождении.
  4. Зажмите [knopka]Shift[/knopka] и впишите букву, аналогичную римской цифре. Просто скопируйте те же буквы, что указаны в документе.
  5. Завершите ввод номера документа оставшимися знаками.

В большинстве ситуаций на этом проблема разрешается – многие сайты и сервисы поддерживают такой вариант ввода. Однако иногда пользователи сталкиваются с ошибкой – номер подчеркивается красным, а ресурс не принимает документ, ссылаясь на неверные данные.

Ввод с помощью средств Word

Если вы все же столкнулись с неприятной ошибкой, описанной чуть выше, то не переживайте. Попробуем «обмануть» систему с помощью текстового редактора – впишем римские цифры в Microsoft Word, скопируем их и вставим в нужную форму. Для этого следуйте инструкции:

  1. Сначала включите цифровую клавиатуру NumPad с помощью клавиши [knopka]Num Lock[/knopka].

  1. Теперь с зажатой клавишей [knopka]Alt[/knopka] на цифровом блоке наберите число, которое соответствует римской цифре. Полный список комбинаций представлен ниже.
  2. Отпустите [knopka]Alt[/knopka]. На месте курсора появится введенный символ. По такому же принципу введите остальные символы. Остается только скопировать полученную комбинацию и вставить ее в форму для заполнения номера свидетельства о рождении.

Есть и альтернативный вариант ввода римских цифр в данном текстовом редакторе. Сначала нажмите сочетание [knopka]Ctrl[/knopka]+[knopka]F9[/knopka]. Появится поле с фигурными скобками. В нем впишите «=арабское число\*ROMAN». Для применения команды нажмите [knopka]F9[/knopka].

Готово! Теперь остается точно так же скопировать число в буфер обмена и вставить его в браузере, где вы заполняли сведения о документах ребенка. Оба варианта приводят к вводу одних и тех же символов, поэтому выбирать нужно только по удобству.

Мнение эксперта

Дарья Ступникова

Специалист по WEB-программированию и компьютерным системам. Редактор PHP/HTML/CSS сайта os-helper.ru.

Спросить у Дарьи

Вариант с Word пригодится и в том случае, если вы заполняете шаблон каких-либо документов перед печатью.

Если ничего не помогло

Если вы перепробовали все методы, но сайт все равно выдает ошибку, попробуйте сделать следующее:

  • Сначала попробуйте другой браузер. Авторизуйтесь на сайте через стандартный Edge или Explorer, а затем введите данные документа.
  • Очистите браузер от кэша и истории. Сделать это можно штатными средствами программы.
  • Обратитесь в техподдержку портала. Возможно, сбои связаны с неправильной работой самого сайта.

Видеоинструкция

Представленный видеоролик подробно отвечает на все вопросы по данной теме. Если вы не полностью разобрались в способах ввода римских цифр, то обязательно смотрите данное видео до того, как приступать к заполнению форм на сайтах!

Заключение

Итак, теперь вы умеете вводить римские цифры. Этот навык вам пригодится для регистрации любых документов в интернете. На некоторых ресурсах поддерживается ввод римских цифр латинскими буквами, а на других выдается ошибка. Во втором случае можно сначала ввести число в текстовом редакторе, затем просто скопировать его и вставить куда нужно.

Мы ждем ваших вопросов и комментариев к статье. Пишите, если возникли трудности при вводе римских цифр, а также расскажите, какой из способов наиболее удобен вам!

XII римские цифры | Как написать XII цифрами?

LearnPracticeDownload

XII Римские цифры можно записать в виде чисел, комбинируя преобразованные римские цифры, т. е. XII = X + I + I = 10 + 1 + 1 = 12. Старшие римские цифры предшествуют младшим, что обеспечивает правильный перевод XII римские цифры. В этой статье мы объясним, как преобразовать XII римские цифры в правильный перевод числа.

  • XII = X + I + I
  • XII = 10 + 1 + 1
  • XII = 12

Как написать XII римские цифры?

Числовое значение XII римских цифр можно получить, используя любой из двух методов, приведенных ниже:

Метод 1: В этом методе мы разбиваем римские цифры на отдельные буквы, пишем числовое значение каждой буквы и добавить/убрать их.

  • XII = Х + I + I = 10 + 1 + 1 = 12

Метод 2: В этом методе мы рассматриваем группы римских цифр для сложения или вычитания, например,

  • XII = X + II = 10 + 2 = 12

Следовательно, числовое значение XII римских цифр равно 12.

Также проверьте: Калькулятор римских цифр

Каковы основные правила написания римских цифр?

  • Когда буква большего размера предшествует букве меньшего размера, буквы добавляются. Например: CL, C > L, поэтому CL = C + L = 100 + 50 = 150
  • Когда буква меньшего размера предшествует букве большего размера, буквы вычитаются. Например: XC, X < C, поэтому XC = C - X = 100 - 10 = 90
  • Когда буква повторяется 2 или 3 раза, они добавляются. Например: II = I + I = 1 + 1 = 2
  • Одну и ту же букву нельзя использовать более трех раз подряд.

Числа, связанные с римскими цифрами XII

Римские цифры использовались в Древнем Риме и представляли собой комбинации букв латинского алфавита I, V, X, L, C, D и M. Может показаться, что они отличаются от цифр, но они похожи. Например, XII римские цифры эквивалентны числу 12. Римские цифры, относящиеся к XII, приведены ниже:

  • Х = 10
  • XI = 10 + 1 = 11
  • XII = 10 + 2 = 12
  • XIII = 10 + 3 = 13
  • XIV = 10 + 4 = 14
  • XV = 10 + 5 = 15
  • XVI = 10 + 6 = 16
  • XVII = 10 + 7 = 17
  • XVIII = 10 + 8 = 18
  • XIX = 10 + 9 = 19

Примеры XII римских цифр

  1. Пример 1: Найдите разницу между XII и IV.

    Решение:

    Римская цифра XII равна 12, а IV равна 4.
    Теперь XII — IV = 12 — 4 = 8
    Так как 8 = VIII
    Следовательно, XII — IV = VIII

  2. Пример 2. Найдите частное 12 и 11.

    Решение:

    Римская цифра XII равна 12, а XI равна 11.
    Теперь, когда мы делим XII на XI, т.е. 12 ÷ 11, в частном получается 1.
    Так как 1 = I
    Следовательно, XII ÷ XI = I

  3. Пример 3: Найдите произведение римских цифр XII и LXXXI.

    Решение:

    XII = 10 + 2 = 12 и LXXXI = 80 + 1 = 81
    Теперь XII × LXXXI = 12 × 81 = 972 
    . Так как CMLXXII = 900 + 70 + 2 = 972
    Следовательно, XII × LXXXI = CMLXXII

  4. Пример 4. Найдите сумму римских цифр MCCCXXIII и XII.

    Решение:

    MCCCXXIII = 1000 + 300 + 20 + 3 = 1323 и XII = 10 + 2 = 12
    Теперь MCCCXXIII + XII = 1323 + 12 = 1335
    . Так как MCCCXXXV = 1000 + 300 + 30 + 5 = 1335
    Следовательно, сумма римских цифр MCCCXXIII и XII равна MCCCXXXV

    .

перейти к слайдуперейти к слайдуперейти к слайдуперейти к слайду

 

Готовы увидеть мир глазами математика?

Математика лежит в основе всего, что мы делаем. Наслаждайтесь решением реальных математических задач на живых уроках и станьте экспертом во всем.

Запишитесь на бесплатный пробный урок

Часто задаваемые вопросы о XII римских цифрах

Что означают XII римские цифры?

Запишем XII римскими цифрами в развернутом виде, чтобы определить его значение. XII = X + I + I = 10 + 1 + 1 = 12. Следовательно, значение римских цифр XII равно 12.

Что нужно добавить к римским цифрам XII, чтобы получить MMMCMXCIX?

Сначала запишем MMMCMXCIX и XII цифрами, т.е. XII = 12 и MMMCMXCIX = 3999. Теперь 3999 — 12 = 3987. И, 3987 = MMMCMLXXXVII. Следовательно, к 12 римским цифрам нужно добавить MMMCMLXXXVII, чтобы получить MMMCMXCIX.

Почему 12 пишется римскими цифрами как XII?

Мы знаем, что римскими цифрами 2 записывается как II, а 10 как X. Следовательно, 12 римскими цифрами записывается как XII = X + II = 10 + 2 = XII.

Как преобразовать XII римские цифры в арабские числа?

Чтобы преобразовать XII римские цифры в числа, преобразование включает разбиение римских цифр на основе разрядности (единицы, десятки, сотни, тысячи), например:

  • Десятки = 10 = X
  • Единицы = 2 = II
  • Число = 12 = XII

Какой остаток при делении XII на VII?

XII = 12 и VII = 7 в цифрах. При делении 12 на 7 получается остаток 7. Теперь 5 = ​​V. Следовательно, при делении XII на VII получается остаток V.

  • LIX Римские цифры — 59
  • MMCDLI Римские цифры — 2451
  • MLII Римские цифры — 1052
  • VI Римские цифры — 6
  • XXXIV Римские цифры — 34
  • Римские цифры MMI — 2001
  • Рабочие листы по математике и
    наглядный учебный план

    XL Римские цифры | Как написать XL цифрами?

    LearnPracticeDownload

    XL Римские цифры можно записать в виде чисел, комбинируя преобразованные римские цифры, т. е. XL = (L — X) = (50 — 10) = 40. Старшие римские цифры предшествуют младшим цифрам, что обеспечивает правильный перевод XL римские цифры. В этой статье мы объясним, как преобразовать римские цифры XL в правильный перевод числа.

    • XL = 40

    Как писать римские цифры XL?

    Числовое значение римских цифр XL можно получить, используя любой из двух методов, приведенных ниже:

    Метод 1: В этом методе мы разбиваем римские цифры на отдельные буквы, пишем числовое значение каждой буквы и добавить/убрать их.

    • XL = (L — X) = (50 — 10) = 40

    Метод 2: В этом методе мы рассматриваем группы римских цифр для сложения или вычитания, например,

    • XL = 40

    Следовательно, числовое значение римских цифр XL равно 40.

    Также проверьте: Калькулятор римских цифр

    Каковы основные правила написания римских цифр?

    • Когда буква большего размера предшествует букве меньшего размера, буквы добавляются. Например: XI, X > I, поэтому XI = X + I = 10 + 1 = 11
    • Когда буква меньшего размера предшествует букве большего размера, буквы вычитаются. Например: IV, I < V, поэтому IV = V - I = 5 - 1 = 4
    • Когда буква повторяется 2 или 3 раза, они добавляются. Например: III = I + I + I = 1 + 1 + 1 = 3
    • Одну и ту же букву нельзя использовать более трех раз подряд.

    Числа, связанные с римскими цифрами XL

    Римские цифры использовались в Древнем Риме и представляли собой комбинации букв латинского алфавита I, V, X, L, C, D и M. Может показаться, что они отличаются от цифр, но они похожи. Например, римские цифры XL эквивалентны числу 40. Римские цифры, относящиеся к XL, приведены ниже:

    • XL = 40
    • XLI = 40 + 1 = 41
    • XLII = 40 + 2 = 42
    • XLIII = 40 + 3 = 43
    • XLIV = 40 + 4 = 44
    • XLV = 40 + 5 = 45
    • XLVI = 40 + 6 = 46
    • XLVII = 40 + 7 = 47
    • XLVIII = 40 + 8 = 48
    • XLIX = 40 + 9 = 49

    Примеры римских цифр XL

    1. Пример 1. Найдите сумму MMCMLI и XL римских цифр.

      Решение:

      MMCMLI = 2000 + 900 + 50 + 1 = 2951 и XL = 40
      Теперь MMCMLI + XL = 2951 + 40 = 2991
      . Так как MMCMXCI = 2000 + 900 + 90 + 1 = 2991
      Следовательно, сумма MMCMLI и римских цифр XL равна MMCMXCI

      .
    2. Пример 2: Найдите разницу между XL и VIII.

      Решение:

      Римская цифра XL равна 40, а VIII равна 8.
      Теперь XL — VIII = 40 — 8 = 32
      Так как 32 = XXXII
      Следовательно, XL — VIII = XXXII

      .
    3. Пример 3. Найдите частное 40 и 26.

      Решение:

      Римская цифра XL равна 40, а XXVI равна 26.
      Теперь, когда мы делим XL на XXVI, то есть 40 ÷ 26, в частном получается 1.
      Так как 1 = I
      Следовательно, XL ÷ XXVI = I

    4. Пример 4. Найдите произведение римских цифр XL и LXXXVIII.

      Решение:

      XL = 40 и LXXXVIII = 80 + 8 = 88
      Теперь XL × LXXXVIII = 40 × 88 = 3520
      . Так как MMMDXX = 3000 + 500 + 20 = 3520
      Следовательно, XL × LXXXVIII = MMMDXX

      .

    перейти к слайдуперейти к слайдуперейти к слайдуперейти к слайду

     

    Готовы увидеть мир глазами математика?

    Математика лежит в основе всего, что мы делаем. Наслаждайтесь решением реальных математических задач на живых уроках и станьте экспертом во всем.

    Запишитесь на бесплатный пробный урок

    Часто задаваемые вопросы о римских цифрах XL

    Какова ценность римских цифр XL?

    Напишем XL римскими цифрами в развернутом виде для определения его номинала. XL = 40. Следовательно, значение римских цифр XL равно 40.

    Сколько будет остатка при делении XL на XVIII?

    XL = 40 и XVIII = 18 цифрами. При делении 40 на 18 получается остаток 18. Теперь 4 = IV. Следовательно, когда XL делится на XVIII, остаток равен IV.

    Как римские цифры XL пишутся цифрами?

    Чтобы преобразовать римские цифры XL в числа, необходимо разбить римские цифры на основе разрядности (единицы, десятки, сотни, тысячи), например:

    • Десятки = 40 = XL
    • Номер = 40 = XL

    Почему 40 пишется римскими цифрами как XL?

    Мы знаем, что римскими цифрами мы пишем 40 как XL. Поэтому 40 римскими цифрами записывается как XL = 40.

    Что нужно вычесть из XL, чтобы получить X?

    Сначала запишем XL и X цифрами, т.

    Сложение и вычитание деление и умножение с разными знаками: Сложение и вычитание целых чисел с разными знаками

    Сложение, вычитание, деление десятичных дробей

    • Сложение и вычитание десятичных дробей
    • Умножение десятичных дробей
    • Деление десятичной дроби на целое число
    • Деление одной десятичной дроби на другую

    Сложение и вычитание десятичных дробей

    Эти операции выполняются так же, как и сложение и вычитание целых чисел. Необходимо только записать соответствующие десятичные знаки один под другим.

    Пример:

    Сложение и вычитание в столбик

    Умножение десятичных дробей

    Умножение десятичных дробей производится так же, как и умножение натуральных чисел, по тем же правилам, но в произведении ставится запятая по сумме разрядов множителей в дробной части, считая справа налево (сумма разрядов множителей — это количество разрядов после запятой у множителей, вместе взятых).

    Пример:

    На первом этапе перемножаем десятичные дроби как целые числа, не принимая во внимание десятичную точку. Затем применяется следующее правило:

    количество десятичных знаков в произведении равно сумме десятичных знаков во всех сомножителях.

    Замечание до простановки десятичной точки в произведении нельзя отбрасывать нули в конце!

    Пример:

    Сумма чисел десятичных знаков в сомножителях равна: 3 + 4 = 7

    Сумма цифр в произведении равна 6. Поэтому необходимо добавить один ноль слева: 0197056 и проставить перед ним десятичную точку: 0.0197056.

    При умножении десятичных дробей в столбик первая справа значащая цифра подписывается под первой справа значащей цифрой, как и в натуральных числах:

    Запись умножения десятичных дробей в столбик:

    Деление десятичной дроби на целое число

    Если делимое меньше делителя, записываем ноль в целой части частного и ставим после него десятичную точку.

    Затем, не принимая во внимание десятичную точку делимого, присоединяем к его целой части следующую цифру дробной части и опять сравниваем полученную целую часть делимого с делителем.

    Если новое число опять меньше делителя, ставим ещё один ноль после десятичной точки в частном и присоединяем к целой части делимого следующую цифру его дробной части.

    Этот процесс повторяем до тех пор, пока полученное делимое не станет больше делителя.

    После этого деление выполняется, как для целых чисел.

    Если делимое больше делителя или равно ему, сначала делим его целую часть, записываем результат деления в частном и ставим десятичную точку. После этого деление продолжается, как в случае целых чисел.

    Пример: Разделить 1.328 на 64

    Деление одной десятичной дроби на другую

    Сначала переносим десятичные точки в делимом и делителе на число десятичных знаков в делителе, то есть делаем делитель целым числом. Теперь выполняем деление, как в предыдущем случае.

    Пример: Разделить 0.04569 на 0.0006

    Переносим десятичные точки на 4 позиции вправо и делим 456.9 на 6:

    Запись деления десятичных дробей в столбик:

    складываются и вычитаются по-разрядово — целые под целыми, десятичные под десятичными
    2,35+
    1,402
    ____
    3,752
    умножаются как простые числа в столбик, только еще нужно подсчитать общее число знаков после запятой: 2,5 * 2,4 = 25 * 24 = 600


    Виленкин Математика Учебник (Мнемозина) 6 класс

    Математика 6 класс УЧЕБНИК 2021 в 2-х частях (УМК Виленкин, Жохов, Чесноков и др. ) Ознакомительная версия перед покупкой. Цитаты из учебного пособия использованы в учебных целях. При постоянном использовании учебника необходимо купить книгу: «Математика. 6 класс : учебник для общеобразоват. учреждений / Н. Я. Виленкин, В. И. Жохов, А. С. Чесноков, С. И. Шварцбурд — М. : Мнемозина» (переход по ссылке в Интернет-магазин). На нашем сайте нет материалов для скачивания!

    Проверенный временем учебник полностью соответствует Примерной основной образовательной программе по математике и ФГОС ООО. Разработан с учётом возрастных и гендерных особенностей восприятия материала учащимися. Глубоко продуманная последовательность подачи теоретического и практического материала эффективно развивает мышление, память и речь учащихся.

     

    Часть I. ОБЫКНОВЕННЫЕ ДРОБИ

    § 1. Делимость чисел.

    1. Делители и кратные. Задачи 1 — 30 с ответами

    2. Признаки делимости на 10, на 5 и на 2. Задачи 31 — 62 с ответами

    3. Признаки делимости на 9 и на 3. Задачи 63 — 95 с ответами

    4. Простые и составные числа. Задачи 96 — 124 с ответами

    5. Разложение на простые множители. Задачи 125 — 149 с ответами

    6. Наибольший общий делитель. Взаимно простые числа. Задачи 150 — 182 с ответами

    7. Наименьшее общее кратное. Задачи 183 — 215 с ответами

    Задания для самопроверки. Проектные задачи.

     

    § 2. Сложение и вычитание дробей с разными знаменателями.

    8. Основное свойство дроби. Задачи 216 — 246 с ответами

    9. Сокращение дробей. Задачи 247 — 279 с ответами

    10. Приведение дробей к общему знаменателю. Задачи 280 — 308 с ответами

    11. Сравнение, сложение и вычитание дробей с разными знаменателями. Задачи 309 — 340 с ответами  Задачи 341 — 380 с ответами

    12. Сложение и вычитание смешанных чисел. Задачи 381 — 431 с ответами

    Задания для самопроверки. Проектные задачи.

     

    § 3. Умножение и деление обыкновенных дробей.

    13. Умножение дробей. Задачи 432 — 488 с ответами

    14. Нахождение дроби от числа. Задачи 489 — 540 с ответами

    15. Применение распределительного свойства умножения.   Задачи 541 — 581 с ответами

    16. Взаимно обратные числа.   Задачи 582 — 600 с ответами

    17. Деление.   Задачи 601 — 652 с ответами

    18. Нахождение числа до его дроби.   Задачи 653 — 697 с ответами

    19. Дробные выражения.   Задачи 698 — 727 с ответами

    Задания для самопроверки. Проектные задачи.

     

    § 4. Отношения и пропорции.

    20. Отношения.

    21. Пропорции.

    22. Прямая и обратная пропорциональные зависимости.

    23. Масштаб.

    24. Длина окружности и площадь круга.

    25. Шар.

    Задания для самопроверки. Проектные задачи.

     

    Часть II. РАЦИОНАЛЬНЫЕ ЧИСЛА

    § 5. Положительные и отрицательные числа.

    26. Координаты на прямой.

    27. Противоположные числа.

    28. Модуль числа.

    29. Сравнение чисел.

    30. Изменение величин.

    Задания для самопроверки. Проектные задачи.

     

    § 6. Сложение и вычитание положительных и отрицательных чисел

    31. Сложение чисел с помощью координатной прямой.

    32. Сложение отрицательных чисел.

    33. Сложение чисел с разными знаками.

    34. Вычитание.

    Задания для самопроверки. Проектные задачи.

     

    § 7. Умножение и деление положительных и отрицательных чисел.

    35. Умножение.

    36. Деление.

    37. Рациональные числа.

    38. Свойства действий с рациональными числами.

    Задания для самопроверки. Проектные задачи.

     

    § 8. Решение уравнений.

    39. Раскрытие скобок.

    40. Коэффициент.

    41. Подобные слагаемые.

    42. Решение уравнений.

    Задания для самопроверки. Проектные задачи.

     

    § 9. Координаты на плоскости

    43. Перпендикулярные прямые

    44. Параллельные прямые.

    45. Координатная плоскость.

    46. Столбчатые диаграммы.

    47. Графики.

    Задания для самопроверки.

    Итоговое повторение.

     


    Вы смотрели: Математика 6 класс УЧЕБНИК 2021 в 2-х частях (УМК Виленкин, Жохов, Чесноков и др.) Цитаты из учебного пособия использованы в учебных целях.

    Математика 6 класс (Виленкин) ОГЛАВЛЕНИЕ:

    Часть I. ОБЫКНОВЕННЫЕ ДРОБИ

    § 1. Делимость чисел.

    § 2. Сложение и вычитание дробей с разными знаменателями.

    § 3. Умножение и деление обыкновенных дробей.

    § 4. Отношения и пропорции.

    Часть II. РАЦИОНАЛЬНЫЕ ЧИСЛА

    § 5. Положительные и отрицательные числа.

    § 6. Сложение и вычитание положительных и отрицательных чисел

    § 7. Умножение и деление положительных и отрицательных чисел.

    § 8. Решение уравнений.

    § 9. Координаты на плоскости

    Первая часть учебника посвящена изучению обыкновенных дробей, отношений и пропорций. Вторая часть учебника посвящена изучению рациональных чисел, составлению и решению уравнений, основам наглядного представления данных.

    Разделение целых чисел Проблемы с словом-Google Suce

    AllebilderVideOsnewsmapsShoppingBücher

    Sucoptionen

    Разделение целых чисел. Мы можем использовать целочисленное деление для решения реальных задач. Для некоторых задач нам может потребоваться выполнить более одного шага.

    Словесные задачи на умножение и деление отрицательных чисел (практика)

    www. khanacademy.org › cc-seventh-grade-math

    Узнайте бесплатно о математике, искусстве, программировании, экономике, физике, химии, биологии, медицине, финансах, истории и многом другом. Khan Academy – это …

    [PDF] Умножение и деление целых чисел

    www.washoeschools.net › cms › lib › Centricity › Domain

    Это может быть представлено путем умножения целых чисел с разными знаками. Как можно использовать умножение и деление целых чисел для решения реальных задач?

    Задачи на умножение и деление целых чисел — YouTube

    www.youtube.com › смотреть

    10.10.2020 · Задачи на умножение и деление целых чисел … В этом уроке объясняется, как использовать стратегии для …
    Dauer: 7:25
    Прислан: 10.10.2020

    Ähnliche Fragen

    Какие примеры задач с целочисленными словами?

    Какой пример задачи на деление целых чисел?

    [DOC] Целочисленные задачи Word — Test Practice.docx

    www. gsiccharter.com › cms › lib › Centricity › Domain › Integer Wo…

    Division Integer Word Problems. Четыре инвестора потеряли 24 процента своих совокупных инвестиций в компанию. Сколько в среднем потерял каждый инвестор?

    Словесная задача на деление целых чисел — Живые рабочие листы Факты Проблемы со словами.

    Как решать задачи со словами на умножение или деление целых чисел

    study.com › навык › учиться › как решать задачи со словами… на словах. Эти типы задач часто моделируют явления реального мира с помощью математики. Integer: An …

    Задачи на умножение и деление целочисленных слов — Quizizz

    quizizz.com › admin › викторина › умножение и деление-i…

    Задачи на умножение и деление целых чисел … Когда умножая целые числа с одинаковыми знаками, ответ будет… Положительным. Отрицательный. нуль. фракция.

    Умножение и деление целых чисел и словесные задачи — Викторина

    quizizz. com › admin › викторина › умножение и деление-i…

    Умножение и деление целых чисел и словесные задачи … При умножении целых чисел с те же знаки, например, . положительный X положительный. ИЛИ. отрицательный X отрицательный,.

    Целое деление | Math Goodies

    www.mathgoodies.com › уроки › ​​том 5 › подразделение

    Проблема: Миссис Дженсон должна 8000 долларов по кредиту на покупку автомобиля. Каждый из ее 4 детей готов платить равную долю этого кредита. Используя целые числа, определите, сколько …

    Целые числа: коэффициент

    ähnliche sucalfragen

    Разделение целых чисел. Проблемы с словом pdf

    Разделение целых чисел слов Проблем Текстовые задачи на умножение и деление целых чисел

    Текстовые задачи на умножение и деление целых чисел с ответами

    Словесные задачи на умножение и деление целых чисел

    модульная арифметика — сложение, вычитание, умножение и деление по обе стороны отношения конгруэнтности

    Задавать вопрос

    спросил

    Изменено 7 месяцев назад

    Просмотрено 8к раз

    $\begingroup$

    Этот пост должен развеять некоторые сомнения, связанные с арифметикой по модулю.

    Дополнение с обеих сторон

    $a \equiv b \pmod p \iff$ $(a+k) \equiv (b+k) \pmod p$

    Вычитание с обеих сторон

    $a \equiv b \pmod p \iff$ $(a-k) \equiv (b-k) \pmod p$

    Умножение с обеих сторон

    $a \equiv b \pmod p \iff$ $ak \equiv bk \pmod p$

    Но для деления,

    $a \equiv b \pmod p \iff$ $a/k \equiv b/k \pmod {p/k}$
    (если $a/k, b/k$ и $p/k$ целые числа)

    Правильно ли я понимаю? Существует ли альтернативная форма деления или почему общая форма отличается от деления? Пожалуйста, помогите мне понять это.

    • модульно-арифметический

    $\endgroup$

    1

    $\begingroup$

    Попробуйте несколько чисел для деления и посмотрите, сможете ли вы понять, почему. 20 = 8 по модулю 12, поэтому 10 = 4 по модулю 12. Почему это не работает?

    Примечание $a = b \mod n$ означает $a = m*n + b$, поэтому $a/k = (m*n)/k + b/k$ при $k|a; к|м*п; к|б$. Ну, мы хотим, чтобы сказал, что это означает $a/k = (m/k)*n + b/k$. Но что, если $k \не \mid m$, а $k | nm$

    Вот что происходит с $20 = 1*12 + 8$ и $20/2 = 1*12/2 = 8/2$. $2 \не \mid 1$, поэтому вместо , мы получаем $20/2 = 12/2 + 8/2$, поэтому $10 = 6 + 4$, поэтому $10 \эквив 4 \mod 6$, а не $10 \эквив 4 \mod 12 $.

    Значит деление сложнее. Is иногда работает , а иногда нет.

    Например, $24 \экв 4 \mod 5$ и $12 \экв 2 \mod 5$. В этом случае $\gcd(5,2) = 1$, так что если $2|24$ и $2|4$, то $2\не \mid 5$, поэтому если $24 = 5*k + 4$, то $\gcd(2, 5) = 1$, поэтому $2|k$, поэтому мы делаем , получаем $12 = 5*(k/2) + 2$.

    $\endgroup$

    2

    $\begingroup$

    Вы правильно понимаете первые три определения, но в последнем у вас должно быть $\frac{p}{\gcd(p,k)}$ по модулю, поскольку $p$ может не делиться на $k $.

    Интеграл арктангенса: ∫ Найти интеграл от y = f(x) = atan(y/x) dx (арктангенс от (у делить на х))

    анализ / Криволинейный интеграл $%∫arctg(y/x)dl$% / Математика

    $$∫arctg(y/x)dl$$

    $%L$%: дуга кардиоиды $%ρ=1+\cos φ; 0 ≤ ρ ≤ \pi/2$%, в ответе: $%\sqrt{2}(\pi+4)-8$%

    У меня получилось в конце интеграл от $%0$% до $%\pi/2$% $%∫φ\sqrt{2+2\cos φ}dφ$% и в ответе $%3\sqrt{2}\pi$%

    кратные-интегралы анализ

    задан 18 Мар ’13 9:44

    piratescnrcu…
    1●1●5

    изменен 18 Мар ’13 14:53

    falcao
    291k●9●38●53

    старыеновыеценные

    В полярных координатах $$ dl=\sqrt{(x'(\varphi))^2+y'(\varphi))^2}\;d\varphi , \\ x'(\varphi)=\frac{d}{d\varphi}(\rho(\varphi)\cos{\varphi}) =-\sin{\varphi}\cos{\varphi}-(1+\cos{\varphi})\sin{\varphi} ,\\ y'(\varphi)=\frac{d}{d\varphi}(\rho(\varphi)\sin{\varphi}) =-\sin{\varphi}\sin{\varphi}+(1+\cos{\varphi})\cos{\varphi}, \\ x'(\varphi))^2+y'(\varphi))^2= \sin^2{\varphi}+(1+\cos{\varphi})^2=2+2\cos{\varphi}=4\cos^2{\frac{\varphi}{2}}, \\ dl={2}\left| \cos{\frac{\varphi}{2}} \right|\;d\varphi, \\ \int\limits_{0}^{\frac{\pi}{2}} \operatorname{arctg}{\frac{y}{x}}\;dl= {2}\int\limits_{0}^{\frac{\pi}{2}} \operatorname{arctg}({\operatorname{tg{\varphi}}})\left| \cos{\frac{\varphi}{2}} \right|\;d\varphi= \\ ={2}\int\limits_{0}^{\frac{\pi}{2}} {\varphi} \cos{\frac{\varphi}{2}} \;d\varphi. $$ Дальше — интегрирование частями.

    ссылка

    отвечен 18 Мар ’13 10:57

    Mather
    3.2k●1●7

    Ваш ответ

    Если вы не нашли ответ, задайте вопрос.

    Здравствуйте

    Математика — это совместно редактируемый форум вопросов и ответов для начинающих и опытных математиков, с особенным акцентом на компьютерные науки.

    Присоединяйтесь!

    регистрация »

    отмечен:

    анализ ×439
    кратные-интегралы ×106

    задан
    18 Мар ’13 9:44

    показан
    2680 раз

    обновлен
    18 Мар ’13 15:01

    Связанные вопросы

    Отслеживать вопрос

    по почте:

    Зарегистрировавшись, вы сможете подписаться на любые обновления

    по RSS:

    Ответы

    Ответы и Комментарии

    Интегрирование подстановкой (внесение под знак дифференциала). Вторая часть.

    Высшая математика » Неопределённые интегралы » Интегрирование подстановкой » Вторая часть. 3x}}{3}+C$.

    Первая часть

    Вторая часть

    Вернуться к списку тем

    Задать вопрос на форуме

    Записаться на занятия

    Онлайн-занятия по высшей математике

    Мэтуэй | Популярные задачи

    92) 9(3x) по отношению к x 92+1
    1 Найти производную — d/dx бревно натуральное х
    2 Оценить интеграл интеграл натурального логарифма x относительно x
    3 Найти производную — d/dx
    21 Оценить интеграл интеграл от 0 до 1 кубического корня из 1+7x относительно x
    22 Найти производную — d/dx грех(2x)
    23 Найти производную — d/dx
    41 Оценить интеграл интеграл от cos(2x) относительно x
    42 Найти производную — d/dx 1/(корень квадратный из х)
    43 Оценка интеграла 9бесконечность
    45 Найти производную — d/dx х/2
    46 Найти производную — d/dx -cos(x)
    47 Найти производную — d/dx грех(3x)
    68 Оценить интеграл интеграл от sin(x) по x
    69 Найти производную — d/dx угловой синус(х)
    70 Оценить предел ограничение, когда x приближается к 0 из (sin(x))/x 92 по отношению к х
    85 Найти производную — d/dx лог х
    86 Найти производную — d/dx арктан(х)
    87 Найти производную — d/dx бревно натуральное 5х92

    5.

    7 Интегралы, приводящие к обратным тригонометрическим функциям

    В этом разделе мы сосредоточимся на интегралах, приводящих к обратным тригонометрическим функциям. Мы уже работали с этими функциями. Напомним из «Функций и графиков», что тригонометрические функции не являются взаимно однозначными, если только домены не ограничены. При работе с обратными тригонометрическими функциями нам всегда нужно быть осторожными, чтобы учитывать эти ограничения. Также в «Производных» мы разработали формулы для производных обратных тригонометрических функций. Разработанные там формулы непосредственно приводят к формулам интегрирования с обратными тригонометрическими функциями. 9{-1}\frac{x}{a}.[/latex] Тогда [latex]a \sin y=x.[/latex] Теперь воспользуемся неявным дифференцированием. Получаем

    [латекс]\begin{array}{ccc}\hfill \frac{d}{dx}(a \sin y)& =\hfill & \frac{d}{dx}(x)\hfill \ \ \\ \hfill a \cos y\frac{dy}{dx}& =\hfill & 1\hfill \\ \hfill \frac{dy}{dx}& =\hfill & \frac{1}{a \ cos y}.

    Решение квадратных уравнений примеры: 8.2.2. Решение полных квадратных уравнений.

    2 +b*x+c. Его еще называют квадратным трехчленом.

    », то есть уравнения первой степени. В этом уроке мы разберем, что называют квадратным уравнением и как его решать.

    Что называют квадратным уравнением

    Важно!

    Степень уравнения определяют по наибольшей степени, в которой стоит неизвестное.

    Если максимальная степень, в которой стоит неизвестное — «2 », значит, перед вами квадратное уравнение.

    Примеры квадратных уравнений

    • 5x 2 − 14x + 17 = 0
    • −x 2 + x + = 0
    • x 2 + 0,25x = 0
    • x 2 − 8 = 0

    Важно! Общий вид квадратного уравнения выглядит так:

    A x 2 + b x + c = 0

    «a », «b » и «c » — заданные числа.

    • «a » — первый или старший коэффициент;
    • «b » — второй коэффициент;
    • «c » — свободный член.

    Чтобы найти «a », «b » и «c » нужно сравнить свое уравнение с общим видом квадратного уравнения «ax 2 + bx + c = 0 ».

    Давайте потренируемся определять коэффициенты «a », «b » и «c » в квадратных уравнениях.

    5x 2 − 14x + 17 = 0 −7x 2 − 13x + 8 = 0 −x 2 + x + = 0 x 2 + 0,25x = 0
    Уравнение Коэффициенты
    • a = 5
    • b = −14
    • с = 17
    • a = −7
    • b = −13
    • с = 8
    • a = −1
    • b = 1
    • с =
    • a = 1
    • b = 0,25
    • с = 0
    x 2 − 8 = 0
    • a = 1
    • b = 0
    • с = −8

    Как решать квадратные уравнения

    В отличии от линейных уравнений для решения квадратных уравнений используется специальная формула для нахождения корней .

    Запомните!

    Чтобы решить квадратное уравнение нужно:

    • привести квадратное уравнение к общему виду «ax 2 + bx + c = 0 ». То есть в правой части должен остаться только «0 »;
    • использовать формулу для корней:

    Давайте на примере разберем, как применять формулу для нахождения корней квадратного уравнения. Решим квадратное уравнение.

    X 2 − 3x − 4 = 0

    Уравнение « x 2 − 3x − 4 = 0 » уже приведено к общему виду «ax 2 + bx + c = 0 » и не требует дополнительных упрощений. Для его решения нам достаточно применить формулу нахождения корней квадратного уравнения .

    Определим коэффициенты «a », «b » и «c » для этого уравнения.

    x 1;2 =
    x 1;2 =
    x 1;2 =
    x 1;2 =

    С её помощью решается любое квадратное уравнение.

    В формуле «x 1;2 = » часто заменяют подкоренное выражение
    «b 2 − 4ac » на букву «D » и называют дискриминантом . Более подробно понятие дискриминанта рассматривается в уроке «Что такое дискриминант ».

    Рассмотрим другой пример квадратного уравнения.

    x 2 + 9 + x = 7x

    В данном виде определить коэффициенты «a », «b » и «c » довольно сложно. Давайте вначале приведем уравнение к общему виду «ax 2 + bx + c = 0 ».

    X 2 + 9 + x = 7x
    x 2 + 9 + x − 7x = 0
    x 2 + 9 − 6x = 0
    x 2 − 6x + 9 = 0

    Теперь можно использовать формулу для корней.

    X 1;2 =
    x 1;2 =
    x 1;2 =
    x 1;2 =
    x =

    x = 3
    Ответ: x = 3

    Бывают случаи, когда в квадратных уравнениях нет корней. Такая ситуация возникает, когда в формуле под корнем оказывается отрицательное число.

    Видеоурок 2: Решение квадратных уравнений

    Лекция: Квадратные уравнения

    Уравнение

    Уравнение — это некое равенство, в выражениях которого имеется переменная.

    Решить уравнение — значит найти такое число вместо переменной, которое будет приводить его в верное равенство.

    Уравнение может иметь одно решение или несколько, или же не иметь его вообще.

    Для решения любого уравнения его следует максимально упростить до вида:

    Линейное: a*x = b;

    Квадратное: a*x 2 + b*x + c = 0.

    То есть любые уравнение перед решением нужно преобразовать до стандартного вида.

    Любое уравнение можно решить двумя способами: аналитическим и графическим.

    На графике решением уравнения считаются точки, в которых график пересекает ось ОХ.

    Квадратные уравнения

    Уравнение можно назвать квадратным, если при упрощении оно приобретает вид:

    a*x 2 + b*x + c = 0.

    При этом a, b, c являются коэффициентами уравнения, отличающиеся от нуля. А «х» — корень уравнения. Считается, что квадратное уравнение имеет два корня или могут не иметь решения вообще. Полученные корни могут быть одинаковыми.

    «а» — коэффициент, который стоит перед корнем в квадрате.

    «b» — стоит перед неизвестной в первой степени.

    «с» — свободный член уравнения.

    Если, например, мы имеем уравнение вида:

    2х 2 -5х+3=0

    В нем «2» — это коэффициент при старшем члене уравнения, «-5» — второй коэффициент, а «3» — свободный член.

    Решение квадратного уравнения

    Существует огромное множество способов решения квадратного уравнения. Однако, в школьном курсе математики изучается решение по теореме Виета, а также с помощью дискриминанта.

    Решение по дискриминанту:

    При решении с помощью данного метода необходимо вычислить дискриминант по формуле:

    Если при вычислениях Вы получили, что дискриминант меньше нуля, это значит, что данное уравнение не имеет решений.

    Если дискриминант равен нулю, то уравнение имеет два одинаковых решения. В таком случае многочлен можно свернуть по формуле сокращенного умножения в квадрат суммы или разности. После чего решить его, как линейное уравнение. Или воспользоваться формулой:

    Если же дискриминант больше нуля, то необходимо воспользоваться следующим методом:

    Теорема Виета

    Если уравнение приведенное, то есть коэффициент при старшем члене равен единице, то можно воспользоваться теоремой Виета .

    Итак, предположим, что уравнение имеет вид:

    Корни уравнения находятся следующим образом:

    Неполное квадратное уравнение

    Существует несколько вариантов получения неполного квадратного уравнения, вид которых зависит от наличия коэффициентов.

    1. Если второй и третий коэффициент равен нулю (b = 0, с = 0) , то квадратное уравнение будет иметь вид:

    Данное уравнение будет иметь единственное решение. Равенство будет верным только в том случае, когда в качестве решения уравнения будет ноль.

    В данной статье мы рассмотрим решение неполных квадратных уравнений.

    Но сначала повторим какие уравнения называются квадратными. Уравнение вида ах 2 + bх + с = 0, где х – переменная, а коэффициенты а, b и с некоторые числа, причем а ≠ 0, называется квадратным . Как мы видим коэффициент при х 2 не равен нулю, а следовательно коэффициенты при х или свободный член могут равняться нулю, в этом случае мы и получаем неполное квадратное уравнение.

    Неполные квадратные уравнения бывают трех видов :

    1) Если b = 0, с ≠ 0, то ах 2 + с = 0;

    2) Если b ≠ 0, с = 0, то ах 2 + bх = 0;

    3) Если b= 0, с = 0, то ах 2 = 0.

    • Давайте разберемся как решаются уравнения вида ах 2 + с = 0.

    Чтобы решить уравнение перенесем свободный член с в правую часть уравнения, получим

    ах 2 = ‒с. Так как а ≠ 0, то разделим обе части уравнения на а, тогда х 2 = ‒с/а.

    Если ‒с/а > 0 , то уравнение имеет два корня

    x = ±√(–c/a) .

    Если же ‒c/a

    Давайте попробуем разобраться на примерах, как решать такие уравнения.

    Пример 1 . Решите уравнение 2х 2 ‒ 32 = 0.

    Ответ: х 1 = ‒ 4, х 2 = 4.

    Пример 2 . Решите уравнение 2х 2 + 8 = 0.

    Ответ: уравнение решений не имеет.

    • Разберемся как же решаются уравнения вида ах 2 + bх = 0.

    Чтобы решить уравнение ах 2 + bх = 0, разложим его на множители, то есть вынесем за скобки х, получим х(ах + b) = 0. Произведение равно нулю, если хотя бы один из множителей равен нулю. Тогда или х = 0, или ах + b = 0. Решая уравнение ах + b = 0, получим ах = ‒ b, откуда х = ‒ b/a. Уравнение вида ах 2 + bх = 0, всегда имеет два корня х 1 = 0 и х 2 = ‒ b/a. Посмотрите как выглядит на схеме решение уравнений этого вида.

    Закрепим наши знания на конкретном примере.

    Пример 3 . Решить уравнение 3х 2 ‒ 12х = 0.

    х(3х ‒ 12) = 0

    х= 0 или 3х – 12 = 0

    Ответ: х 1 = 0, х 2 = 4.

    • Уравнения третьего вида ах 2 = 0 решаются очень просто.

    Если ах 2 = 0, то х 2 = 0. Уравнение имеет два равных корня х 1 = 0, х 2 = 0.

    Для наглядности рассмотрим схему.

    Убедимся при решении примера 4, что уравнения этого вида решаются очень просто.

    Пример 4. Решить уравнение 7х 2 = 0.

    Ответ: х 1, 2 = 0.

    Не всегда сразу понятно какой вид неполного квадратного уравнения нам предстоит решить. Рассмотрим следующий пример.

    Пример 5. Решить уравнение

    Умножим обе части уравнения на общий знаменатель, то есть на 30

    Сократим

    5(5х 2 + 9) – 6(4х 2 – 9) = 90.

    Раскроем скобки

    25х 2 + 45 – 24х 2 + 54 = 90.

    Приведем подобные

    Перенесем 99 из левой части уравнения в правую, изменив знак на противоположный

    Ответ: корней нет.

    Мы разобрали как решаются неполные квадратные уравнения. Надеюсь, теперь у вас не будет сложностей с подобными заданиями. Будьте внимательны при определении вида неполного квадратного уравнения, тогда у вас все получится.

    Если у вас появились вопросы по данной теме, записывайтесь на мои уроки , мы вместе решим возникшие проблемы.

    сайт, при полном или частичном копировании материала ссылка на первоисточник обязательна.

    Квадратные уравнения изучают в 8 классе, поэтому ничего сложного здесь нет. Умение решать их совершенно необходимо.

    Квадратное уравнение — это уравнение вида ax 2 + bx + c = 0, где коэффициенты a , b и c — произвольные числа, причем a ≠ 0.

    Прежде, чем изучать конкретные методы решения, заметим, что все квадратные уравнения можно условно разделить на три класса:

    1. Не имеют корней;
    2. Имеют ровно один корень;
    3. Имеют два различных корня.

    В этом состоит важное отличие квадратных уравнений от линейных, где корень всегда существует и единственен. Как определить, сколько корней имеет уравнение? Для этого существует замечательная вещь — дискриминант .

    Дискриминант

    Пусть дано квадратное уравнение ax 2 + bx + c = 0. Тогда дискриминант — это просто число D = b 2 − 4ac .

    Эту формулу надо знать наизусть. Откуда она берется — сейчас неважно. Важно другое: по знаку дискриминанта можно определить, сколько корней имеет квадратное уравнение. А именно:

    1. Если D
    2. Если D = 0, есть ровно один корень;
    3. Если D > 0, корней будет два.

    Обратите внимание: дискриминант указывает на количество корней, а вовсе не на их знаки, как почему-то многие считают. Взгляните на примеры — и сами все поймете:

    Задача. Сколько корней имеют квадратные уравнения:

    1. x 2 − 8x + 12 = 0;
    2. 5x 2 + 3x + 7 = 0;
    3. x 2 − 6x + 9 = 0.

    Выпишем коэффициенты для первого уравнения и найдем дискриминант:
    a = 1, b = −8, c = 12;
    D = (−8) 2 − 4 · 1 · 12 = 64 − 48 = 16

    Итак, дискриминант положительный, поэтому уравнение имеет два различных корня. Аналогично разбираем второе уравнение:
    a = 5; b = 3; c = 7;
    D = 3 2 − 4 · 5 · 7 = 9 − 140 = −131.

    Дискриминант отрицательный, корней нет. Осталось последнее уравнение:
    a = 1; b = −6; c = 9;
    D = (−6) 2 − 4 · 1 · 9 = 36 − 36 = 0.

    Дискриминант равен нулю — корень будет один.

    Обратите внимание, что для каждого уравнения были выписаны коэффициенты. Да, это долго, да, это нудно — зато вы не перепутаете коэффициенты и не допустите глупых ошибок. Выбирайте сами: скорость или качество.

    Кстати, если «набить руку», через некоторое время уже не потребуется выписывать все коэффициенты. Такие операции вы будете выполнять в голове. Большинство людей начинают делать так где-то после 50-70 решенных уравнений — в общем, не так и много.

    Корни квадратного уравнения

    Теперь перейдем, собственно, к решению. Если дискриминант D > 0, корни можно найти по формулам:

    Основная формула корней квадратного уравнения

    Когда D = 0, можно использовать любую из этих формул — получится одно и то же число, которое и будет ответом. Наконец, если D

    1. x 2 − 2x − 3 = 0;
    2. 15 − 2x − x 2 = 0;
    3. x 2 + 12x + 36 = 0.

    Первое уравнение:
    x 2 − 2x − 3 = 0 ⇒ a = 1; b = −2; c = −3;
    D = (−2) 2 − 4 · 1 · (−3) = 16.

    D > 0 ⇒ уравнение имеет два корня. Найдем их:

    Второе уравнение:
    15 − 2x − x 2 = 0 ⇒ a = −1; b = −2; c = 15;
    D = (−2) 2 − 4 · (−1) · 15 = 64.

    D > 0 ⇒ уравнение снова имеет два корня. Найдем их

    \[\begin{align} & {{x}_{1}}=\frac{2+\sqrt{64}}{2\cdot \left(-1 \right)}=-5; \\ & {{x}_{2}}=\frac{2-\sqrt{64}}{2\cdot \left(-1 \right)}=3. \\ \end{align}\]

    Наконец, третье уравнение:
    x 2 + 12x + 36 = 0 ⇒ a = 1; b = 12; c = 36;
    D = 12 2 − 4 · 1 · 36 = 0.

    D = 0 ⇒ уравнение имеет один корень. Можно использовать любую формулу. Например, первую:

    Как видно из примеров, все очень просто. Если знать формулы и уметь считать, проблем не будет. Чаще всего ошибки возникают при подстановке в формулу отрицательных коэффициентов. Здесь опять же поможет прием, описанный выше: смотрите на формулу буквально, расписывайте каждый шаг — и очень скоро избавитесь от ошибок.

    Неполные квадратные уравнения

    Бывает, что квадратное уравнение несколько отличается от того, что дано в определении. Например:

    1. x 2 + 9x = 0;
    2. x 2 − 16 = 0.

    Несложно заметить, что в этих уравнениях отсутствует одно из слагаемых. Такие квадратные уравнения решаются даже легче, чем стандартные: в них даже не потребуется считать дискриминант. Итак, введем новое понятие:

    Уравнение ax 2 + bx + c = 0 называется неполным квадратным уравнением, если b = 0 или c = 0, т.е. коэффициент при переменной x или свободный элемент равен нулю.

    Разумеется, возможен совсем тяжелый случай, когда оба этих коэффициента равны нулю: b = c = 0. В этом случае уравнение принимает вид ax 2 = 0. Очевидно, такое уравнение имеет единственный корень: x = 0.

    Рассмотрим остальные случаи. Пусть b = 0, тогда получим неполное квадратное уравнение вида ax 2 + c = 0. Немного преобразуем его:

    Поскольку арифметический квадратный корень существует только из неотрицательного числа, последнее равенство имеет смысл исключительно при (−c /a ) ≥ 0. Вывод:

    1. Если в неполном квадратном уравнении вида ax 2 + c = 0 выполнено неравенство (−c /a ) ≥ 0, корней будет два. Формула дана выше;
    2. Если же (−c /a )

    Как видите, дискриминант не потребовался — в неполных квадратных уравнениях вообще нет сложных вычислений. На самом деле даже необязательно помнить неравенство (−c /a ) ≥ 0. Достаточно выразить величину x 2 и посмотреть, что стоит с другой стороны от знака равенства. Если там положительное число — корней будет два. Если отрицательное — корней не будет вообще.

    Теперь разберемся с уравнениями вида ax 2 + bx = 0, в которых свободный элемент равен нулю. Тут все просто: корней всегда будет два. Достаточно разложить многочлен на множители:

    Вынесение общего множителя за скобку

    Произведение равно нулю, когда хотя бы один из множителей равен нулю. Отсюда находятся корни. В заключение разберем несколько таких уравнений:

    Задача. Решить квадратные уравнения:

    1. x 2 − 7x = 0;
    2. 5x 2 + 30 = 0;
    3. 4x 2 − 9 = 0.

    x 2 − 7x = 0 ⇒ x · (x − 7) = 0 ⇒ x 1 = 0; x 2 = −(−7)/1 = 7.

    5x 2 + 30 = 0 ⇒ 5x 2 = −30 ⇒ x 2 = −6. Корней нет, т.к. квадрат не может быть равен отрицательному числу.

    4x 2 − 9 = 0 ⇒ 4x 2 = 9 ⇒ x 2 = 9/4 ⇒ x 1 = 3/2 = 1,5; x 2 = −1,5.

    Решение квадратных уравнений различными способами | План-конспект урока по алгебре (8 класс) на тему:

    РЕШЕНИЕ КВАДРАТНЫХ УРАВНЕНИЙ РАЗЛИЧНЫМИ СПОСОБАМИ.

    Обоснование выбора темы урока.

               С примерами применения различных формул учащиеся неоднократно встречаются как на уроках алгебры, геометрии, так и на других уроках.   Накопленный опыт позволяет восьмиклассникам довольно успешно применять для решения уравнений новые формулы, устанавливающие связь между коэффициентами и корнями квадратного уравнения.  

       Задача учителя при изучении темы «Квадратные уравнения» — добиться безусловного усвоения её каждым учащимся, поскольку умение решать квадратные уравнения относится к числу важнейших умений в курсе алгебры  8 класса. Без этого умения учащиеся не смогут усваивать материал следующих тем. Кроме того, умение решать квадратные уравнения необходимо и при решении тригонометрических, логарифмических, иррациональных, показательных уравнений и неравенств в курсе «Алгебра и начала анализа».

            Данный урок является уроком обобщения и систематизации знаний. В ходе его учащиеся предоставляют отчеты по исследовательским работам, которые были проведены учащимися в течение  двух недель; выполняют исследовательскую работу на наличие и величину корней квадратного уравнения в случае, когда сумма коэффициентов равна нулю, т.е. а+в+с=0; устно решают задания. На данном уроке проводится письменный и устный контроль знаний учащихся, заполняется специальная таблица по итогам прослушанных отчетов.         

    Предметные знания

    Обогащение  методологического аппарата   правомерностью использования новых формул для решения квадратных уравнений.

    Предметные умения

    • Учить распознавать уравнения, сумма коэффициентов которых равна нулю.
    • Учить находить корни уравнений с помощью новых формул.

    Межпредметные умения (ОУУН)

    • Учить выделять признаки объекта и на их основе проводить сравнение.
    • Учить выделять по общности признаков группы объектов.                                            
    • Учить выделять классификационный критерий на основе обобщения.
    • Учить формулировать на основе признаков вывод.
    • Учить выполнять умозаключения.

    Развитие речи

    • Учить грамотно читать тексты, записанные на математическом языке.
    • Учить грамотно записывать фразы естественного языка с математическим содержанием на математическом языке.
    • Учить грамматически и логически правильно выражать свои мысли средствами как естественного, так и математического языков.

    Развитие внимания

    • Концентрации.
    • Переключаемости.
    • Мобильности.

    Развитие познавательного интереса

    Развитие интереса к предмету и познавательного интереса через показ красоты и простоты нахождения корней квадратного уравнения новым способом.

    Развитие личностных качеств

    • Целеустремлённости через потребности ставить перед собой цели и достигать их.
    • Чувства собственного достоинства через формирования адекватной самооценки у учащихся.

    Формирование активной жизненной позиции

    • Формирование собственной точки зрения и умения её обосновывать.
    • Формирование собственных способов действий.

    Тема: Решение квадратных уравнений. Способы решения.

    Цель: 1.Актуализировать знания учащихся по изучаемой теме «Решение квадратных    

                уравнений».

               2.Содействовать формированию познавательной деятельности учащихся в

                обобщение способов решения квадратных уравнений.

               3.Способствовать формированию умения выделить главное в изучаемой теме,

                наиболее общее и существенное в способах решения квадратных уравнений.

               4.Содействовать умению осуществлять самоконтроль и самокоррекцию.

    Задачи урока: 1.Обобщить изученные способы решения квадратных уравнений.

                             2.Систематизировать знания учащихся в умении решать квадратные

                             уравнения разными способами.

                             3.Проверить полученные знания средствами информатизации и

                              осуществить самоконтроль.

    Тип урока: Урок обобщения и систематизации знаний, умений, навыков средствами

                  информатизации.

     1. Организационный момент:

    1. Объявление темы и целей урока.

     Сегодня на уроке мы обобщим и систематизируем знания по теме « Квадратные уравнения и способы решения»

    Задачи урока:

    Информация о квадратных уравнениях:

    Квадратные уравнения – это основа, фундамент, на котором покоится величественное здание – алгебра. Квадратные уравнения находят широкое применение при решении тригонометрических, показательных, логарифмических, иррациональных уравнений, с которыми вам предстоит познакомиться в старших классах.

            На уроке мы с вами поговорим лишь о некоторых способах решения квадратных уравнений.

    2. Актуализация знаний учащихся.

       

    1. Обобщающая работа по теме « Квадратные уравнения» Соломатина Алена.
    1. Кроссворд как проверка теоретических знаний учащихся

    КВАДРАТНОЕ

                                              ПРИВИДЕННОЕ

                                      РАВНОСИЛЬНОЕ

                                                   КОЭФФИЦИЕНТ

                                              КОРЕНЬ

                                  УРАВНЕНИЕ

                                        АРИФМЕТИЧЕСКИЙ

                                                ДИОФАНТ

                                      НЕПОЛНОЕ

                                                РАЗЛИЧИТЕЛЬ

                                   СВОБОДНЫЙ

                                           ВИЕТ

    4. Тест как результат обобщения теоретических знаний, полученных из сообщения и устного опроса.

    1. Какое из данных уравнений является квадратным?

    А) 3х2 – 4х = 6х3    б) 6х – 4 = 5 – 3х    в) 4х2 – 3х + 2 = 0      г) 4/х = х + 1

    1. Назовите коэффициенты квадратного уравнения -4х2 + х -5 = 0

               А) а=4   в=1   с=5    б) а= -4   в = 1  с= 5    в) а=-4  в =х  с= -5     г) а=-4  в=1  с= -5

    1. Уравнение 2х2 – 4х +6 = 0 сделайте приведенным:

    А) х2 – 4х + 6 = 0    б) х2 – 2х + 3 = 0  в) –х2 + 4х – 6 = 0

               4.   Найдите дискриминант квадратного уравнения 2х2 -4х +1 = 0

               а) Д=24   б) Д=-8   в)  Д=8    г) Д=16

         

               5.   Определите количество корней квадратного уравнения из номера 4

                а) 2 корня    б) 1 корень    в) нет корней

               6.   Из данных уравнений выберите неполные квадратные уравнения

                а) х2 – 4х + 3 = 0  б) х2 – 5х= 0    в) х2 = 0     г) 2х – 3х2 +5 = 0

               

    Ответы:              1     2    3     4     5    6

                               В     г     б     в     а    б, в

     

            5. Исследовательская работа на тему « КАК ЗАВИСЯТ ЗНАЧЕНИЯ КОРНЕЙ КВАДРАТНОГО УРАВНЕНИЯ ОТ ВЫБОРА СПОСОБА РЕШЕНИЯ ГРАФИЧЕСКИ» (Миронов Илья)

    7. Самостоятельная работа

    1) решите уравнение алгебраически   а) 3х2 – 4 х=0  б) 6х2=0

    2) Решите уравнение графически  х2 – 4х+3=0

    Ответы:  а) 0 и 4/3  б) 0    2) 3 и 2

    8. Исследовательская работа на наличие и величину корней уравнения в случае, когда а+в+с=0.

    •   Упражнение 1.. Даны квадратные уравнения  
    • 1) х2-5х+1=0;    
    • 2) 9х2-6х+10=0;
    • 3) х2+2х-2=0;    
    • 4) х2-3х-1=0;    
    • 5) х2+2х-3=0;      
    • 6) 5х2-8х+3=0;        

    Найти сумму коэффициентов этих уравнений. ( ПРЕЗЕНТАЦИЯ « СВОЙСТВО КОРНЕЙ УРАВНЕНИЯ»)

    Сумма коэффициентов

    • 1-5+1=-3.
    • 9-6+10=13.
    • 1+2-2=1.
    • 1-3-1=-3.
    • 1+2-3=0.
    •  5-8+3=0.

    Упражнение 2.  Даны 4 уравнения  х2+4х-5=0    3х2+3х-6=0   5х2-8х+3=0  -7х2+2х+5=0

    Найти сумму коэффициентов и решить одно из них.  Сделать вывод о величине корней.

    ВЫВОД: Если сумма коэффициентов  квадратного уравнения равна 0, то

                   Один из корней равен 1, а другой вычисляется по формуле  х= с / а

    ДОМАШНЕЕ ЗАДАНИЕ: исследовать корни квадратного уравнения в случае, когда

    а —  в + с = 0.

    Доп. Задание:

             

    7. Самостоятельная работа

    1) решите уравнение алгебраически   а) 3х2 – 4 х=0  б) 6х2=0

    2) Решите уравнение графически  х2 – 5х+6=0

    7. Самостоятельная работа

    1) решите уравнение алгебраически   а) 3х2 – 4 х=0  б) 6х2=0

    2) Решите уравнение графически  х2 – 5х+6=0

    7. Самостоятельная работа

    1) решите уравнение алгебраически   а) 3х2 – 4 х=0  б) 6х2=0

    2) Решите уравнение графически  х2 – 5х+6=0

    7. Самостоятельная работа

    1) решите уравнение алгебраически   а) 3х2 – 4 х=0  б) 6х2=0

    2) Решите уравнение графически  х2 – 5х+6=0

    7. Самостоятельная работа

    1) решите уравнение алгебраически   а) 3х2 – 4 х=0  б) 6х2=0

    2) Решите уравнение графически  х2 – 5х+6=0

    7. Самостоятельная работа

    1) решите уравнение алгебраически   а) 3х2 – 4 х=0  б) 6х2=0

    2) Решите уравнение графически  х2 – 5х+6=0

    Раздел 2.1: Решение квадратных уравнений

    Результаты обучения

    К концу этого раздела вы сможете:

    • Факторизировать квадратное уравнение для его решения.
    • Используйте свойство квадратного корня для решения квадратного уравнения.
    • Используйте теорему Пифагора и свойство квадратного корня, чтобы найти неизвестную длину стороны прямоугольного треугольника.
    • Заполните квадрат, чтобы решить квадратное уравнение.
    • Используйте квадратную формулу для решения квадратного уравнения.
    • Используйте дискриминант для определения количества и типа решений квадратного уравнения.

    Левый компьютерный монитор на изображении ниже — это 23,6-дюймовая модель, а правый — 27-дюймовая модель. Пропорционально мониторы очень похожи. Если пространство ограничено и нам нужен максимально большой монитор, как нам решить, какой из них выбрать? В этом разделе мы узнаем, как решать такие проблемы, используя четыре различных метода. 9{2}-4=0[/latex] — квадратные уравнения. Они бесчисленным образом используются в инженерии, архитектуре, финансах, биологических науках и, конечно же, в математике.

    Часто самым простым методом решения квадратного уравнения является факторизация . Факторинг означает нахождение выражений, которые можно перемножить, чтобы получить выражение на одной стороне уравнения.

    Если квадратное уравнение можно разложить на множители, оно записывается как произведение линейных членов. Решение факторингом зависит от свойства нулевого произведения, которое гласит, что если [латекс]а\cdot b=0[/латекс], то [латекс]а=0[/латекс] или [латекс]b=0[/латекс] , где a и b — действительные числа или алгебраические выражения. Другими словами, если произведение двух чисел или двух выражений равно нулю, то одно из чисел или одно из выражений должно быть равно нулю, потому что ноль, умноженный на что-либо, равен нулю.

    Умножение коэффициентов расширяет уравнение до строки членов, разделенных знаками плюс или минус. Таким образом, в этом смысле операция умножения отменяет операцию факторизации. Например, разложите факторизованное выражение [латекс]\влево(х — 2\вправо)\влево(х+3\вправо)[/латекс], перемножив два множителя. 9{2}+x — 6=0[/latex] имеет стандартную форму.

    Мы можем использовать свойство нулевого произведения для решения квадратных уравнений, в которых мы сначала должны вынести наибольший общий делитель (НОД), а также для уравнений, которые имеют специальные формулы факторизации, такие как разность квадратов, оба из которых мы увидим позже в этом разделе.

    Общее примечание: свойство нулевого произведения и квадратные уравнения текст{ или }b=0[/латекс], 9{2}[/latex], равно 1. У нас есть один метод факторизации квадратных уравнений в этой форме.

    Как: Дано квадратное уравнение со старшим коэффициентом 1, разложить его на множители

    1. Найдите два числа, произведение которых равно c и сумма которых равна b .
    2. Используйте эти числа для записи двух множителей вида [латекс]\влево(х+к\вправо)\текст{ или }\влево(х-к\вправо)[/латекс], где k — одно из найденных чисел на шаге 1. {2}+2х+3х+3[/латекс], а затем разложим каждую часть выражения, чтобы получить [латекс]2х\влево(х+1\). вправо)+3\влево(x+1\вправо)[/латекс]. Затем мы извлекаем GCF [латекс]\влево(х+1\вправо)[/латекс], чтобы найти факторизованное выражение. 9\circ [/latex] угол, а [latex]c[/latex] относится к гипотенузе. Он имеет неизмеримое применение в архитектуре, технике, естественных науках, геометрии, тригонометрии и алгебре, а также в повседневных приложениях.

      Мы используем теорему Пифагора, чтобы найти длину одной стороны треугольника, зная длины двух других. Поскольку каждое из слагаемых в теореме возводится в квадрат, когда мы находим сторону треугольника, у нас получается квадратное уравнение. Мы можем использовать методы решения квадратных уравнений, которые мы изучили в этом разделе, чтобы найти недостающую сторону. 9{\ circ }[/latex] угол, а [latex]c[/latex] относится к гипотенузе.

      Пример 5. Нахождение длины недостающей стороны прямоугольного треугольника

      Найдите длину недостающей стороны прямоугольного треугольника.

      Показать раствор

      Попробуйте

      Используйте теорему Пифагора, чтобы решить задачу о прямоугольном треугольнике: Катет a измеряет 4 единицы, катет b измеряет 3 единицы. Найдите длину гипотенузы.

      Показать раствор

      https://ohm.lumenlearning.com/multiembedq.php?id=48710&theme=oea&iframe_resize_id=mom5

      Заполнение квадратной и квадратной формул

      Не все квадратные уравнения можно разложить на множители или решить в исходной форме с помощью свойство квадратного корня. В этих случаях мы можем использовать метод решения квадратного уравнения , известного как , дополняющего квадрат . Используя этот метод, мы добавляем или вычитаем члены с обеих сторон уравнения, пока у нас не получится идеальный квадратный трехчлен с одной стороны от знака равенства. Затем мы применяем свойство квадратного корня. Чтобы завершить квадрат, старший коэффициент, 9{2}-6x=13[/латекс].

      Показать раствор

      https://ohm.lumenlearning.com/multiembedq.php?id=1384&theme=oea&iframe_resize_id=mom1

      https://ohm.lumenlearning.com/multiembedq.php?id=79619&theme=oea&iframe_resize_id=mom2

      Использование Квадратная формула

      Четвертый метод решения квадратного уравнения заключается в использовании квадратной формулы , которая решит все квадратные уравнения. Хотя квадратная формула работает с любым квадратным уравнением в стандартной форме, легко сделать ошибку при подстановке значений в формулу. Будьте внимательны при подстановке и используйте круглые скобки при вставке отрицательного числа. 9{2}+3x — 2=0[/латекс].

      Показать раствор

      https://ohm.lumenlearning.com/multiembedq.php?id=4014&theme=oea&iframe_resize_id=mom3

      https://ohm.lumenlearning.com/multiembedq.php?id=35639&theme=oea&iframe_resize_id=mom4

      Дискриминант

      Квадратная формула не только порождает решения квадратного уравнения, но и сообщает нам о характере решений, когда мы рассматриваем дискриминант или выражение под радикалом, [латекс]{b}^{2 }-4ac[/латекс]. Дискриминант говорит нам, являются ли решения действительными числами или комплексными числами, а также сколько решений каждого типа следует ожидать. В таблице ниже значение дискриминанта связано с решениями квадратного уравнения. 9{2}-10x+15=0[/латекс]

    Показать решение

    Попробуйте

    https://ohm.lumenlearning.com/multiembedq.php?id=35145&theme=oea&iframe_resize_id=mom6

    Основные понятия

    • Многие квадратные уравнения можно решить с помощью факторизации, если уравнение имеет старший коэффициент 1 или если уравнение представляет собой разность квадратов. Затем свойство нулевого фактора используется для поиска решений.
    • Многие квадратные уравнения со старшим коэффициентом, отличным от 1, могут быть решены путем факторизации с использованием метода группировки.
    • Другим методом решения квадратичных уравнений является свойство квадратного корня. Переменная возводится в квадрат. Мы выделяем квадрат члена и берем квадратный корень из обеих частей уравнения. Решение даст положительное и отрицательное решение.
    • Завершение квадрата — это метод решения квадратных уравнений, когда уравнение нельзя разложить на множители.
    • Очень надежным методом решения квадратных уравнений является формула квадратного уравнения, основанная на коэффициентах и ​​постоянном члене уравнения.
    • Дискриминант используется для указания характера решений, которые даст квадратное уравнение: действительные или комплексные, рациональные или иррациональные, а также их количество.
    • Теорема Пифагора, одна из самых известных теорем в истории, используется для решения задач о прямоугольных треугольниках и имеет приложения во многих областях. Чтобы найти длину одной стороны прямоугольного треугольника, нужно решить квадратное уравнение.

    Глоссарий

    заполнение квадрата
    процесс решения квадратных уравнений, в котором члены добавляются или вычитаются из обеих частей уравнения, чтобы сделать одну сторону правильным квадратом
    дискриминант
    выражение под корнем в квадратной формуле, указывающее на природу решений, действительных или комплексных, рациональных или иррациональных, одинарных или двойных корней.
    Теорема Пифагора
    теорема, устанавливающая соотношение между длинами сторон прямоугольного треугольника, используемая для решения задач прямоугольного треугольника 9{2}[/latex] член изолирован, так что квадратный корень из обеих частей уравнения можно взять для решения для x
    свойство нулевого продукта
    свойство, которое формально утверждает, что умножение на ноль равно нулю, так что каждый множитель квадратного уравнения может быть установлен равным нулю для решения уравнений

     

    Раздел 2.1 Домашнее задание Упражнения

    1. Как распознать квадратное уравнение?

    9{2}+280x-1000[/latex], где [latex]x[/latex] — количество товаров, проданных на аукционе, а [latex]p[/latex] — прибыль, полученная компанией, проводившей аукцион. . Сколько пунктов решения сделает эту прибыль максимальной? Решите это, нарисовав выражение в графической утилите и найдя максимум, используя 2nd CALC max . {2}-0,02А+120[/латекс]. Найдите возраст с точностью до года человека, у которого нормальное артериальное давление равно 125 мм рт. 9{2} + bx + c = 0$, где $a$, $b$, $c$ — вещественные числа и $a \ne 0$. Квадратные уравнения имеют два нуля или корня, которые могут быть действительными числами, мнимыми числами или и тем, и другим.

    Существуют различные методы решения квадратных уравнений. Наиболее популярным методом является решение квадратных уравнений методом факторизации. Давайте разберемся с этими различными методами решения квадратных уравнений на примерах.

    Методы решения квадратных уравнений

    Решение уравнения — это процесс нахождения нулей (или корней) уравнения. Нули (или корни) также известны как решения уравнения. Это значения переменной, удовлетворяющей уравнению.

    Поскольку квадратное уравнение является уравнением степени $2$, оно может иметь максимум $2$ действительных корней, т. е. квадратное уравнение может иметь любой из следующих

    • нулевой действительный корень
    • один действительный корень (или два равных действительных корня)
    • два действительных корня (или два неравных действительных корня)

    Примечание: Термины «нет действительного корня» или «один действительный корень» не означают, что квадратное уравнение может иметь корни $0$ или $1$. Квадратное уравнение всегда имеет $2$ корней, где

    • оба настоящие
    • один реальный и один воображаемый
    • оба мнимые

    Существуют разные способы решения квадратных уравнений. Наиболее распространенные методы:

    • Решение квадратных уравнений с помощью графика
    • Решение квадратных уравнений методом факторизации
    • Решение квадратных уравнений путем заполнения квадрата
    • Решение квадратных уравнений по квадратной формуле

    Помимо этих методов, существуют и другие методы, которые используются только в особых случаях (когда в квадратном уравнении отсутствуют члены), которые также обсуждаются в следующих разделах.

    СКАЧАТЬ БЕСПЛАТНО КАРТОЧКИ ПО МАТЕМАТИКЕ:

    Красиво оформленные карточки для печати, которые помогут вам запомнить все важные математические понятия и формулы.

    Решение квадратных уравнений с помощью графика

    $x$-пересечения уравнения – это точки, в которых его кривая пересекает ось $x$. {2} + bx + c$ 9{2} + 10  = 7x$ означает, что $4$ не имеет действительных корней.

    Решение квадратных уравнений методом факторинга

    Решение квадратных уравнений методом факторизации — один из популярных методов решения квадратных уравнений. Это этапы решения квадратных уравнений методом факторинга.

    Шаг 1: Приведите уравнение к стандартной форме. т. е. получить все члены с одной стороны (обычно с левой стороны) уравнения так, что другая сторона равна $0$

    Шаг 2: 9{2} = \frac {5}{2}$

    Извлечение квадратного корня из обеих сторон

    $=>x + 2 = \pm \sqrt{\frac {5}{2}}$

    $=>x + 2 = \pm \frac{\sqrt{5}}{\sqrt{2}}$

    $=>x + 2 = \pm \frac{\sqrt{10}}{2}$

    $=>x = -2 \pm \frac{\sqrt{10}}{2}$

    $=>x = -2 + \frac{\sqrt{10}}{2}$, $=>x = -2 — \frac{\sqrt{10}}{2}$

    $=> x = \frac{-4 + \sqrt{10}}{2}$, $=> x = \frac{-4 – \sqrt{10}}{2}$

    Решение квадратных уравнений по квадратичной формуле

    Как видно из вышеизложенного, другие методы решения квадратных уравнений имеют некоторые ограничения, такие как метод факторизации полезен только тогда, когда квадратное выражение может быть факторизовано, метод построения графика полезен только тогда, когда квадратное уравнение имеет действительные корни и т. {2} + dx + e = 0$. Степень таких уравнений равна $4$. 9{2} + 12x + 7$

    Часто задаваемые вопросы

    В чем смысл решения квадратных уравнений?

    Решение квадратных уравнений означает нахождение их решений или корней. т. е. это процесс нахождения значений переменной, удовлетворяющих уравнению.

    Какие существуют четыре способа решения квадратных уравнений?

    Четыре способа решения квадратных уравнений:
    a) Решение квадратных уравнений путем построения графика
    b) Решение квадратных уравнений путем факторизации
    в) Решение квадратных уравнений путем заполнения квадрата
    г) Решение квадратных уравнений по квадратной формуле

    Какие способы решения квадратных уравнений наиболее популярны?

    Существуют разные способы решения квадратных уравнений. Но самыми популярными способами являются «решение квадратных уравнений факторингом» и «решение квадратных уравнений по квадратной формуле».

    Заключение

    Решение квадратных уравнений означает нахождение их решений или корней.

    Прямая пропорциональность y = kx свойства и график функции

    1. Определение прямой пропорциональности
    2. График прямой пропорциональности
    3. Примеры

    Определение прямой пропорциональности

    Если машина движется со скоростью 50 км/ч, пройденное расстояние (в километрах) в зависимости от времени (в часах) s = 50t. Время мы определяем как $t\geq0$. Но механика позволяет нам рассчитать не только будущее положение тела, но и прошлое, подставив в формулу $t \lt 0$ и запросто «прокрутив» время назад. Поэтому в общем случае, если движение было и остаётся постоянным, мы получаем:

    $${\left\{ \begin{array}{c} — \infty \lt t\lt + \infty \\ s = 50t \end{array} \right.}$$

    Можно представить себе не только отрицательное время («поход в прошлое»). Ещё проще ввести отрицательные координаты: направо идём – координата растёт, становится положительной, поворачиваем налево – уменьшается, становится отрицательной.

    В задачах, связанных с экономикой, величины также могут уходить в «плюс» и «минус»: покупки/продажи, кредиты/депозиты, доходы/затраты, прибыли/убытки . Часто эти величины изменяются на какую-то постоянную сумму с течением времени.

    Если обобщить формулы, описывающие подобные зависимости, то получаем:

    Если $k \gt 0$, то чем больше x, тем больше y – функция возрастает.

    Если $k \lt 0$, то чем больше x, тем больше y – функция убывает.

    График прямой пропорциональности

    Графиком прямой пропорциональности является прямая, проходящая через начало координат.

    Согласно аксиоме планиметрии, через любые две точки можно провести прямую и притом только одну. Значит, положение прямой на плоскости полностью определяется двумя точками . Получаем:

    Например: построим график функции y = 2x

    Примеры

    Пример 1. Постройте графики прямых пропорциональностей.

    Укажите, возрастает или убывает функция.

    а) y = x

    $k = 1 \gt 0$ – функция возрастает

    б) y = 3x

    $k = 3 \gt 0$ – функция возрастает

    в) $y = \frac{1}{3} x$

    $k = \frac{1}{3} \gt 0$ – функция возрастает

    г) y = -x

    $k = -1 \lt 0$ – функция убывает

    д) y = -2x

    $k = -2 \lt 0$ – функция убывает

    е) $y = — \frac{1}{2} x$

    $k = -\frac{1}{2} \lt 0$ – функция убывает

    Пример 2. Известно, что график прямой пропорциональности проходит через точку A(5;22). Проходит ли этот график через точки B(7;32,4)и C(9;39,6)?

    Точка A определяет коэффициент пропорциональности:

    $$ k= \frac{y_A}{x_A} = \frac{22}{5} = 4,4 $$

    При $x = 7:y = 4,4 \cdot 7 = 30,8 \neq 32,4 \Rightarrow$ B не принадлежит графику.

    При $x = 9:y = 4,4 \cdot 9 = 39,6 \Rightarrow C$ принадлежит графику.

    Пример 3. Является ли прямой пропорциональностью функция, проходящая через точки:

    а) A(1,5;2,75) и B(12;22)

    Найдём коэффициенты пропорциональностей для каждой из точек:

    $$ k_A = \frac{y_A}{x_A} = \frac{2,75}{1,5} \stackrel{\text{ × 4}}{=} \frac{11}{6} = \frac{15}{6} $$

    $$ k_B = \frac{y_B}{x_B} = \frac{22}{12} = \frac{11}{6} = \frac{15}{6} $$

    $k_A = k_B \Rightarrow$ прямая AB $y=1 \frac{5}{6} x$ является прямой пропорциональностью.

    б) A(3;4,5) и B(5;8)

    Найдём коэффициенты пропорциональностей для каждой из точек:

    $$ k_A = \frac{y_A}{x_A} = \frac{2,75}{1,5} = \frac{4,5}{3} = 1,5 $$

    $$ k_B = \frac{y_B}{x_B} = \frac{8}{5} = 1,6 $$

    $k_A \neq k_B \Rightarrow$ прямая AB не является прямой пропорциональностью.

    Рейтинг пользователей

    за неделю

    • за неделю
    • один месяц
    • три месяца

          Помогай другим

          Отвечай на вопросы и получай ценные призы каждую неделю

          См. подробности

          Уравнение прямой

          Уравнение прямой обычно записывается так:

          у = мх + б

          (или «y = mx + c» в Великобритании, см. ниже)

           

          Что это означает?

          Наклон или
          Градиент
          y  значение, когда x=0
          (см. Пересечение Y)

          y = Как далеко

          x = как далеко вдоль

          M = наклон или градиент (насколько крутой линия)

          B = Значение y 8, когда 7788, 9005

          B = значение Y 8888887777777777888877777888888

          B = стоимость 7. 87777778888777777778887778788788

          b = значение 88888

          B . х=0

          Как найти «м» и «б»?

          • b легко: просто посмотрите, где линия пересекает ось Y.
          • м (Уклон) нуждается в расчете:

          м  = Изменение Y Изменение X

          Зная это, мы можем составить уравнение прямой линии:

          Пример 1

          м = 2 1 = 2

          b = 1 (значение y , когда x=0)

          Подставляя это в y = mx + b , получаем:

          y = 2x + 1

          Теперь с помощью этого уравнения мы можем …

          … выберите любое значение для x и найдите соответствующее значение для y

          Например, когда x равно 1:

          y = 2× 1 + 1 = 3

          Убедитесь сами, что x=1 и y=3 на самом деле находится на линии.

          Или мы могли бы выбрать другое значение для x, например 7:

          y = 2× 7 + 1 = 15

          Таким образом, когда x=7, у вас будет y=15

          Положительный или отрицательный наклон?

          Двигаясь слева направо, велосипедист должен P проехать по P положительный наклон:

             

          Пример 2

          м = −3 1 = −3

          b = 0

          Это дает нам:

          y = −3x + 0

          Нуль нам не нужен! Итак:

          y = −3x

           

          Пример 3: Вертикальная линия

          Какое уравнение составляет вертикальная линия?
          Наклон undefined … и где он пересекает ось Y?

          На самом деле это частный случай , и мы используем другое уравнение, не « y =…», а вместо этого используем « x =…».

          Вот так:

          x = 1,5

          Каждая точка на прямой имеет координату x 1,5 ,
          , поэтому ее уравнение x = 1,5

          Вставай и беги

          Иногда употребляются слова «вставать» и «бежать».

          • Подъем — это то, насколько высоко
          • Бежать — это как далеко

          Таким образом, наклон «м» равен:

          м = подъем пробег

          Возможно, вам будет легче это запомнить.

           

          Теперь поиграйте с графиком!

          Вы можете увидеть влияние различных значений м (наклон) и b (пересечение по оси y) в разделе Исследование прямолинейного графика

           

          Другие формы

          Мы смотрели на форму «наклон-пересечение». Уравнение прямой можно записать многими другими способами .

          Другой популярной формой является уравнение точки-наклона прямой линии.

           

          358 359 517 518, 1156, 1157, 3204, 3205, 3206, 3207

           

          Сноска

          Страна Примечание:

          В разных странах учат разным «обозначениям» (присланным мне добрыми читателями):

          В США, Австралии, Канаде, Эритрее, Иране, Мексике, Португалии, Филиппинах и Саудовской Аравии обозначение: у = мх + б
          В Великобритания, Австралия (также), Багамы, Бангладеш, Бельгия, Бруней, Болгария, Кипр, Египет, Германия, Гана, Индия, Индонезия, Ирландия, Ямайка, Кения, Кувейт, Малайзия, Малави, Мальта, Непал , Новая Зеландия, Нигерия, Оман, Пакистан, Перу, Сингапур, Соломоновы острова, Южная Африка, Шри-Ланка, Турция, ОАЭ, Замбия и Зимбабве у = мх + с
          В Афганистан, Албания, Алжир, Бразилия, Китай, Чехия, Дания, Эфиопия, Франция, Ливан, Нидерланды, Косово, Кыргызстан, Норвегия, Польша, Румыния, Южная Корея, Суринам, Испания, Тунис и Вьетнам Имя: у = топор + б
          В Азербайджан, Китай, Финляндия, Россия и Украина : у = кх + б
          В Греция : ψ = αχ + β
          В Италия : у = мх + q
          В Япония : у = мх + д
          В Куба и Израиль : у = мх + п
          В Румыния : у = гА + С
          В Латвии и Швеции : у = кх + м
          В Сербия и Словения : у = кх + п
             
          В вашей стране: дайте нам знать!

          .